Sie sind auf Seite 1von 215

Sundaram & Das: Derivatives - Problems and Solutions . . . . . . . . . . . . . . . . . . . . . . . . . . . . . . . .

88
Chapter 7. Options Markets
1. What is the dierence between an American option and a European option?
Answer: A European option may be exercised only at maturity, whereas an American
option may be exercised at any time prior to maturity and at maturity.
2. Explain the following terms in the context of options: long, short, call, put, American,
European, in-the-money, out-of-the-money, at-the-money, strike, holder, buyer, writer,
seller, expiry, premium, over-the-counter, and exchange-traded.
Answer: The denitions are as follows:
(a) Long = a bought position in a security.
(b) Short = a sold position in a security. A short is a net sold position in a security,
and the term is usually used when you sell a security when you do not own it. To
do so, you need to borrow the security and sell it.
(c) Call = a contract that provides the holder the right but not the obligation to buy an
underlying asset at a prespecied maturity for a prespecied price. For specicity,
we take the underlying asset to be a stock throughout this question.
(d) Put = a contract that provides the holder the right but not the obligation to sell
a stock at a prespecied maturity for a prespecied price.
(e) American option = an option that may be exercised on or before the maturity date.
(f) European option = an option that may only be exercised at maturity.
(g) In-the-money = an option where the current stock price is such that the instrinsic
value is positive. In the case of calls, if the option is in-the-money, the current
stock price S is greater than the strike priceK (i.e., we have S K > 0), and it
is the reverse for puts (K S > 0).
(h) Out-of-the-money = the reverse notion to in-the-money, i.e., for calls, S K < 0
and for puts, K S < 0.
(i) At-the-money = when the option is neither in- or out-of-the-money. That is,
S = K and the intrinsic value of the option is zero.
(j) Strike = the preset price at which the option may be exercised.
(k) Holder = the owner or buyer of the option.
(l) Buyer = holder.
(m) Writer = the seller of the option.
(n) Expiry = the maturity date of the option. It is the last day at which the option
may be exercised.
Sundaram & Das: Derivatives - Problems and Solutions . . . . . . . . . . . . . . . . . . . . . . . . . . . . . . . . 89
(o) Premium = the payment made by the buyer to the seller of the option. The
premium is commonly paid up-front, but may also be paid at other times if agreed
upon by the buyer and seller.
(p) Over-the-counter = where the option is traded directly between two counterparties,
for example, when a bank sells an option to a corporation.
(q) Exchange-traded = when the option is traded on an exchange.
3. What is a Bermudan option?
Answer: A Bermudan option allows for early exercise but only on a limited set of dates
specied in the contract. Hence, it is an option that is intermediate to a European
option and an American option.
4. Why is being long a put option somewhat analogous to being in a short stock position?
Answer: A put option, like a short stock position, makes money when the price of the
underlying stock decrases. Hence, a put, like a short stock position, is a bearish position
on the underlying stock price.
5. What is the main dierence between a forward and an option?
Answer: A forward contract is the obligation to perform a purchase or sale of an under-
lying asset (e.g., a stock) in the future. An option is the right, but not the obligation,
to buy or sell the asset. An option is evidently more valuable than a forward, and so
involves an up-front payment from option buyer to option seller; a forward, on the other
hand, involves no up-front payment.
6. What is the dierence between over-the-counter (OTC) contracts and exchange-traded
contracts?
Answer: An OTC contract is bilateral and is made directly between two counterparties
with no contracting intermediary. Each counterparty bears the risk of the others default.
In an exchange-traded contract, the he two parties to the trade transact through the
exchange. Each one has the exchange as the eective counterparty because if one
of the counterparties fails to settle at maturity, the exchange takes the place of that
counterparty, thereby protecting both parties to the transaction from counterparty risk.
Sundaram & Das: Derivatives - Problems and Solutions . . . . . . . . . . . . . . . . . . . . . . . . . . . . . . . . 90
7. Make a list of securities that you can think of that contain embedded options. For each
embedded option, state the underlying source of risk.
Answer: Open-ended. Exercise for the reader. Some examples are convertible bonds,
callable bonds, collateralized default obligations, executive compensation contracts, etc.
8. Give an example of an option contract that is both exchange-traded and provided over-
the-counter. Which of these forms is more widely in use? Explain.
Answer: Currency options are traded both over-the-counter and on exchanges, but as
the numbers in Appendix 7A show, the volume in the over-the-counter market far exceeds
the volume traded on exchanges. This is because the major participants in the currency
options market are big banks who nd it less expensive to trade directly with each other
and without an intermediary.
9. Explain why an option is like an insurance contract. How is it dierent from a futures
contract? Can an option, like a future, be used for hedging?
Answer: An option is like an insurance contract because it is the right to a benet, with
no residual obligation. There is a trigger event which makes the option pay o to the
buyer, just as there is one in the case of an insurance contract. If the event does nor
occur then, just as with the insurance contract, no settlement is made. Call options
provide insurance to buyers against price increases and put options provide insurance to
sellers against price decreases.
A forward or futures contract can be used to oset the risk in an underlying position so
that the net position is hedged, i.e., has a riskless cash ow. Static options positions
cannot be used to hedge positions in the underlying in this way because the cash ow
from the combined instruments (option + underlying) is not riskless. However, if a com-
pany has an option-like exposure from its business commitments (for example, because
it oers its customers the choice between date-of-order and date-of-shipment pricing),
options may be used to hedge this exposure.
10. What position in naked options would you adopt if you believe that the price of the
stock is going to drop and the volatility of the stock is going to decrease?
Answer: A short call option. This position benets from both the drop in stock prices
and the drop in volatility.
11. Why do options usually increase in value with volatility? What essential feature of the
gross payo diagram leads to this result?
Sundaram & Das: Derivatives - Problems and Solutions . . . . . . . . . . . . . . . . . . . . . . . . . . . . . . . . 91
Answer: Option values usually increase with volatility because of the asymmetry of the
payo function. As volatility increases, price swings become larger. On one side (where
the option is in-the-money), the larger price swings create larger payos for the option
holder (the option gets thrown more into-the-money by the higher volatility). On the
other side (where the option is out-of-the-money), getting thrown more out-of-the-
money doesnt hurt because the payo is zero anyway.
12. Explain the dierence between the payo and price of an option. Write down the payo
formula for a call option and for a put option. What is the dierence between the
gross and net payos of an option (as widely applied in common usage)? Which
concept do you think is the more useful one for valuing an option? Why?
Answer: The payo of an option is the nal cash ow received at maturity. For a call
option the payo is given by
max[0, S
T
K]
and for a put it is
max[0, K S
T
].
These are known as the gross payos. By common convention, the net payo is deemed
to be the gross payo minus the initial premium (price) paid for the option. However,
this notion is to be taken with a pinch of salt, as the gross payo and premium payments
are made at dierent points in time and the net payo is expressed as their dierence
without making adjustments for the time value of money. Hence, the notion of net
payos has some problems.
The gross payo is an important concept as it deals with cash ows at maturity. Two
dierent options or portfolios that have the same payos at maturity must have equal
prices today, else we could buy the cheaper one and sell the more expensive one and lock
in a prot today. This kind of argument based on gross cashows is at the heart of the
material in subsequent chapters, and is used to analyze no-arbitrage relations between
options (Chapters 9 and 10) and to price options (Chapter 11 and beyond).
13. Draw a gross payo diagram for a short position in a call at strike 100. Also draw the
gross payo diagram for a long position in a put option at the same strike and maturity
as the call. Overlay these plots on the same axis to get an aggregate payo diagram for
the portfolio of call and put. What other security do you know of with the same payo
diagram as this portfolio?
Answer: The payo diagram of a short call and a long put is depicted below here.
Sundaram & Das: Derivatives - Problems and Solutions . . . . . . . . . . . . . . . . . . . . . . . . . . . . . . . . 92
Option Payoffs
-60
-40
-20
0
20
40
60
50 60 70 80 90 100 110 120 130 140 150
Stock Price
P
a
y
o
f
f
Put
Call
The sum of these two contracts results in a negatively sloped line extending from a
positive payo of 50 to a negative payo of -50. This is identical to the payo one
would obtain from a short forward contract at a delivery price of 100.
14. Why does a callable bond contain embedded options? Explain what kind of option this
bond has. Who benets from this option? Based on your answer, is a callable bond
priced higher or lower than a noncallable bond?
Answer: A callable bond may be called by the issuer at specied times (calls may typically
occur at any time, though there is sometimes an initial non-call period). Calling the
bond means the issuer pays the holder a strike price (usually, though not always, the
par value of the bond) and buys the bond back from the holder. If the price at which
the bond is trading exceeds the strike, then there may be sucient economic benet to
the issuer to call the bond. Since the bond is called at a time when its price is high and
the investor only receives the lower strike price as compensation, holding a callable bond
may entail a possible loss to the holder that does not exist in the case of a noncallable
bond. Therefore, the callable bond will trade at a lower price than a noncallable one.
The pricing of callable bonds is described in Chapters 29 and 30.
15. Explain what options exist in a convertible-callable corporate bond.
Answer: The bond has the following embedded options:
(a) Option held by the issuer: the callability feature.
(b) Option held by the bondholder: The right to convert the bond into equity of the
issuing company.
Sundaram & Das: Derivatives - Problems and Solutions . . . . . . . . . . . . . . . . . . . . . . . . . . . . . . . . 93
(c) Other options: Convertibles may also be puttable by the holder to the issuing
company at specied times in the bonds life.
Convertible bonds are analyzed in Chapter 21 where these options and their interactions
are discussed.
16. If you hold a callable bond and the volatility of interest rates increase, what do you think
usually happens to the value of your bond?
Answer: The present value of the cash ows from a bond depends on the level of
interest rates. If interest rate volatility rises, it increases the variability of bond values,
so increases the value of the call option held by the issuer and reduces the value of the
bond to the investor.
17. If you hold a convertible bond and the volatility of equity prices declines, what is the
eect on bond value, assuming nothing else changes?
Answer: The reduction in equity volatility causes a fall in the value of the conversion
option (which is like a call option on the underlying stock), and therefore, the value of
the bond to the investor falls.
18. A quanto (quantity) option is one in which the option contains price risk from two
sources. Quantos are discussed in Chapter 18 on path-indendepent exotic options.
An example is where you buy a put option on the Nikkei stock index (which is yen
denominated), but the strike price of the option is stated in dollars. Explain what the
dierent sources of risk in such an option might be. For each source of risk, state in
which direction it must move for the value of the option to increase.
Answer: The Nikkei put with dollar strike has the following risk underlyings:
(a) The level of the Nikkei index. The option increases in value when the Nikkei index
falls, all else remaining the same.
(b) The level of the Yen/USD exchange rate. Since the strike is xed in USD, the
payos in Yen on option maturity are converted into USD at the rate prevailing at
the maturity of the option, and then compared to the strike. If the USD appreciates
relative to the Yen, it will result in a depreciation in value in USD of the Nikkei
index, meaning that the put option will be worth more. Hence, the option increases
in value when the Yen/USD exchange rate increases, ceteris paribus.
(c) The correlation between the Nikkei index and the Yen/USD exchange rate. If the
Nikkei falls and the Yen/USD appreciates, then from the arguments in (a) and
(b) above, the option will increase in value on both counts. Hence, a decrease
Sundaram & Das: Derivatives - Problems and Solutions . . . . . . . . . . . . . . . . . . . . . . . . . . . . . . . . 94
in correlation between the Nikkei and the Yen/USD exchange rate will cause an
increase in this options value.
19. A European investor in the US equity markets wants to buy a quanto call on the S&P 500
index, where the strike is written in euros. (See the previous question for the denition
of a quanto). Can you explain why the investor wants such an option? Also explain
what risks the investor is hedging by buying a quanto call on the equity index rather
than a plain call on the S&P 500.
Answer: A European investor might want to eliminate the risk of exchange rates when
buying the U.S. equity index. Notice that if the U.S. equity index appreciates, but the
euro appreciates as well, then some of the gains from the equity position may be lost
when translating back the USD gains into euros. Hence, the risk that the investor is
hedging is that of the appreciation in the euro relative to the USD.
20. If you are manufacturing consumer products that use oil-based chemicals as inputs, then
you are subject to oil price risk. Suppose you order your oil from Saudi Arabia and
usually pay for it in Saudi rials. You are now concerned that the appreciation in the rial
will aect your protability.
(a) How would you use forward contracts to hedge the risk of your oil purchases?
(b) What type of quanto option would you like to buy to hedge this risk? (See Question
18 for the denition of a quanto.)
Answer: There are two risks here. (i) The risk of oil price increases in Saudi rials, and
(ii) the risk that the Saudi rial increases in value relative to the dollar so that when the
time comes to pay for the oil, the dollar cost increases even if the oil price itself has
remained the same.
(a) Using forwards, we would buy a forward contract on oil at a xed rial price. We
would then enter into another forward on the USD/Rial exchange rate to lock in
the USD cost of the oil.
(b) Using quantos, we can hedge both risks by purchasing a call on the oil at a strike
price in USD.
21. Employee stock options have additional risk over and above standard call options in
that the employee may not be able (or allowed) to cash in the option in the event of
termination of the employees job with the rm if the option is not vested. But if the
Sundaram & Das: Derivatives - Problems and Solutions . . . . . . . . . . . . . . . . . . . . . . . . . . . . . . . . 95
option is vested, and immediate exercise in the event of termination is possible, should
it be worth as much as a standard American option on the rms stock price? Explain.
Answer: When an employee stock option is vested then it may be exercised by the
employee at any time prior to maturity. Hence, it appears to be exactly like a standard
American option. For many reasons, however, the value of such an option may dier
(from the employees perspective) from a standard American option. One reason is
liquidity: the employee is not allowed to sell the option, only to exercise it. Another is
hedgeability: the employee will typically not be able to hedge the option since hedging
a call requires taking a short position in the stock, and, of course, an employee cannot
short his own companys stock. (We will see the hedging of options using the stock in
Chapter 11 and beyond, but it is intuitive that to oset the risk from a call, which is
a bullish instrument, one needs to have a short position in the underlying.) Yet other
reasons may exist. For instance, if the employee is required to exercise the option upon
leaving the rm, this results in the premature exercise of the option if the employee
chooses to leave.
22. Market timers are traders who vary their allocation between equity and bonds so as to
optimize the performance of their portfolios by trading o one market versus the other.
Rather than physically trade in the two markets, you want to avail yourself of the best
return from the bond or stock markets over the next year using an option. Suggest an
option that will provide you this result. (Feel free to dene the options terms.) What
factors drive the value of this option?
Answer: An option that oers as payo the better of the return of the stock and bond
market is known as a maximum option. This option returns the maximum of two
variables. The value of this option depends on the returns characteristics of each market
as also on their correlation. The pricing of maximum options is discussed in Chapter 18.
Sundaram & Das: Derivatives - Problems and Solutions . . . . . . . . . . . . . . . . . . . . . . . . . . . . . . . . 96
Chapter 8. Option Payos & Trading Strategies
1. Draw the payo diagram for the following portfolio of options, all with the same maturity:
(a) long a call at strike 75, (b) long two calls at strike 80, and (c) long three calls at
strike 85. What is the view of the stock price change consistent with this portfolio?
Answer: The plot of the three options as well as the combined portfolio is below:
Option Payoffs
-50
0
50
100
150
200
250
300
350
400
450
70 75 80 85 90 95 100 105 110
S(T)
75 Call
2 80 Calls
3 85 Calls
Total
As can be seen from this payo diagram, the payo grows slowly at rst between 75
and 80, faster between 80 and 85, and then even faster after that. The view taken here
is that there will be explosive growth in the stock price.
2. You are interested in creating the following gross payo prole using an options portfolio:
Stock price 60 70 80 90 100 110 120 130 140 150
Payo 10 30 20 10 0 10 30 10 0 10
What options, at what strikes, would you hold in your portfolio? Assume that the desired
payos are zero for any stock price less than 50 or greater than 160.
Answer: We can set up this entire payo prole using only calls. The following table
presents the options required and their quantities:
Stock price: 50 60 70 80 90 100 110 120 130 140 150
Quantity: 1 1 3 0 0 2 1 4 1 0 0
Sundaram & Das: Derivatives - Problems and Solutions . . . . . . . . . . . . . . . . . . . . . . . . . . . . . . . . 97
The reader is encouraged to check these numbers to make sure that the quantities of
call options held do indeed result in the correct payos at the various terminal price
points. For more details and a verication, see the answer to the next question.
3. (Dicult) Using the principles of the previous question, create a spreadsheet-based
algorithm to generate an option portfolio for any target gross payo prole, such as the
one in the previous question. Assume, as in the previous question, that option payos
are provided for stock prices taken at regular intervals (e.g., intervals of $10). Create a
table for the given problem on a spreadsheet and then use solver to nd the solution.
Answer: This is not hard to do on a spreadsheet, and the solution may be obtained
using solver. The above question has been solved using a spreadsheet and the solution
screen and approach are described in the following screen shot:
Sundaram & Das: Derivatives - Problems and Solutions . . . . . . . . . . . . . . . . . . . . . . . . . . . . . . . . 98
The rst column shows the stock price at maturity. For each of these stock prices we
compute the payos for all the options we hold. Columns 2-11 contain the payos for
each option at increasing strikes from 50 to 150. The cell formulas multiple the payo
per single option by the number of options held for each strike. The number of options
held is stored in the row titled Number. This row is where we choose the number
of options of each strike to hold. This is the row of numbers that the solver function
in Excel chooses so as to make the payo prole in the second last column match the
target prole in the last column. To do this we use solver and specify that the solution
should be integer-valued. A simple spreadsheet can be designed in this way to allow you
Sundaram & Das: Derivatives - Problems and Solutions . . . . . . . . . . . . . . . . . . . . . . . . . . . . . . . . 99
to generate the required holdings of options for any payo prole.
4. You are managing a separate portfolio dedicated to your retirement income. You do
not wish to take excessive risk, and would prefer to limit the downside. What common
option structure would suce?
Answer: A protective put. For example, you may choose to buy a put on the stock
market index for a notional amount that covers the value of your portfolio. This option
will limit your downside on the portfolio.
5. What gross payo prole do you get if you short a covered call position and go long a
protective put position? Would you pay or receive net premiums on this position? What
is the view taken on the movement of the stock price if you hold this position? What
other options strategy does your portfolio remind you of? Assume a common strike for
all options of $100.
Answer: A protective put is long the underlying and long a put. A covered call is long
the underlying and short a call. If we are long the protective put and short the covered
call, the portfolio is just long the put and the call. If the call and put have the same
strikes, we have a straddle.
The following gure illustrates. The gross payo prole is presented in the gure below
assuming all options are of strike 100.
Long Protective Put, Short Covered Call
-150
-100
-50
0
50
100
150
200
0 20 40 60 80 100 120 140 160
S(T)
P
a
y
o
f
f
Short Cov Call
Long Prot Put
Total
Straddle
Sundaram & Das: Derivatives - Problems and Solutions . . . . . . . . . . . . . . . . . . . . . . . . . . . . . . . 100
The implicit view in this portfolio is of being bullish on volatility with no view on direction.
6. If you had a view opposite to that taken in the previous question, what portfolio structure
of options would you choose?
Answer: We can go short a straddle, i.e., sell a call and a put. A short position in
a buttery spread also has a similar payo to a short straddle for stock prices in the
region between the lowest and highest strike prices, so this is another possibility. Other
possibilities too exist; for instance, we could go long the underlying, short two calls with
strike K, and borrow PV (K). As the reader may check, this portfolio leads to the same
payo as a short straddle.
7. Microsoft is currently trading at $26. You expect that prices will increase but not rise
above $28 per share. Options on Microsoft with strikes of $22.50, $25.00, $27.50, and
$30 are available. What options portfolio would you construct from these options to
incorporate your views?
Answer: You would create a bullish vertical spread by selling the call at a strike of 27.50
and buying the call at a strike of 25.00. This way you do not pay for the appreciation
above 28, however, you collect any appreciation between 26 and 27.50.
An alternative is to sell the 30 strike call instead of the 27.50 call. This is more expensive
since the short 30-call brings in less premium than the short 27.50 call (put dierently,
you are now also paying for the range from 28 to 30, which you do not believe will
occur). But it builds in some error in the upper bound of the view on the upside.
8. Suppose your view in the previous question were instead that Microsofts shares will fall
but a fall below $22 is unlikely. Now what strategy will you use?
Answer: Here a bearish vertical put spread would be used, constructed by purchasing
the 25 strike put, and selling the 22.50 put. This will capture gains if the stock falls
from its current price of 26 all the way to 22.50.
9. Calls are available on IBM at strikes of 95, 100, and 105. Which should cost more, the
95100 bullish vertical spread, or the 95100105 buttery spread?
Answer: The price of the 95-100 bullish vertical spread is C(95)C(100), where C(K)
is the price of a call on IBM with strike K. The price of the 95100105 buttery spread
is
C(95) 2 C(100) +C(105) = [C(95) C(100)] [C(100) C(105)]
Sundaram & Das: Derivatives - Problems and Solutions . . . . . . . . . . . . . . . . . . . . . . . . . . . . . . . 101
The rst bracket is the 95100 bullish vertical spread, and the second one is a 100105
bullish vertical spread. Both are positive valued. Hence, the buttery spread must be
worth less than the 95100 bullish vertical spread.
10. A bullish call spread is bullish on direction. Is it also bullish on volatility?
Answer: Not necessarily. It depends on what the stock price is. For an intuitive expla-
nation, look at the payo diagram of a 95100 call bull spread. Near a stock price of 95,
the payo from the spread looks locally convex (it looks similar in that neighborhood
to the payo of a long call), so the spread reacts positively to increases in volatility.
But around a stock price of 100, the payo from the spread looks locally like it has
negative convexity (i.e., as if it comes from a short option position), so reacts negatively
to volatility.
Put more technically, at-the-money options are, as we explain in Chapter 17, more
sensitive to volatility than ones away from the money. So if the stock price is near 95,
the 95-strike option we are long is more sensitive to volatility than the 100-strike option
we are short, and the portfolio increases in value with volatility. If the strike price is near
100, the 100-strike option we are short is more sensitive and the position loses value as
volatility increases.
11. What is the directional view in a long put buttery spread?
Answer: A long position in a put buttery spread involves the view that the price of the
underlying at maturity will be around the middle strike price with very low volatility. If
the middle strike price is, for example, equal to the current price of the underlying, the
view is that the underlying price will not move much from its current level.
12. How would your answer to the previous question change if this buttery spread were
constructed using calls instead?
Answer: The payo from a call buttery spread is identical to the payo from a put
buttery spread, so there is no change in the answer. (The equivalence between call and
put buttery spreads can be derived using put-call parity which is discussed in Chapter
10, but a direct verication is easy.)
13. How does a horizontal spread exploit time-decay of options?
Answer: Horizontal or calendar spreads are pairs of calls (or puts) with the same strike
but dierent maturities. If you buy a T
2
maturity call and sell a T
1
maturity call, where
T
2
> T
1
, then you are long a calendar spread. It is known that time decay is the
Sundaram & Das: Derivatives - Problems and Solutions . . . . . . . . . . . . . . . . . . . . . . . . . . . . . . . 102
greatest when an option is closer to maturity than when it is far from maturity. Hence,
in a calendar spread the short position in the T
1
call decays faster than the long position
in the T
2
call, which is advantageous to the holder of the horizontal call spread.
14. What is the volatility view implied by a long horizontal call spread? What about a short
horizontal put spread?
Answer: If you hold a long horizontal call spread and volatility rises, the position gener-
ally appreciates in value since the increase in volatility increases the value of the longer
maturity option more than that of the shorter maturity one. (There is more time for
the increased volatility to have an impact with a longer-dated option.) So the position
is implicitly bullish on volatility. The opposite view is true of the short horizontal put
spread.
15. Assume the current volatility of oil is high. What options portfolio oers you a gain
from the high volatility if you do not have a view on direction?
Answer: One strategy is to hold a straddle, which is a long position in a call plus a put
on the price of oil. If both options are struck at the money, then such a position gains
when the price of oil moves, irrespective of direction. Note however that straddles are
not cheap, especially when volatility is high, so substantial moves may be required to
make the position protable after taking into account the initial cost.
16. You are planning to trade on the fortunes of a biotech rm that has a drug patent
pending FDA approval. If the patent is approved, the stock price is expected to go up
sharply. If it is not approved, the stock will drop sharply, In your view, it is unlikely to
move more than 20% in either direction. Describe a portfolio combining straddles and
strangles that takes advantage of your view.
Answer: Suppose (for example) that the stock price of the rm is currently at 100.
Then we buy a straddle, at strike 100, i.e., C(100) + P(100). Since we dont expect
movement below 80 or above 120, we also sell a call at 120 and sell a put at 80, i.e., we
sell an 80-120 strangle. The sale of the strangle subsidises the purchase of the straddle.
17. Firm A is likely to be the target in a takeover attempt by rm B. The stock price is
likely to rise over the next few weeks as the takeover progresses, but if it fails, the stock
price of A is likely to fall even more than the rise. What option strategy might exploit
this information?
Answer: In such a case, since the resulting payos on the upside and downside are
asymmetric, the purchase of a strip option is appropriate. A strip is a long position in
Sundaram & Das: Derivatives - Problems and Solutions . . . . . . . . . . . . . . . . . . . . . . . . . . . . . . . 103
puts and calls, where there are more puts than calls. In our case, the possible downside
is more than the upside, and hence a strip is useful.
18. The options for Microsoft (stock price $25.84) are trading at the following prices:
Strike Calls Puts
22.50 3.40 0.10
25.00 1.25 0.30
27.50 0.15 1.80
State the trading ranges at maturity in which the net payo of the following option
positions is positive: (a) 25.00 straddle, (b) 22.50 strip, (c) 27.50 strap, and (d) 22.50
27.50 strangle.
Answer: (a) The cost of the 25.00 straddle is 1.25 + 0.30 = 1.55. Hence, the option
position will be protable if the stock lies outside the 25 1.55 range at maturity.
(b) The cost of the 22.50 strip is 2 0.10 + 3.40 = 3.60. Hence, the option position
pays o when the stock range is below 22.50 1.80 and when it is above 22.50 +3.60.
(c) The cost of the 27.50 strap is 2 0.15 + 1.80 = 2.10. Hence, the option position
pays o when the stock range is below 22.502.10 and when it is above 22.50+2.10/2.
(d) The cost of the 22.50-27.50 strangle is 0.10 + 0.15 = 0.25. Hence, the option
position pays o when the stock range is below 22.50 0.25 and when it is above
22.50 + 0.25.
19. What are collars? What is the investors objective when using a collar?
Answer: A collar is a long position in a put with some strike (say, K
1
) and a short
position in a call with a higher strike K
2
. Collars are commonly combined with positions
in the underlying. The purchase of the put protects the holder of the collar from a fall
in the price of the underlying; the sale of the call subsidizes the purchase of the put at
the cost of putting a ceiling on the maximum payo from the portfolio (the collar plus
the underlying).
20. Is the price of a collar positive, zero or negative?
Answer: It could be any of the three possibilitiesit depends on whether the purchased
put is more or less expensive than the sold call. For example, in cashless collars, the
call and put prices exactly oset each other (the strike prices are chosen carefully to
ensure this), so have zero initial cost.
Sundaram & Das: Derivatives - Problems and Solutions . . . . . . . . . . . . . . . . . . . . . . . . . . . . . . . 104
21. Suppose options trade at two strikes: K
1
< K
2
. You notice that whereas C(K
2
)
P(K
2
) = S PV (K
2
) (put-call parity) holds for the K
2
strike option, it does not
hold for the K
1
strike option, specically C(K
1
) P(K
1
) = S PV (K
1
) + , where
> 0. Show how you would use a box spread to take advantage of this situation by
constructing a risk-less arbitrage strategy. Assume there are no dividends.
Answer: A box spread is the dierence of two collars. We will buy the K
1
collar and
sell the K
2
collar. When this is done, we realize the following cash-ow:
[C(K
1
) P(K
1
)] [C(K
2
) P(K
2
)] = [S PV (K
1
) +] [S PV (K
2
)]
= PV (K
2
) PV (K
1
) + > 0
Keep dollars aside and invest the proceeds PV (K
2
)PV (K
1
) > 0 till option maturity
T.
At maturity, there are three broad possible outcomes, and we construct a table to
consider the payos of the box spread held by the counterparty:
Individual option S
T
< K
1
K
1
S
T
< K
2
K
2
S
T
C(K
1
) 0 S
T
K
1
S
T
K
1
P(K
1
) (K
1
S
t
) 0 0
C(K
2
) 0 0 (S
T
K
2
)
P(K
2
) K
2
S
T
K
2
S
T
0
Sum K
2
K
1
K
2
K
1
K
2
K
1
We can see that the payo we will owe on this position is exactly what we will earn back
at maturity when we invest PV (K
2
) PV (K
1
). Hence, at maturity our cash ow is
net zero, but at inception we collect for free, indicating that we capture that sum in
the arbitrage strategy.
22. What is a ratio spread? Construct one to take advantage of the fact that you expect
stock prices S to rise by about $10 from the current price but are not sure of the
appreciation of more than $10.
Answer: A ratio spread is like a bull or bear spread except the number of options of the
long and short positions is unequal. In this case, if the current stock price is S, then we
should buy two calls at K
1
= S and sell one call at K
2
= S + 10. Hence, this takes
advantage of the view that stock prices will rise and if they do rise more than $10, one
call still remains to take advantage of it. In a standard bull spread, the long and short
positions would be equal and then no upside more than $10 would be availed of.
Sundaram & Das: Derivatives - Problems and Solutions . . . . . . . . . . . . . . . . . . . . . . . . . . . . . . . 105
23. Can the cost of a ratio spread be negative?
Answer: Yes, it depends on the number and prices of the calls that are bought and sold.
24. What is more expensive to buy: (a) a 100110120 buttery spread using calls or (b) a
90100110120 condor? Can you decompose condors in any useful way?
Answer: The cost of the buttery spread is as follows:
C(100) 2C(110) +C(120)
The cost of the condor is
C(90) C(100) C(110) +C(120)
If we subtract the buttery spread from the condor we get
C(90) 2C(100) +C(110)
which is the same as a 90100110 buttery spread. Since all buttery spreads are
positive in price, the condor costs more than the buttery spread. Also, notice that the
condor is the sum of two buttery spreads. This is a buttery spread decomposition of
condors.
25. If you are long futures and long a straddle, what is your view on direction? On volatility?
Answer: You are hoping that there will be a price increase (the straddle makes money
regardless of the direction of the price move, but the long futures only makes money
from an increase in the price). You are also hoping for high volatility.
26. How would your answer to the previous question change if you were short futures instead?
Answer: Now you are bearish in direction, but you are still bullish on volatility.
27. If you take the view that volatility will drop over the next three months and then increase
thereafter, what options strategy would you like to execute? Would the value of this
portfolio today be positive or negative?
Answer: If you are condent about your view, you could short a call today (for example,
a three-month call) to bet on the declining volatility. If, after three months, you still feel
volatility is going to go up from there, you could buy a three-month call at that point.
(Since no view on direction is given in the question, we could also employ puts instead
of calls; or we could set up straddles.)
Sundaram & Das: Derivatives - Problems and Solutions . . . . . . . . . . . . . . . . . . . . . . . . . . . . . . . 106
28. Compute the gross payos for the following two portfolios in separate tables:
Calls (strikes in parentheses): C(90) 2C(100) +C(110).
Puts (strikes in parentheses): P(90) 2P(100) +P(110).
What is the relationship between the two portfolios? Can you explain why?
Answer: The payos for the two portfolios are as follows:
Portfolio of Calls
S(T) C(90) -2C(100) C(110) Total
50 0 0 0 0
60 0 0 0 0
70 0 0 0 0
80 0 0 0 0
90 0 0 0 0
100 10 0 0 10
110 20 -20 0 0
120 30 -40 10 0
130 40 -60 20 0
140 50 -80 30 0
150 60 -100 40 0
Portfolio of Puts
S(T) P(90) -2P(100) P(110) Total
50 40 -100 60 0
60 30 -80 50 0
70 20 -60 40 0
80 10 -40 30 0
90 0 -20 20 0
100 0 0 10 10
110 0 0 0 0
120 0 0 0 0
130 0 0 0 0
140 0 0 0 0
150 0 0 0 0
As we can see, the payos are the same in both tableaus. The reason for this is that
the two positions are equivalent given put call parity (put-call parity is discussed later
in Chapter 10). Since each call is equal to a put plus the stock price minus the present
value of the strike price (per put-call parity), then the values of the stock minus strike
present value cancel out given that there are two long option positions and two short
Sundaram & Das: Derivatives - Problems and Solutions . . . . . . . . . . . . . . . . . . . . . . . . . . . . . . . 107
option positions in each portfolio. Hence, we get a buttery spread payo in each case,
irrespective of whether we use puts or calls.
29. Draw the payo diagrams at maturity for the following two portfolios:
A: Long a call at strike K and short a put at strike K, both options for the same
maturity.
B: Long the stock plus a borrowing of the present value of the strike K. The
payo of this portfolio is the cash-ow received at maturity from an unwinding of
the positions in the portfolio.
Compare your two payo diagrams and explain what you see.
Answer: The payo diagrams for both cases are as follows:
The payo at maturity of Portfolio A (the long call and short put) is as follows:
max(0, S
T
K) max(0, K S
T
) = S
T
K
At maturity, the long stock position is worth S
T
while the borrowing of PV (K) requires
a repayment of K. Hence, Portfolio B is also worth S
T
K at time T.
Thus the payos to the two positions are identical at maturity. What we have derived
here is just one version of the result known as put-call parity which is explored in greater
detail in Chapter 10.
Sundaram & Das: Derivatives - Problems and Solutions . . . . . . . . . . . . . . . . . . . . . . . . . . . . . . . 108
Chapter 9. No-Abitrage Restrictions on Option Prices
1. What is meant by payout protection? Are options payout protected?
Answer: Payouts are cash dividends on the underlying stock. When a dividend is paid
on a stock, its price falls. In a payout-protected option, the features of the option are
adjusted upon payment of a dividend to leave the value of the option unchanged or
nearly unchanged. Payout-protection often takes on the form of a reduction in the strike
price by the amount of the cash dividend. Options in practice are generally not payout
protected, which means holders of calls generally lose and holders of puts generally gain
from the payment of dividends. But if the dividend payment is large and unexpected
(a threshold of 10% of the stock price is common), exchanges sometimes intervene and
reduce the strike prices of all options by the size of the dividend to negate the eect of
the dividend.
2. How does the payment of an unexpected dividend aect (a) call prices and (b) put
prices?
Answer: The payment of an unexpected dividend results in a drop in the stock price.
Hence, this reduces the value of a call option, but increases the value of a put option.
3. As we have seen, options always have non-negative value. Give an example of a derivative
whose value may become negative.
Answer: When a derivative security requires its holder to participate in an o-market
trade at an unfavorable price, the derivative will generally have negative value. A simple
example is a long position in a forward contract in which the locked-in delivery price
K is greater than the currently-prevailing forward price F for the same contract (see
Chapter 3 for details on valuation of forwards).
4. What are the upper and lower bounds on call option prices?
Answer: The upper bound on the price of a call is the current price S of the underlying:
when the asset can be purchased directly today for S , why pay more than S for the call
which only gives you the right to buy the underlying asset by making a further payment
of K?
Lower bounds for an American call are given by
C
A
max[0, S K, S PV (K) PV (D)]
Sundaram & Das: Derivatives - Problems and Solutions . . . . . . . . . . . . . . . . . . . . . . . . . . . . . . . 109
where PV (K) and PV (D) are the present values of the strike price and dividends
receivable over the calls life. For European calls, the lower bound S K is not valid
because the call cannot be exercised early, so the lower bounds are
C
E
max[0, S PV (K) PV (D)]
5. What are the upper and lower bounds on put option prices?
Answer: Assuming that the price of the underlying security cannot become negative, the
upper bound on an American put price is the strike price K, since that is the maximum
prot that can be earned upon exercise of the put. For a European put, even this
maximum prot can only be earned at maturity, so the upper bound is PV (K).
Lower bounds for an American put are given by
P
A
max[0, K S, PV (K) +PV (D) S]
where PV (K) and PV (D) are the present values of the strike price and dividends
receivable over the puts life. For European puts, the lower bound K S is not valid
because the put cannot be exercised early, so the lower bounds are
P
E
max[0, PV (K) +PV (D) S]
6. What is meant by the insurance value of an option? Describe how it may be measured.
Answer: Holding an option provides an investor with protection against unfavorable price
movements. The value of this protection is called the insurance value of the option. The
insurance value comes from the fact that options contain a right but no obligation to
perform. So the insurance value of an option may be measured as the dierence in value
between the price of the option (which has the right) and the value of a corresponding
forward (which contains an obligation). As explained in Chapter 10, this results in a
decomposition of call values on a non-dividend-paying stock as
C = (S K)
. .
Intrinsic Value
+ (K PV (K))
. .
Time Value
+ Insurance value
The insurance value of a put may be analogously expressed.
Sundaram & Das: Derivatives - Problems and Solutions . . . . . . . . . . . . . . . . . . . . . . . . . . . . . . . 110
7. What does the early-exercise premium measure?
Answer: The early exercise premium measures the value of having the right to exercise
the option before maturity; it is the dierence in price between an American option and
an otherwise identical European option.
8. What is meant by convexity of option prices in the strike price?
Answer: Mathematically, convexity is a curvature property that (for a twice-dierentiable
function) is equivalent to having a non-negative second derivative. Thus, an option pric-
ing function C(K) is convex in the strike price K if
2
C(K)/K
2
0. [Here, in obvious
notation, we are letting C(K) denote the price of an option with strike price K when
all other parameters are held xed.] Equivalently, it also implies that for any two strike
prices K
1
and K
2
, and for any w [0, 1], the function must satisfy
wC(K
1
) + (1 w)C(K
2
) C[wK
1
+ (1 w)K
2
]
In terms of a graph, what this means is that if we plot option prices on the y-axis, as
functions of K on the x-axis, the pricing curve will bulge towards the origin.
9. There are call and put options on a stock with strike 40, 50, and 55. Which of the
following inequalities must hold?
(a) 0.5C(40) + 0.5C(55) > C(50)
(b) (1/3)C(40) + (2/3)C(55) > C(50)
(c) (2/3)C(40) + (1/3)C(55) > C(50)
(d) 0.5P(40) + 0.5P(55) > P(50)
(e) (1/3)P(40) + (2/3)P(55) > P(50)
(f) (2/3)P(40) + (1/3)P(55) > P(50)
Note: This question uses the general form of convexity in the strike for options:
wC(K
1
) + (1 w)C(K
3
) C(K
2
)
and
wP(K
1
) + (1 w)P(K
3
) P(K
2
)
where
w =
K
3
K
2
K
3
K
1
Answer:
Sundaram & Das: Derivatives - Problems and Solutions . . . . . . . . . . . . . . . . . . . . . . . . . . . . . . . 111
(a) Because options are convex in their strike prices, we know that
0.5C(40) + 0.5C(55) > C(47.5).
Since we also know that C(47.5) > C(50), the given inequality is valid.
(b) (1/3)C(40) + (2/3)C(55) > C(1/3 40 + 2/3 55) = C(50). Therefore, this
also holds.
(c) Compared to (b) above, since this expression overweights the more expensive op-
tion, the LHS is even more valuable than in (b); thus, the expression is valid.
(d) This is exactly the same as (a); put options are also convex in their strike prices,
and hence this may be approached in the same way, i.e.,
0.5P(40) + 0.5P(55) > P(47.5) < P(50)
Therefore, this may not be valid.
(e) From convexity, we see that (1/3)P(40) + (2/3)P(55) > P(50), hence valid.
(f) This may not be valid as it overweights the cheaper option compared to (e) above.
(2/3)P(40) + (1/3)P(55) > P(45)
but this may not be greater than P(50).
10. There are call and put options on ABC stock with strikes of 40 and 50. The 40-strike
call is priced at $13 while the 50-strike put is at $12.8. What are the best bounds you
can nd for (a) the 40-strike put and (b) the 50-strike call?
Answer:
(a) We note that for puts, the relation P(K
2
) P(K
1
) K
2
K
1
must hold when
K
1
< K
2
.
P(50) P(40) 50 40 =12.8 P(40) 10 =P(40) 2.8
(b) We note that for calls, the relation C(K
1
) P(K
2
) K
2
K
1
must hold.
C(40) C(50) 50 40 =13 C(50) 10 =C(50) 3.
These are the lower bounds. The general upper bounds are the usual ones: for the
call, it is the stock price and for the put it is the strike price. But here, we have
a bit more information. The right to sell at 40 cannot be more valuable that the
right to sell at 50, so an upper bound on the price of the 40-strike put is 12.80.
and the right to buy at 50 cannot cost more than the right to buy at 40, so an
upper bound on the 50-strike call is 13.
Sundaram & Das: Derivatives - Problems and Solutions . . . . . . . . . . . . . . . . . . . . . . . . . . . . . . . 112
11. The following three call option prices are observed in the market, for XYZ stock:
Type Strike Option Price
Call 50 10
Call 60 7
Call 70 2
Are these prices free from arbitrage? How would you determine this? If they are incorrect,
suggest a strategy that you might employ to make sure prots.
Answer: We can check the convexity condition for this set of options. Under this
condition, the prices of the three options must satisfy the following relationship:
C(50) 2C(60) +C(70) > 0
We can see that this is violated for
10 2(7) + 2 = 2 < 0.
Hence, to arbitrage these prices, we can buy a long 506070 buttery spread. At the
time of inception, the purchase of this portfolio results in an inow of $2 in cash. And
since this is a buttery spread, there are no cash outows (but only possible cash inows)
at maturity.
12. The current price of a stock is $60. The one-year call call option on the stock at a strike
of $60 is trading at $10. If the one-year rate of interest is 10%, is the call price free
from arbitrage, assuming that the stock pays no dividends? What if the stock pays a
dividend of $5 one day before the maturity of the option?
Answer: We check the following no-arbitrage condition, developed in the chapter:
C S PV (K)
We can see that
10 > 60 60/1.10,
so the arbitrage bound is not violated. If dividends are also paid then the condition
changes to:
C S PV (K) PV (D) = 60 60/1.10 5/1.10
which is also satised, and more easily so than when the stock does not pay a dividend.
Sundaram & Das: Derivatives - Problems and Solutions . . . . . . . . . . . . . . . . . . . . . . . . . . . . . . . 113
13. The current price of ABC stock is $50. The term structure of interest rates (continuously
compounded) is at at 10%. What is the six-month forward price of the stock? Denote
this as F. The six-month call price at strike F is equal to $8. The six-month put price
at strike F is equal to $7. Explain why there is an arbitrage opportunity given these
prices.
Answer: First, we compute the forward price, which is
F = Se
rT
= 50e
0.100.5
= 52.564.
We now demonstrate the existence of an arbitrage opportunity. Set up the following
portfolio: Short the call and go long the put option; go long the stock and nance the
purchase by borrowing the required amount at the 10% interest rate. Net cash ow:
+$1. Note that the amount owed on the borrowing at maturity is e
(0.10)(1/2)
50 =
52.564.
At maturity, there are two possibilities
(a) S
T
< 52.564: Here the call expires worthless, but the put pays us 52.564 S
T
.
Also, we sell the stock and get cash-ow S
T
, and repay the borrowing with interest
i.e., 52.564. The net cash-ow is zero.
(b) S
T
> F: In this case, the call is exercised leading to a cash outow of (S
T
52.564)
and the put nishes worthless. We sell the stock for a cash inow of S
T
, and repay
the borrowing with interest i.e., 52.564. The net cash-ow is zero.
With an initial cash inow of $1 and no net cash outows, we have the desired arbitrage.
How did we identify the arbitrage opportunity and the strategy to be employed? If we
go long a put and short a call with a given strike price K, the payo at maturity is
identical to that generated by a short forward position with delivery price K. Thus, the
cost of the long put-short call portfolio should be identical to the value of a forward
contract with delivery price K. In particular, if we set the delivery price K to be the
current forward price, the value of the forward contract is zero, so the long put-short
call portfolio should also cost zero (i.e., the call and put should have the same price).
Since the portfolio costs 1, it is undervalued, so we buy the portfolio and sell the short
forward, i.e., buy a forward.
14. The prices of the following puts P(K) at strike K are given to you:
P(40) = 2, P(45) = 6, P(60) = 14.
The current stock price is $50. What is inconsistent about these prices? How would
you create arbitrage prots?
Sundaram & Das: Derivatives - Problems and Solutions . . . . . . . . . . . . . . . . . . . . . . . . . . . . . . . 114
Answer: These prices may be shown to violate the convexity condition. Note that the
middle strike price of 45 is the weighted sum of the surrounding strikes with weights 3/4
and 1/4 respectively, i.e.,
45 =
3
4
40 +
1
4
60
Hence, by the convexity result for options, we should have that
P(45) <
3
4
P(40) +
1
4
P(60)
Testing this condition we see that
6 >
3
4
2 +
1
4
14 = 5
hence, there is a violation of convexity. To arbitrage this, we buy a portfolio where
we short P(45) and go long 3/4 of P(40) and 1/4 of P(60). It is easy to show that
the cash-ow from inception of this position is $1 (inow). It may also be shown that
at maturity, this positions cash ows will be non-negative (this is just the asymmetric
buttery spread discussed in the appendix to this chapter). Hence, we get an arbitrage.
15. The price of a three-month at-the-money call option on a stock at a price of $80 is
currently $5. What is the maximum possible continuously compounded interest rate in
the market for three-month maturity that is consistent with the absence of arbitrage?
Answer: We may use the no-arbitrage condition for calls, i.e.,
C S PV (K).
The maximum interest rate R allows the above equation to be an equality, i.e.,
5 = 80 80e
0.25r
Solving this equation, we get that r = 0.25815, or 25.82%.
16. The six-month continuously compounded rate of interest is 4%. The six-month forward
price of stock KLM is $58. The stock pays no dividends. You are given that the price of
a put option P(K) is $3. What is the maximum possible strike price K that is consistent
with the absence of arbitrage?
Answer: We solve the problem in the following steps:
Sundaram & Das: Derivatives - Problems and Solutions . . . . . . . . . . . . . . . . . . . . . . . . . . . . . . . 115
First, we nd the current stock price, using the forward pricing equation:
F = Se
rT
, i.e., 58 = Se
0.04(0.5)
= Se
0.02
.
This gives us that S = 58e
0.02
.
Next, we invoke the put pricing condition:
P PV (K) S
3 Ke
0.04(0.5)
58e
0.02
.
Solving this inequality we determine that
K 58 + 3e
0.02
= 61.061.
17. (Dicult) Suppose there are ve call options C(K), i.e.,
{C(80), C(90), C(100), C(110), C(120)}. The prices of two of these are C(110) = 4,
C(120) = 2. Find the best possible lower bound for the call option C(80).
Answer: We make use of the following three convexity conditions:
C(80) +C(100) 2C(90)
C(90) +C(110) 2C(100)
C(100) +C(120) 2C(110)
Using these we establish the relationship of C(80) to the two traded calls as follows:
C(80) 2C(90) C(100)
2[2C(100) C(110)] C(100)
= 3C(100) 2C(110)
3[2C(110) C(120)] 2C(110)
= 4C(110) 3C(120)
= 4(4) 3(2)
= 10.
Hence we have that C(80) 10.
Note that you may also solve this problem directly using the convexity formula, but that
would not be as much fun as doing it the way we just did!
Sundaram & Das: Derivatives - Problems and Solutions . . . . . . . . . . . . . . . . . . . . . . . . . . . . . . . 116
18. In the previous problem, also nd the minimum prices of C(90) and C(100).
Answer: This easily follows from the logic used in the preceding question:
C(90) 2C(100) C(110)
2[2C(110) C(120)] C(110)
= 3C(110) 2C(120)
= 3(4) 2(2)
= 8.
Thus, C(90) 8.
The next inequality is:
C(100) 2C(110) C(120)
= 2(4) 2
= 6.
Thus, C(100) 6.
Note that these are only lower bounds, and the actual prices of the calls may be much
higher. The bounds, as one may well imagine, are quite loose.
19. The following are one-year put option prices: the put at strike 90 is trading at $12, and
the put at strike 80 is trading at $2.50. The rate of interest (continuously compounded)
for one year is 10%. Show how you would construct an arbitrage strategy in this market.
Answer: The put prices are in violation of the following required no-arbitrage condition:
P(K
2
) P(K
1
) PV (K
2
K
1
), K
2
> K
1
.
To check that there is an arbitrage, we compute
12 2.50 = 9.50 > (90 80)e
0.10
= 9.0484.
We may set up the following arbitrage strategy:
Sell the put at strike 90.
Buy the put at strike 80.
Invest 9.0484 at 10%.
Sundaram & Das: Derivatives - Problems and Solutions . . . . . . . . . . . . . . . . . . . . . . . . . . . . . . . 117
At inception, this results in the following positive inow of cash:
12 2.50 9.0484 = 0.45163.
At maturity there are three dierent scenarios that may arise:
S
T
< 80: In this case, the puts mature in the money, and the investment is returned
with interest, so that the total cash-ow is:
(90 S
T
) + (80 S
T
) + 10 = 0.
80 S
T
< 90: Here only the 90 strike put ends up in the money, and added to
the return of our loan with interest, delivers the following cash-ow:
(S
T
90) + 10 0.
90 S
T
: Now both puts expire worthless, but we still receive our loan back with
interest, i.e., an inow of 10.
Hence, an examination of all three cases reveals that the cash-ow at maturity is either
zero or positive. Since the cash-ow at inception is also positive, we have created
risk-free prots.
20. The one-year European put option at strike 100 (current stock price = 100), is quoted at
$10. The two-year European put at the same strike is quoted at $4. The term structure
of interest rates is at at 10% (continuously compounded). Is this an arbitrage?
Answer: Not necessarily. As we have noted in the chapter, European put values do
not necessarily increase with maturity. In the language of the option decomposition
undertaken in the next chapter, when maturity increases there are two opposing eects:
the insurance value of a put increases but its time value declines. We will see examples
later in later chapters where this tradeo can lead to lower prices for longer-dated (even
at-the-money) puts.
21. Given the following data, construct an arbitrage strategy: S = 100, K = 95, T = 1/2
year, D = 3 in three months , r = 0.05, and C
E
= 4.
Answer: Clearly C
E
S and C
E
0, so it remains to be checked that C
E
S
PV (K) PV (D). We have
PV (K) = e
(1/2)0.05
95 = 92.654, PV (D) = e
(1/4)?0.05
3 = 2.9627.
This means S PV (K) PV (D) = 4.3833 4, and the no-arbitrage bound is
violated. This tells us the call is undervalued.To take advantage, we buy the call and
sell S PV (K) PV (D). That is:
Sundaram & Das: Derivatives - Problems and Solutions . . . . . . . . . . . . . . . . . . . . . . . . . . . . . . . 118
Buy the call. Cash outow = 4.
Short the stock. Cash inow = 100.
Invest PV(D) for three months. Cash outow = 2.9627.
Invest PV(K) for six months. Cash outow = 92.654.
The initial cash ow from this strategy is 4 +100 2.9627 92.654 = 0.3833. At the
end of three months, we receive $3 from the investment of PV (D), and use this to pay
the dividend due on the shorted stock. Thus, there is no net cash ow at this interim
time point. At the end of two months, there are two possibilities:
S
T
< 95. In this case, we let the call lapse, buy the stock for S
T
and use it to
close out the short position, and receive K = 95 from the investment. Net cash
ow: 95 S
T
> 0.
S
T
95. Now, we exercise the call, buy the stock for 95, use it to cover the short
position and receive K = 95 from the investment. Net cash ow: 0.
With all cash ows being zero or positive, we have identied the desired arbitrage.
22. Given the following data, construct an arbitrage strategy: S = 95, K = 100, T = 1/2
year, D = 3 in three months , r = 0.05, and P
E
= 4.
Answer: We check that P
E
PV (K)+PV (D)S. PV (K) = 97.531 and PV (D) =
2.9627, hence PV (K) + PV (D) S = 97.531 + 2.9627 95 = 5.4937. Note that
the price of the put is less than 5.4937, thereby oering an arbitrage opportunity. We
construct this as follows:
Buy the put since it is undervalued: outow = 4.
Buy the stock, outow = 95.
Borrow PV (K), inow = 97.531.
Borrow PV (D), inow = 2.9627.
Hence, the net cash-ow at inception is 4 95 + 97.531 + 2.9627 = 1.4937.
At the end of three months, collect the dividend and repay the borrowing of 2.9627 with
interest, i.e., repay $3.
At the end of 6 months, if S
T
> 100, sell the stock and repay the borrowing of 95,
resulting in a positive cash ow of S
T
95. If S
T
< 100, then exercise the put, sell the
stock and repay 100. The result here is net zero cash ow.
Since, all cash-ows are zero or positive, we have a risk-free arbitrage strategy.
Sundaram & Das: Derivatives - Problems and Solutions . . . . . . . . . . . . . . . . . . . . . . . . . . . . . . . 119
23. We are given that S = 100, K = 100, T = 1/4, r = 0.06, and C
A
= 1. Is there an
arbitrage opportunity?
Answer: Clearly, C
A
S. The lower bounds C
A
0 and C
A
S K are also
clearly satised. Thus, it remains only to be checked if the third no-arbitrage lower
bound holds. Since D = 0 we must check if C
A
S PV (K). We have PV (K) =
e
(1/4)0.06
100 = 98.511.
Therefore, S PV (K) = 1.4888, and the third no-arbitrage bound is violated. This
means the call is undervalued. The arbitrage bound says it should be worth at least
1.4888, whereas it is trading for only 1. To take advantage of this opportunity, we must
Buy the call.
Short the stock.
Invest PV(K).
This results in an initial cash inow of 1 +100 98.511 = +0.4888. At time T, there
are two possibilities:
S
T
< 100. In this case, we let the call lapse, buy the stock from the market
to cover the short position, and receive K from our investment. Net cash ow:
S
T
+ 100 > 0.
S
T
100. Now, we exercise the call and buy the stock for K, use the stock to
close out the short position, and receive K from the investment. Net cash ow: 0.
Since the strategy has cash inows with no net cash outows, we have derived an
arbitrage opportunity.
24. Given that there are two put options with strikes at 40 and 50, with prices 3 and 14,
respectively, show the arbitrage opportunity if the option maturity is T and interest rates
are r for this maturity.
Answer: The no-arbitrage bound P(K
2
) P(K
1
) < K
2
K
1
is violated here as we
can see that 14 3 > 50 40. The arbitrage is undertaken as follows:
Sell the P(50) option and buy the P(40) one. Inow = 14-3 = 11.
Invest the 11 into the risk-free asset at r.
At maturity, we have three cases:
S
T
> 50: the net cash-ow is 11e
rT
> 0.
40 < S
T
< 50: net cash-ow is 11e
rT
(50 S
T
) > 0.
S
T
< 40: net cash-ow is 11e
rT
(50 S
T
) + (40 S
T
) > 0.
Sundaram & Das: Derivatives - Problems and Solutions . . . . . . . . . . . . . . . . . . . . . . . . . . . . . . . 120
Hence, the cash-ow at all times is positive, resulting in an arbitrage portfolio.
25. Given the price of three calls, construct an arbitrage strategy: C(10) = 13, C(15) = 8,
C(20) = 2.
Answer: Check the buttery spread condition that
0.5 C(10) + 0.5 C(20) > C(15)
We can see this is violated given the existing prices.
The arbitrage strategy is as follows: (a) Buy half a unit of the C(10) option. (b) Sell
the C(15) option. (c) Buy half a unit of the C(20) option.
The cash-ows from this strategy at t = 0 and at maturity are represented in the
following table:
t = 0 cash-ows at t = T
Trades S
T
< 10 10 < S
T
< 15 15 < S
T
< 20 20 < S
T
Buy 1/2 of C(10) -6.5 0
1
2
(S
T
10)
1
2
(S
T
10)
1
2
(S
T
10)
Sell 1 of C(15) +8 0 0 (S
T
15) (S
T
15)
Buy 1/2 of C(20) -1 0 0 0
1
2
(S
T
20)
Sum of cash-ow 0.5 0 > 0 > 0 0
Since the cashows are all zero or positive we have an arbitrage.
26. A call option on a stock is trading for $1.80. The option matures in two months. The
stock is currently trading for $52 and will pay a dividend of $2 in one month. The
risk-free rate of interest (on investments of all maturities) is 12%. Finally, suppose that
the strike price of the option is $50. Examine whether there is an arbitrage opportunity
in this problem. If so, show how it may be exploited to make a risk-less prot.
Answer: Check that C > S PV (K) PV (D). Plugging in the numbers we get
S PV (K) PV (D) = 1.01. this is less than the price of the call, and hence, there
is no arbitrage if the call option is European. But, if the option is American, then there
is an arbitrage because the immediate exercise value is $2 versus a purchase price of
$1.80.
27. ABC stock is currently trading at 100. There are three-month American options on
ABC stock with strike prices 90, 100, and 110. The risk-free interest rate is 12% for
all maturities in continuously compounded terms. Which of the following sets of prices
oers an arbitrage opportunity? How can the opportunity be exploited?
Sundaram & Das: Derivatives - Problems and Solutions . . . . . . . . . . . . . . . . . . . . . . . . . . . . . . . 121
(a) The 90 call is selling for 10 1/4.
(b) The 90 put is at 4, and the 100 put is at 3.
(c) The 100 call is at 12, and the 110 call is at 1.
(d) The 90 call is 13, the 100 call is 8, and the 110 call is 1.
Answer: (a) Check that C(90) = 10.25 > S PV (K) = 100 90e
0.12/4
= 12.66.
We can see that there is a violation. To arbitrage this, we buy C(90), sell the stock,
and invest PV (90) for three months . This gives an initial cash-ow of 2.41.
At maturity if S
T
> 90, then we exercise the call, buy the stock, and receive 90 from
the investment. The net cash-ow is (S
T
90) S
T
+ 90 = 0.
If S
T
< 90, then we buy the stock and receive 90 from investment. The net cash ow
is 90 S
T
> 0.
Hence, we pick up positive cash-ows with no corresponding outow, which is a free
lunch.
(b) This is an obvious arbitrage. We should buy P(100) and sell P(90), resulting in a
net inow of 1. Then at maturity the P(100) option always dominates the P(90) option
resulting in a certain non-negative ow as well.
(c) Here we conduct three dierent tests for presence of arbitrage:
Check that C(100) C(110) 110 100, which it is not, hence there is an
arbitrage. For the arbitrage, buy the 110-strike call, sell the 100-strike call, and
roll over the net inow of $11 in a money-market account. If the 100-strike call is
exercised at any point, exercise the 110-strike call immediately, and close out the
money market acount. The cash ow is
(S
T
100) + (S
T
110) + 11 + interest = 1 + interest
If the 100-strike call is never exercised, exercise the 110-strike call at maturity if it
is in-the-money at that point. Net cash ow:
11 + interest + max(S
T
110, 0).
Check that C(100) > S PV (100) = 2.9554, which it is, again no arbitrage.
Check that C(110) > S PV (110) = 6.7490 which it is, again no arbitrage.
(d) This case results in a violation of the buttery spread condition in that
1
2
C(90) +
1
2
C(110) > C(100)
is violated. To arbitrage this, buy half each of the C(90) and C(110) options, and sell
one unit of the C(100) option. This results in a net inow of $1 at inception. From the
condition above we know that at maturity, cash-ow will be non-negative, and hence
this is an arbitrage.
Sundaram & Das: Derivatives - Problems and Solutions . . . . . . . . . . . . . . . . . . . . . . . . . . . . . . . 122
Chapter 10. Early-Exercise & Put-Call Parity
Note: All interest rates, unless stated otherwise, are assumed to be continuously compounded.
1. A stock is trading at $100. The interest rate for one year is 5% continuously com-
pounded. If a European call option on this stock at a strike of $99 is priced at $8.50,
break down the call option value into
(a) Instrinsic value.
(b) Time value.
(c) Insurance value.
Answer: The intrinsic value of the call is 1 = 100 99.
The pure time value is the value of the strike minus the present value of the strike, that
is the saving from not having to pay for the stock but retain the right to it. This is
equal to
K PV (K) = 99 99e
rT
= 99(1 e
0.05
) = 4.8283.
Finally, the insurance value of the option is the remaining value, i.e.,
8.50 1 4.8283 = 2.6717.
In other text books, it is common to describe as an options time value the sum of
what we have described here as its time value and its insurance value. In this example,
that would be $7.50.
2. In the question above, what is the insurance value of the corresponding put option?
Answer: First, we need to nd the price of the put option. This is obtained from put-call
parity.
P = C S +PV (K) = 8.50 100 + 99e
0.05
= 2.6717.
Note that this is nothing but the insurance value of the call!
The intrinsic value of the put is $-1.
The pure time value of the put is
PV (K) K = $ 4.8283.
Hence, the insurance value of the put is
P Intrinsic value Pure time value = 2.6717 (1) (4, 8283) = 8.50
which is nothing but the value of the call option!
Sundaram & Das: Derivatives - Problems and Solutions . . . . . . . . . . . . . . . . . . . . . . . . . . . . . . . 123
3. Explain why a European call on a stock that pays no dividends is never exercised early.
What would you do instead to eliminate the call option position?
Answer: It only pays to exercise a call option if the current exercise value exceeds the
remaining time value plus the insurance value of the option. We may derive this based
on the value decomposition of the call, which is as follows:
C = (S K) + [K PV (K)] +IV (C)
which presents the call value as the sum of instrinsic value, time value and insurance
value.
Exercise occurs if the call value C is less than or equal to its current exercise value. This
implies that
(S K) C
which in turn implies that
[K PV (K)] +IV (C) 0
But since neither of the time value nor the insurance value is negative, the condition
above is never satised, and hence the call should never be exercised, as it is always
worth more than the exercise value.
To monetize the option position, the optimal thing to do is to sell the call itself, not
exercise it.
4. Stock ABC pays no dividends. The current price of an American call on the stock at
a strike of 41 is $4. The current stock price is $40. Compute the time value of the
European put option if it is trading at a price of $3.
Answer: Since there are no dividends, the price C
A
of the American call is equal to that
of its European counterpart C
E
. Now, the value of the European put is the sum of its
intrinsic, time, and insurance values. The intrinsic value is K S = 41 40 = +1. As
noted in the text, the insurance value of a European put option is just the value of the
corresponding call, so the put has an insurance value of $4. Since the value of the put
is given to be 3, we have
3 = 1 + Time Value + 4,
so the time value of the put is 2. Note that the time value of puts in general is
negative.
Sundaram & Das: Derivatives - Problems and Solutions . . . . . . . . . . . . . . . . . . . . . . . . . . . . . . . 124
5. Stock ABC is trading at a price of $50. At a strike price of 55, there is a traded six-
month American put. There are no dividends on the stock, and maturity of the option
is a half year. If the half-year rate of interest is 5%, what must the minimum insurance
value of the put be for the put not to be exercised?
Answer: If the put is exercised now, the take is $5. For the put to be unattractive for
exercise, it must trade at a value greater than 5, which means that the sum of its time
value and insurance value must be greater than zero. This iimplies that
PV (K) K +IV (P) > 0
or
IV (P) > K PV (K) = 55 55e
0.05(0.5)
= 1.3580.
Hence, the minimum insurance value that makes the holder retain the option is roughly
$1.36.
6. Stock XYZ is trading at a price of $105. An American-style call option on Stock XYZ
with maturity one year and strike 100 is traded in the market. The term structure of
interest rates is at at 1% and there is a dividend payment in six months of $8. What
is the maximum insurance value for the call at which it still makes sense to exercise it?
Answer: If we exercise the call now we collect $5.
The call is better o exercised when it is worth less than its exercise value, i.e., its sum
of time and insurance value should be non-positive, i.e.,
K PV (K) PV (D) +IV (C) 0
100 100e
0.01
8e
0.01(0.5)
+IV (C) 0
Re-arranging, this implies that
IV (C) 2.1366
to make the call worth immediately exercising. Thus, the maximum insurance value of
the call is roughly $2.14.
7. Assume that the true formula for pricing options is unknown, e.g., Black-Scholes is not
applicable. Hence, you are asked to use the following approximation for the insurance
value of a put option:
IV (P) = exp
_
S
K

2
T
_
Sundaram & Das: Derivatives - Problems and Solutions . . . . . . . . . . . . . . . . . . . . . . . . . . . . . . . 125
where S is the current price of the stock, K is the strike price, is the volatility of the
stock return, and T is option maturity.
You are given that S = 100, K = 105, and the interest rate r = 1%. Option maturity
is T = 1 year, and there are no dividends.
What is the maximum volatility for which early exercise of the option is induced?
Answer: For the put to be exercised, the sum of the puts time value and insurance
value needs to be non-positive, i.e.,
105e
0.01
105 + exp
_
100
105

2
1
_
0.
Re-arranging this expression we have that

_
_
[ln(105 105e
0.01
)]
105
100
_
= 0.21444.
Hence, the maximum volatility is 21.44%. Above this it pays to retain the option rather
than exercise it.
8. If a stock does not pay dividends, what is the relationship between call prices and interest
rates for early exercise of a put to occur?
Answer: A put option is worth exercising if the sum of its time and insurance values is
non-positive. This is:
PV (K) K +IV (P) 0.
Since the insurance value of a put is also the price of the corresponding call (assuming
the options are European), we may write this as
Ke
rT
K +C 0.
If we re-arrange this expression, we have
r >
1
T
ln
_
1
C
K
_
which gives the formula linking r on the LHS with C on the RHS of the equation above.
We can see that if C increases, so must r.
The inequality above shows that as the call price increases, i.e., the insurance value of
the put rises, the interest rate r needs to be increasingly higher to make early exercise
of the put optimal.
Sundaram & Das: Derivatives - Problems and Solutions . . . . . . . . . . . . . . . . . . . . . . . . . . . . . . . 126
9. You are given the following data about options: S = 60, K = 60, r = 2%, T = 0.5,
D = 0 (dividends). If the American call is trading at a price of $5, what is the minimum
price of the American put?
Answer: With European options we may use the put-call parity relationship to determine
the price of the European put. Put-call parity is as follows:
C
E
P
E
= S PV (K)
where the E subscript denotes European, and we will use the A subscript to denote
American. The equation above implies that
P
E
= C
E
S +PV (K)
= C
A
S +PV (K) because there are no dividends, C
A
= C
E
= 5 60 + 60e
0.02(0.5)
= 4.4030.
Since the American put is worth at least as much, we have that
P
A
4.4030.
10. In the preceding question, rene the upper bound on the European put if there is a
dividend to be received after three months of an amount of $2. Assume that the term
structure is at and the American call with dividends is worth $6.
Answer: First, we compute the present value of the dividends, i.e.,
PV (D) = 2e
0.02(0.25)
= 1.99.
Then
P
E
= C
E
S +PV (K) +PV (D)
C
A
S +PV (K) +PV (D)
= 6 60 + 60e
0.02(0.5)
+ 1.99
= 7.3930.
Hence, the European put will be worth at most $7.3930.
Sundaram & Das: Derivatives - Problems and Solutions . . . . . . . . . . . . . . . . . . . . . . . . . . . . . . . 127
11. Company WHY pays no dividends. Its stock price is $30. The three-month European
call at strike 29 is trading at $3. The three-month interest rate is 1%. What is the price
of the European put?
Answer: Here we directly apply put-call parity:
P
E
= C
E
S +Ke
rT
= 3 30 + 29e
0.01(0.25)
= 1.9276.
12. Stock ABC is trading at $43 and pays no dividends. If the six-month 50-strike call and
put are equal in price, what is the six-month risk-free interest rate?
Answer: From put call parity, if C = P, then it is also the case that S PV (K) = 0.
Hence
S Ke
rT
= 43 50e
0.5r
= 0
Solving we get
r = 2 ln(43/50) = 0.30165,
or 30.17%.
13. Stock XYZ is currently priced at $50. It pays no dividends. The one-year maturity
60-strike European call and put are trading at $10 and $12, respectively. What is the
one-year forward price on the stock?
Answer: Once again, we use put-call parity to determine the rate of interest.
C P = S Ke
rT
and entering the values in the formula gives us:
10 12 = 50 60e
r
which is solved to give
r = ln(52/60).
Sundaram & Das: Derivatives - Problems and Solutions . . . . . . . . . . . . . . . . . . . . . . . . . . . . . . . 128
The forward price is given by
F = Se
rT
= 50e
r1
= 50e
ln(52/60)
= 50e
ln(60/52)
= 50 60/52
= 57.69
14. You observe the following European option prices in the OTC market on stock QWY,
which does not pay dividends:
T (years) Call Puts Strike
0.5 7 5 100
1.0 19 12 100
However, the rm you work for does not subscribe to price quote services for the equity
and interest rate markets. All you know is that the term structure of interest rates is
at.
You receive a call from a client wishing to buy a forward on the stock QWY for two
years. What price should you quote for this contract?
Answer: We can set up the following two equations using the put-call parity condition:
C P = S PV (K):
7 5 = S 100e
0.5r
19 12 = S 100e
1.0r
We may solve these two equations for the stock price and interest rate. The solutions
are
S = 96.72, r = 0.1085.
Now we can easily price the forward using the cost-of-carry model, i.e.,
F = Se
rT
= 96.72e
0.1085(2)
= 120.17.
This is an example of how we can extract information about the equity and bond markets
from the option markets.
Sundaram & Das: Derivatives - Problems and Solutions . . . . . . . . . . . . . . . . . . . . . . . . . . . . . . . 129
15. You are working on an option trading desk in charge of arbitrage trading. The following
data is presented to you on screen, and you immediately see an arbitrage. What is it,
and how much money can you make risk-free?
S = 60
K = 60
r = 3%
T = 0.25
D = 0
C
E
= 5
P
E
= 3
Answer: We check put-call parity for the arbitrage opportunity.
Check that C P = S PV (K).
C P = 5 3 = 2.
S Ke
rT
= 60 60e
0.03(0.25)
= 60 59.552 = 0.448.
This means the call is overpriced relative to the put. We set up the following portfolio:
Sell the call.
Buy the put.
Buy the stock at 60.
Borrow 59.552 at 3%.
At inception, the net cash-ow is
5 3 60 + 59.552 = 1.5520.
At maturity, we have two scenarios:
(a) If S
T
< K: In this case the put is in the money and the call is out of the money.
We receive the following cahow (listed for each of the 4 portfolio positions:
0 + (60 S
T
) +S
T
60 = 0.
Note that the borrowing is repaid with interest, i.e., a net payment of 60.
(b) If S
T
> K: Here the cash ows at maturity will be:
(S
T
60) + 0 +S
T
60 = 0.
So, in both cases, the net cash-ow at maturity is zero. Since we are able to pocket the
positive cash-ow at inception, we have created arbitrage prots.
Sundaram & Das: Derivatives - Problems and Solutions . . . . . . . . . . . . . . . . . . . . . . . . . . . . . . . 130
16. Stock DEF is trading at $100 and is expected to pay a dividend of $3 in three months.
The European call at strike 95 with half-year maturity is priced at $7. If the at term
structure of interest rates is 5% nd a lower bound on the price of the American put
option.
Answer: From put call parity we have that
P
E
= C
E
S +PV (K) +PV (D)
= 7 100 + 95e
0.05(0.5)
+ 3e
0.05(0.25)
= 2.6172.
Hence, a lower bound on the American put is the value of the European put, i.e., roughly
$2.62.
17. Stock CBA is trading at price $50 and is not expected to pay any dividends. The
following puts are traded at maturity in three months :
P(K = 50) = 5
P(K = 60) = 13
The three-month interest rate is 2%. What is the price of a (50,60) bullish call spread?
Answer: A bullish call spread is the purchase of a call at strike 50 and sale of a call at
strike 60. Using put-call parity, we can price each of these in turn:
The price of the 50-strike call is:
C(50) = P(50) +S PV (K) = 5 + 50 50e
(0.02)(0.25)
= 5.2494.
The price of the 60-strike call is:
C(60) = P(60) +S PV (K) = 13 + 50 60e
0.02(0.25)
= 3.2993.
So the cost of the call spread is
C(50) C(60) = 5.2494 3.2993 = 1.9501
Sundaram & Das: Derivatives - Problems and Solutions . . . . . . . . . . . . . . . . . . . . . . . . . . . . . . . 131
18. Stock KLM trades at $100 and pays no dividends. The one-year straddle struck at $102
is trading at a price of $10. The one-year interest rate is 2%. Find the price of the
one-year European call and put.
Answer: Starting with put-call parity we can tease out the value of the call and put
separately as follows:
C P = S PV (K)
C = S PV (K) +P
C +P = S PV (K) + 2P, the LHS is a straddle
S PV (K) + 2P = 10, Solve this for the put
Hence solving we have:
P =
1
2
[10 100 + 102e
0.02(1)
] = 4.99.
And the price of the call is easy to obtain:
C = Straddle P = 10 4.99 = 5.01.
We may also solve this problem using a second approach, as follows. We set up a system
of 2 simultaneous equations:
C +P = 10
C P = S PV (K) = 100 102e
0.02(1)
= 0.02.
Solving these two equations gives the solutions for C and P.
19. An investor buys a call on ABC stock with a strike price of K and writes a put with
the same strike price and maturity. Assuming the options are European and that there
are no dividends expected during the life of the underlying, how much should such a
portfolio cost?
Answer: From put-call parity, C(K, T) P(K, T) = S PV (K). The portfolio is
equal in value to the stock price minus the present value of the strike.
20. Use put-call parity to show that the cost of a buttery spread created using European
puts is identical to the cost of a buttery spread using European calls.
Answer: Assume three strikes K
1
, K
2
, K
3
set equal distance apart from each other.
Note that K
1
+ K
3
= 2 K
2
, so PV (K
1
) + PV (K
3
) = 2 PV (K
2
). Using this and
put-call parity, we have
Sundaram & Das: Derivatives - Problems and Solutions . . . . . . . . . . . . . . . . . . . . . . . . . . . . . . . 132
C(K
1
) 2C(K
2
) +C(K
3
) = [S PV (K
1
) +P(K
1
)]
2[S PV (K
2
) +P(K
2
)]
+[S PV (K
3
) +P(K
3
)]
= P(K
1
) 2P(K
2
) +P(K
3
)
21. A stock is trading at S = 50. There are one-month European calls and puts on the
stock with a strike of 50. The call is trading at a price of C
E
= 3. Assume that the
one-month rate of interest (annualized) is 2% and that no dividends are expected on
the stock over the next month.
(a) What should be the arbitrage-free price of the put?
(b) Suppose the put is trading at a price of P
E
= 2.70. Are there any arbitrage
opportunities?
Answer: We check put-call parity as follows:
P
E
= C
E
S +PV (K) = 3 50 + 50e
0.02/12
= 2.917
This is greater than 2.70, the quoted price of the put, hence we should buy the put as
it is cheap. Buy it for 2.70. Then sell the call, buy the stock, and borrow the present
value of the strike. These latter three trades bring in a net cash-ow of 2.917. Hence,
at inception, we get a net positive cash-ow of 0.217. At maturity the entire set of
trades results in a net zero cash-ow by the usual put-call parity arguments; we omit
the details. Thus, we have a risk-less prot.
22. A stock is trading at S = 60. There are one-month American calls and puts on the
stock with a strike of 60. The call costs 2.50 while the put costs 1.90. No dividends
are expected on the stock during the options lives. If the one-month rate of interest
(annualized) is 3%, show that there is an arbitrage opportunity available and explain
how to take advantage of it.
Answer: Noting that when there are no dividends, the American call and European call
are equal in price, we get that C
A
= C
E
. We check put-call parity as follows:
P
A
P
E
= C
A
S +PV (K) = 2.50 60 + 60e
0.03/12
= 2.35.
This is greater than 1.90, the quoted price of the American put, hence we should buy
the put as it is cheap. Buy it for 1.90. Then sell the American call, buy the stock, and
Sundaram & Das: Derivatives - Problems and Solutions . . . . . . . . . . . . . . . . . . . . . . . . . . . . . . . 133
borrow the present value of the strike. These latter three trades bring in a net cash-ow
of 2.35. Hence, at inception, we get a net positive cash-ow of 0.45. Since we hold the
put, we can always hold it to maturity. There are two possibilities concerning the call:
(a) The call is exercised early. (Note that it is not optimal to do so, but we cannot
insist on optimal behavior from the buyer of the call.) In this case, deliver the
stock, receive K = 60 and invest this to maturity. At maturity, repay K = 60
on the borrowing, and exercise the put if this is optimal. The net cash ow at
maturity is at least the value of the interest earned on the strike priced invested,
so is positive.
(b) The call is not exercised early. In this case, by the usual put-call parity arguments
(we omit the details and leave them as an exercise), there is no net cash ow at
maturity.
Thus, in all cases, there is an initial cash inow and no further outows, which means
we have created arbitrage prots.
Sundaram & Das: Derivatives - Problems and Solutions . . . . . . . . . . . . . . . . . . . . . . . . . . . . . . . 134
Chapter 11. Option Pricing: An Introduction
1. Explain intuitively why the delta of a call will lie between zero and unity. When will it
be close to zero?When will it be close to unity?
Answer: See Section 11.8 of the book.
2. Give an example of a derivative where the delta may be either positive or negative for
dierent ranges of the stock price. (Use your imagination here.)
Answer: Delta measures the sensitivity of the price of the derivative to changes in the
price of the underlying. Intuitively, if the payo from a derivative increases as the price
of the underlying increases, the delta will be positive; while f it decreases as the price
of the underlying increases, the delta will be negative. So for a derivative to have zones
of positive and negative deltas, we want there to be ranges of the stock price where
the derivatives payo increases with the stock price S
T
at maturity and ranges where
it decreases as S
T
increases.
A simple example of such a derivative is one whose whose payo is identical to that of
a buttery spread with strikes K
1
, K
2
, K
3
. The payo from this derivative is zero for
S
T
K
1
or S
T
K
3
; increases between K
1
and K
2
; and decreases between K
2
and
K
3
.
A second example is a range option. A range option is a derivative security that pays a
xed amount if the stock price at maturity is in a specied range and nothing otherwise.
Take a range option that pays $1 if at maturity the stock is in the interval [K
1
, K
2
] and
nothing otherwise. For stock prices well below K
1
, an increase in the stock price improves
the prospects of a positive payo from the option, so the option value increases and
delta is positive; while for stock prices well above K
2
, a decrease in the price increases
the prospect of a positive payo, so the option value increases and delta is negative.
3. A stock is currently trading at 80. You hold a portfolio consisting of the following:
(a) Long 100 units of stock.
(b) Short 100 calls, each with a strike of 90.
(c) Long 100 puts, each with a strike of 70.
Suppose the delta of the 90-strike call is 0.45 while the delta of the 70-strike put is
0.60. What is the aggregate delta of your portfolio?
Answer: We consider each component of the portfolio individually, and then add them
up.
(a) You are long 100 units of the stock.
Sundaram & Das: Derivatives - Problems and Solutions . . . . . . . . . . . . . . . . . . . . . . . . . . . . . . . 135
(b) Each 90strike call in the portfolio has a delta of 0.45, so is like being long 0.45
units of the stock. Thus, 100 units of the 90-strike call are like 45 units of the
stock. Since you are short the calls, it is as if you are short 45 units of the stock.
(c) Each 70-strike put has a delta of 0.60, so each put is like a short position in
0.60 units of the stock. Since you are long 100 puts, it is as if you are short 60
units of the stock.
In aggregate, the portfolio is as if you have +1004560 = 5 units of the stock, i.e.,
you are short ve units of the stock. An increase of $1 in the stock price will decrease
the value of your portfolio by around $5.
4. (Dicult) Compare the replication of an option in a binomial model versus replication
in a trinomial model by answering the following questions:
(a) How many securities do we need to carry out replication in each model?
(b) Is the risk-neutral probability dened in each model unique?
Answer: In a binomial model, there are two next-period prices that emanate from
each node. With two securities (usually, the underlying stock and cash), we can span
these states and replicate the option by solving two simultaneous equations, as we did
in Section 11.3. The risk-neutral probability is unique in this situation, as we showed in
Section 11.6.
In a trinomial model, there will be three branches emanating from each node correspond-
ing to the three possible next-period prices. So spanning the prices and replicating
the option means solving a three equation system, meaning we need three unknowns.
So three securities (for example, the stock, cash, and a dierent option) are required
for replication. The risk-neutral probabilities will not be unique in such a model. See
Appendix 11C.
5. In a binomial framework, if the risk-neutral probability on the up branch is given as
p = 0.8956, the risk-free rate per period is 2%, and the down move is the reciprocal of
the up move, then, given a current stock price of $100, what are the two prices a period
from now?
Answer: Given that d = 1/u, the risk-neutral probability is given by
p =
R d
u d
=
R 1/u
u 1/u
.
or in numbers:
0.8956 =
1.02 1/u
u 1/u
.
Sundaram & Das: Derivatives - Problems and Solutions . . . . . . . . . . . . . . . . . . . . . . . . . . . . . . . 136
Solving, we get
u = 1.0252, d = 1/u = 0.9754
Therefore, the two prices after one period will be
uS = 102.52, dS = 97.54
6. In the question above, suppose we have a one-period call option with a strike price of
$100, what is the delta of the call? If the up-shift parameter u is increased to 1.5, then
what is the delta of the call? Is it higher or lower? Why?
Answer: The call delta is given by
C
u
C
d
uS dS
where C
u
and C
d
are the values of the call at the notes uS and dS, respectively. So the
delta is
=
30 0
130 76.92
= 0.5652.
If u = 1.5, then (assuming that d = 1/u still holds), the stock prices after one period
are uS = 150 and dS = 66.67, leading to option payos of C
u
= 50 and C
d
= 0. So
the new delta is
=
50 0
150 66.67
= 0.60.
7. A stock is currenly trading at 80. There are one-month calls and puts on the stock with
strike prices of 70, 75, 80, 85, and 90. The price and delta of each of these options is
given below:
Strike 70 75 80 85 90
Call Price 10.60 6.47 3.39 1.50 0.56
Put Price 0.30 1.15 3.05 6.14 10.18
Call 0.92 0.77 0.54 0.31 0.14
Put 0.08 0.23 0.46 0.69 0.86
Sundaram & Das: Derivatives - Problems and Solutions . . . . . . . . . . . . . . . . . . . . . . . . . . . . . . . 137
For each of the following portfolios, identify (i) the current value of the portfolio, and
(ii) the approximate value of the portfolio following a $1 decrease in the stock price.
(a) Long 100 units of stock, short 100 units of the 80-strike call.
(b) Long 1000 units of the 80-strike call and 1174 units of the 80-strike put.
(c) Long 100 units of stock, long 100 units of the 75-strike put, and short 100 units
of the 85-strike call.
(d) Long 100 units of the 70-strike call, long 100 units of the 90-strike call, and short
200 units of the 80-strike call.
(e) Long 100 units of the 85-strike put and short 100 units of the 75-strike put.
Answer: The answers to each part are give below:
(a) The value of the portfolio is:
(100 80) (100 3.39) = 7, 661
The delta of the portfolio is
(100 1) (100 0.54) = 46.
From the delta, the portfolio gains about $46 for every $1 increase in the stock
price. Thus, following a $1 decrease in the stock price, the portfolio will lose $46,
so its value will fall to about $7615.
(b) The value of the portfolio is:
(1000 3.39) + (1174 3.05) = 6, 970
The delta of the portfolio is
(1000 0.54) + (1174 0.46) = 0.04
Note that this portfolio is (almost) delta-neutral, i.e., it has a delta near zero and
hardly responds to changes in the stock price. A $1 decrease in the stock price will
increase the portfolio value by just 0.04.
(c) This portfolio is a collar (see Chapter 8). Its current value is:
(100 80) + (100 1.15) (100 1.50) = 7, 965
The delta of the portfolio is
(100 1) + (100 0.23) (100 0.31) = +46
Thus, the portfolio will lose $46 of value if the stock price decreases by $1, so the
new value of the portfolio is 7,919.
Sundaram & Das: Derivatives - Problems and Solutions . . . . . . . . . . . . . . . . . . . . . . . . . . . . . . . 138
(d) This portfolio is a buttery spread (see Chapter 8). The current value of the
portfolio is:
(100 10.60) + (100 0.56) (200 3.39) = 438
The delta of the portfolio is
(100 0.92) + (100 0.14) (200 0.54) = 2
Thus, the portfolio will increase by $2 if the stock price decreases by $1, so the
new portfolio value will be 440.
(e) This is a bear spread using puts (see Chapter 8). Its current value is
(100 6.14) (100 1.15) = 499
The delta of the portfolio is
(100 0.69) (100 0.23) = 46
Thus, the portfolio will gain by $46 if the stock price falls by $1, so the new
portfolio value will be $545.
8. ABC stock is currently trading at 100. In the next period, the price will either go up by
10% or down by 10%. The risk-free rate of interest over the period is 5%.
(a) Construct a replicating portfolio to value a call option written today with a strike
price of 100. What is the hedge ratio?
(b) Calculate the risk-neutral probabilities in the model. Value the same call option
using the risk-neutral probabilities. Check that you get the same answer as in part
(a).
(c) Using the risk-neutral probabilities, nd the value of a put option written today,
lasting one period, and with an exercise price of 100.
(d) Verify that the same price for the put results from put-call parity.
Answer: We are given the following information: S = 100, u = 1.10, d = 0.90, and
r = 1.05.
(a) Since K = 100, the payos from the call at maturity are C
u
= max{uSK, 0} =
10 and C
d
= max{dS K, 0} = 0.
Let the replicating portfolio consist of units of the stock and borrowing/investment
of $B. The initial cost of the portfolio is
S +B = 100 +B.
Sundaram & Das: Derivatives - Problems and Solutions . . . . . . . . . . . . . . . . . . . . . . . . . . . . . . . 139
If the portfolio is to be replicate the call, we must have
(uS) +rB = C
u
(dS) +rB = C
d
Substituting for these values, we obtain
110 + 1.05 B = 10
90 + 1.05 B = 0
This implies = 1/2 and B = 42.86, i.e., the replicating portfolio involves a
long position in a half unit of the stock and a borrowing of 42.86. Thus, the initial
cost of the replicating portfolio is
(1/2) 100 42.86 = 7.14.
Since the portfolio replicates the call, the call must also have an initial cost of 7.14.
(b) The risk-neutral probability p in this model is obtained by solving
p(1.10) + (1 p)(0.90) = 1.05,
so p = 3/4. For this value of p, the expected value of the call payos after one
period are
3
4
(10) +
1
4
(0) = 7.50.
Discounting back by 1.05, we obtain
7.50
1.05
= 7.14,
which is the same answer as obtained earlier.
(c) The put pays P
u
= 0 if the stock price moves up, and P
d
= 10 if the stock price
moves down. Using p, the value of the put is, therefore,
1
r
(p P
u
+ (1 p) P
d
) =
1
1.05
(2.50) = 2.38.
(d) Putcall parity states that C + PV (K) = P + S. Applied to the numbers here,
this means
7.14 +
1
1.05
100 = P + 100,
or P = 7.14 + 95.24 100 = 2.38.
Sundaram & Das: Derivatives - Problems and Solutions . . . . . . . . . . . . . . . . . . . . . . . . . . . . . . . 140
9. ABC stock is currently at 100. In the next period, the price will either increase by 10%
or decrease by 10%. The risk-free rate of return per period is 2%. Consider a call option
on ABC stock with strike K = 100.
(a) Set up a replicating portfolio to value the call.
(b) Suppose the call is trading for $7. Explain how you would exploit the resulting
arbitrage opportunity.
Answer: We are given the following information: S = 100, u = 1.10, d = 0.90,
r = 1.02, and K = 100. Note that the possible call payos after one period are
C
u
= 10 and C
d
= 0.
(a) Let the replicating portfolio consist of units of the stock and $B of investment.
The initial cost of the portfolio is
S +B = 100 +B.
Since the portfolio must replicate the option payos after one period, we must pick
and B to satisfy
uS +rB = C
u
dS +rB = C
d
Solving these equations, we get = 1/2 and B = 44.12. Under these values,
the initial cost of the portfolio is
100 +B = 50 44.12 = 5.88.
Since the portfolio replicates the option, the initial cost C of the option must be
5.88.
(b) If the option is selling for 7.00, it is overpriced relative to the replicating portfolio.
To exploit the arbitrage, sell the option and buy the replicating portfolio, i.e., sell
the call, buy 1/2 unit of the stock, and borrow 44.12 for one period. This leads to
an initial cash inow of
7
1
2
(100) + 44.12 = 1.12.
At the end of the period, there are two possibilities: the stock price either moves
up to uS = 110, or down to dS = 90. If the stock price moves up to 110, the
holder of the stock will exercise the call, leading to a cash outow of 10. [When
the holder of the call exercises, the writer is giving up a stock worth 110 for 100.]
Moreover, a cash outow of 45 is due on the borrowed amount. However, a cash
Sundaram & Das: Derivatives - Problems and Solutions . . . . . . . . . . . . . . . . . . . . . . . . . . . . . . . 141
inow of
1
2
(110) = 55 is obtained by selling the 1/2 unit of the long stock in the
portfolio. Thus, the net cash inow is
55 45 10 = 0.
If the stock price after one period is 90, the call will lapse unexpired. There is a
cash outow of 45 associated with the borrowing, and a cash inow of
1
2
(90) = 45
from selling the 1/2 unit of the long stock in the portfolio. Thus, the net cash
inow is again
45 45 = 0.
Since the portfolio involves an initial cash inow and no further cash outows, it
is an arbitrage.
10. ABC stock is currently at 100. In the next period, the price will either increase by 5%
or decrease by 5%. The risk-free rate of return per period is 3%. Consider a put option
on ABC stock with strike K = 100.
(a) Set up a replicating portfolio to value the put.
(b) Suppose the put is trading for $2. Explain how you would exploit the resulting
arbitrage opportunity.
Answer: Since this is similar to a previous question, the details are omitted. The portfolio
that replicates the put involves a short position in 0.50 units of the stock and lending
of $50.97. Since the stock is trading at 100, the initial price of the put is 50.97
(1/2)(100) = 0.97.
If the put price is $2, the put is overvalued. To take advantage of the arbitrage opening,
you should sell the put and buy the replicating portfolio (i.e., sell the put, short 1/2 unit
of the stock, and invest $50.97. This will result in an immediate cash inow of $1.03,
with no further cash outows.
11. Consider a one-period binomial model with the parameters u = 1.05, d = 0.95, and
r = 1.01. Let the initial stock price be S = 100.
(a) Identify the price and delta of a call with strike K = 100.
(b) Repeat this exercise for K = 96, K = 98, K = 102, and K = 104.
(c) Use put-call parity to identify the value of the corresponding put options and the
put deltas.
Sundaram & Das: Derivatives - Problems and Solutions . . . . . . . . . . . . . . . . . . . . . . . . . . . . . . . 142
Answer: We solve for the option prices and deltas by using replicating portfolios.
(a) K = 100:
Set up a replicating portfolio with
100
units of stock and B
100
units invested/borrowed
at the risk-free rate. Then,
100
and B
100
must satisfy:

100
(105) +B
100
(1.01) = 5.

100
(95) +B
100
(1.01) = 0.
Solving the equations, we obtain:
100
= 1/2 and B
100
= 47.03. Thus, the
replicating portfolio involves a long position in a half-unit of the stock and borrowing
of 47.03. The initial cost of this replicating portfolio is
1
2
(100) 47.03 = 2.97.
Since the option must cost the same as the replicating portfolio, we have the nal
answers C
100
= 2.97 and
100
= 0.50.
(b) For the remaining strike prices, repeating these arguments yields:
K = 96: C
96
= 5.347 and
96
= 0.90.
K = 98: C
98
= 4.158 and
98
= 0.70.
K = 102: C
102
= 1.782 and
102
= 0.30.
K = 104: C
104
= 0.594 and
104
= 0.10.
(c) For the put options, we have the following results:
K = 96: P
96
= 0.40 and
p
96
= 0.10.
K = 98: P
98
= 1.19 and
p
98
= 0.30.
K = 100: P
100
= 1.98 and
p
100
= 0.50.
K = 102: P
102
= 2.77 and
p
102
= 0.70.
K = 104: P
104
= 3.56 and
p
104
= 0.90.
12. There are two stocks, A and B, both trading at price $20. Consider a one-period binomial
model in which stock As price can go to either of {35, 5}. Stock Bs price can take one
of the following values after one period: {36, 18}. An investment in $1.00 of bonds at
the start of the period delivers a risk-free value at the end of the period of $1.10.
(a) Using replication, nd the prices of call options on both stocks A and B if the calls
have a strike of $20.
(b) Which call is worth more, that on stock A or on stock B? Why?
Sundaram & Das: Derivatives - Problems and Solutions . . . . . . . . . . . . . . . . . . . . . . . . . . . . . . . 143
Answer: First we price the call on stock A. To replicate the call, we invest in s units of
the stock and b units of the bond. To get the correct call replicating values of s and b,
we set up the following system of simultaneous equations:
35s + 1.1b = 15
5s + 1.1b = 0
Solving, we have s = 0.5 and b = 2.2727. Hence the call price at inception is the
value of this replicating portfolio at time 0, i.e.
C
A
= 20s + 1b = 20(0.5) 2.2727 = 7.7273.
Next, we replicate the call option C
B
. To get the correct call replicating values of s and
b, we set up the following system of simultaneous equations:
36s + 1.1b = 16
18s + 1.1b = 0
Solving, we have s = 0.8889 and b = 14.545. Hence the call price at inception is the
value of this replicating portfolio at time 0, i.e.
C
B
= 20s + 1b = 20(0.8889) 15.545 = 3.2325.
The call option on stock A is worth more! Even though it looked like the payos to B
dominated those of A (which is unclear since there were no probabilities given for the
respective up and down moves), the call on A is worth more than twice that on B.
The reason for this is that the volatility of stock A is higher than that of B, and this
is the determinant of option value. Looked at in another way, we can see that holding
the stock A as a hedge (if we were the option writer) would mean a greater loss if the
option ended up out of the money, as the hedge would suer a greater loss than with
stock B. Thus, option A must cost more.
13. In a one-period setting, suppose there are three states of the world at the end of the
period. Suppose there are three securities, stocks A and C, and a risk-free bond B. The
initial prices of securities A, B, and C are, respectively, 20, 1, and 10. The prices after
one period are as given in the table below:
Security State 1 State 2 State 3
A 50 20 5
B 1.10 1.10 1.10
C 20 30 2
Sundaram & Das: Derivatives - Problems and Solutions . . . . . . . . . . . . . . . . . . . . . . . . . . . . . . . 144
(a) Using replication, nd the price of a call option on stock A at a strike price of
K = 15.
(b) Using replication, nd the price of a call option on stock B at a strike price of
K = 15.
(c) What are state prices? Compute these for the three states in the model. (State
prices are dened in Appendix 11B.)
(d) Show how you would price the two call options above using state prices.
Answer: Solving these questions requires us to rst determine the risk-neutral probabil-
ities of the three states, since these are needed to price options. If we set the bond B
as the numeraire asset, then the prices of all three securities (after normalization) must
be martingales. Dene the risk-neutral probabilities as
Q = {q
1
, q
2
, q
3
}.
For security A, the martingale condition is (current price must equal the expected future
normalized prices):
20 = (50/1.10)q
1
+ (20/1.10)q
2
+ (5/1.10)q
3
For security B, this is
1 = (1.10/1.10)q
1
+ (1.10/1.10)q
2
+ (1.10/1.10)q
3
which is trivially valid. For security C, this is
10 = (20/1.10)q
1
+ (30/1.10)q
2
+ (2/1.10)q
3
Solving these equations, we obtain
Q = [0.3444, 0.1000, 0.5556]
Now that we have these risk-neutral probabilities, we can proceed to pricing options.
(a) Call at strike 15 on A: We take the payos in each of the three states, weight
them by the risk-neutral probabilities to get the expected value of the call and then
discount it to today as follows:
C
A
= [max(0, 50 15)q
1
+ max(0, 20 15)q
2
+ max(0, 5 15)q
3
]/1.10 = 11.414.
(b) Likewise the call on stock C is:
C
C
= [max(0, 20 15)q
1
+ max(0, 30 15)q
2
+ max(0, 2 15)q
3
]/1.10 = 2.9293.
Sundaram & Das: Derivatives - Problems and Solutions . . . . . . . . . . . . . . . . . . . . . . . . . . . . . . . 145
(c) State Prices: State prices are the time-0 values of special securities that pay o a
$1 in one state and nothing otherwise. It is easy to see that state prices are just
the discounted value of risk-neutral state probabilities (see Appendix 11B). Hence,
in our problem, the three state prices ( = [
1
,
2
,
3
]) are given by
= Q/1.10 = [0.3131, 0.0909, 0.5050]
(d) To price the call on A using state prices, we carry out the following calculation,
where we multiply the payo in each state by the price of each state.
C
A
= 35
1
+ 5
2
+ 0
3
= 11.414.
And for the call on C:
C
B
= 5
1
+ 15
2
+ 0
3
= 2.9293.
14. The price of XYZ stock is currently at $100. After one period, the price will move to
one of the following two values: {130, 80}. A $1.00 investment in the risk-free asset
will return $1.05 at the end of the period.
(a) Find the risk-neutral probabilities governing the movement of the stock price.
(b) Find the state prices for each of the states in the following period.
(c) Calculate the price of a $102-strike put directly using the state prices.
Answer:
(a) The risk-neutral probabilities are:
q =
R d
u d
=
1.05 0.80
1.30 0.80
= 0.5.
(b) The state prices are just the discounted risk-neutral probabilities (see Appendix
11B), so each state price is
0.5
1.05
= 0.47619.
(c) The price of the put is the payo at each state multiplied by the state price summed
over all states. Int his case, this works out to
0(0.47619) + 22(0.47619) = 10.476.
15. The price of ABC stock is currently at S = $100. After one period, the price will move
to one of the following two values: {uS, dS}, where {u = 1.2, d = 0.9}. A $1.00
investment in the risk-free asset will return $1.10 at the end of the period.
Sundaram & Das: Derivatives - Problems and Solutions . . . . . . . . . . . . . . . . . . . . . . . . . . . . . . . 146
(a) Find the risk-neutral probabilities governing the movement of the stock price.
(b) For a strike-100 call, nd the delta of the call.
(c) For a strike-100 put, nd the delta of the put.
(d) Compute the dierence between the call delta and the put delta and explain the
answer you get.
Answer:
(a) The risk-neutral probabilities are:
q =
R d
u d
=
1.10 0.9
1.2 0.9
= 2/3
(b) The call delta is equal to the dierence in call values at the next two nodes divided
by the dierence in stock prices at those nodes:

c
=
max(0, 120 100) max(0, 90 100)
120 90
= 2/3.
(c) We can do the same to get the delta of the put:

p
=
max(0, 100 120) max(0, 100 90)
120 90
= 1/3.
(d) The dierence of the two deltas is:

p
= 2/3 (1/3) = 1.
Put-call parity shows that the dierence between the call and put deltas must
always be +1 for European options. From put-call parity, we have
C P = S PV (K).
If the stock price changes by $1, then, by denition, the call and put prices change
by
c
and
d
, respectively, so the change in the left-hand side is
c

p
. Since
put-call parity is an identity, the change in the right-hand side must equal the total
change in the left-hand side which says precisely that
c

p
= 1.
16. In the previous question, if the stock price rises to $110, then
(a) Recompute
c
,
p
.
(b) Explain why the deltas moved in their respective directions.
(c) Conrm that the dierence in the deltas is still equal to +1.
Answer: When the stock price today is $110, then the next periods prices will be $132
and $99 respectively.
Sundaram & Das: Derivatives - Problems and Solutions . . . . . . . . . . . . . . . . . . . . . . . . . . . . . . . 147
(a) The two deltas are now as follows:

c
=
32 0
132 99
= 32/33.

p
=
0 1
132 99
= 1/33.
(b) The call has moved deeper into the money (than it was at the initial node in the
previous question), so its delta has increased. The put is deeper out-of-the-money,
so its delta has decreased.
(c) We conrm that the dierence in deltas is unity:

p
= 32/33 (1/33) = 1.
17. The current price of a stock is $50. The one-period rate of interest is 10%. The up-
move parameter for the stock movement over one period is u = 1.5, and the down-move
parameter is d = 0.5.
(a) If the delta of the call at strike K is 0.5, what is the strike of this option?
(b) What is the delta of the put at the same strike?
(c) What is the price of this put?
Answer:
(a) The formula for the delta of the call is

c
=
C
u
C
d
uS dS
=
max(0, 75 K) max(0, 25 K)
75 25
Since this is given to be 0.50, we can solve the equation to get K = 50.
(b) To nd the delta of the put, we use the relationship

p
= 1.
We have
0.5
p
= 1
Hence,
p
= 0.5.
(c) To nd the put price we compute the risk-neutral probability:
q =
1.1 0.5
1.5 0.5
= 0.60.
Then using the risk-neutral valuation relationship we have the put price:
P =
1
1.1
[0(0.6) + 25(0.4)] = 9.091.
Sundaram & Das: Derivatives - Problems and Solutions . . . . . . . . . . . . . . . . . . . . . . . . . . . . . . . 148
We could have also solved for the put price using replication. In this case, the
replicating portfolio is a short position in the stock of 0.50 units and an investment
of $34.091 at the risk-free rate.(The details are left to the reader.) This gives the
price of the put as
P = 0.5(50) + 34.091 = 9.091.
This the same price we obtained from risk-neutral valuation.
18. (Dicult) The current price of a stock is $100. After one period, this stock may move
to three possible values: {150, 110, 60}. The value of $1.00 invested in the risk-free
asset compounds to a value of $1.05 in one period. Find the upper and lower bounds
of the call price if its strike is $100.
Answer: Note that there are three next-period states and only two securities, so
markets are incomplete in the sense dened in Appendix 11C. That is, not all derivatives
can be replicated using just the two assets.
As Appendix 11C shows, in the presence of market incompleteness, the model will admit
more than one risk-neutral probability or equivalent martingale measure. The price
obtained by using a risk-neutral probabilityi.e., by taking discounted expectations
of option payos under that probabilityis always arbitrage-free (though, of course,
a unique no-arbitrage price exists only if there is a unique risk-neutral probability or
equivalently if the claim is replicable, i.e., the market is complete).
So to nd the minimum and maximum arbitrage-free prices for the given option, we
search over all risk-neutral probabilities in the model and identify those probabilities that
result in the lowest and highest prices, respectively.
Denote a typical risk-neutral probability by {q
1
, q
2
, q
3
}. If we use this probability, the
price for the option is
50q
1
+ 10q
2
+ 0q
3
= 50q
1
+ 10q
2
So, to identify the maximum price, we solve the following problem: we choose (q
1
, q
2
, q
3
)
to maximize
50q
1
+ 10q
2
subject to the following conditions:
1
100
[150q
1
+ 110q
2
+ 60q
3
] = 1.05
Sundaram & Das: Derivatives - Problems and Solutions . . . . . . . . . . . . . . . . . . . . . . . . . . . . . . . 149
q
1
+q
2
+q
3
= 1 q
i
0, i = 1, 2, 3.
The rst of these conditions is simply the risk-neutral pricing condition that that the
expected return on the stock under (q
1
, q
2
, q
3
) be equal to the risk-free rate. The second
condition ensures that (q
1
, q
2
, q
3
) is a probability, that is, that the probabilities are
non-negative and sum to one.
Using the Solver in Excel, we nd the solution to be
{q
1
, q
2
, q
3
} = {0.5556, 0.00, 0.4444}
which results in a call price of
C
max
= 25.2525.
The minimum arbitrage-free call price is similarly solved for by minimizing the same
objective function subject to the same constraints. The solution is
{q
1
, q
2
, q
3
} = {0, 1, 0}
corresponding to a call value of
C
min
= 9.0909.
The bounds are extremely wide in this case.
19. Portfolio insurance: The current price of the stock we are holding is $100. We want to
continue to hold the stock position but modify it so that the portfolio value never drops
below $90. If the stock may move up to $130 or down to $80 after one period, how do
we modify our holding of $100 so as to make sure that it is at least of $90 value at the
end of the period? The rate of simple interest for the period is 10%.
Answer: Portfolio insurance involves adding puts to an existing portfolio so as to achieve
a oor value. The put is itself equivalent to a short position in the underlying portfolio
plus a risk-free investment, so adding the put to the portfolio is equivalent to reducing
our holding of the underlying and taking out an investment at the risk-free rate.
To identify the amount of the stock we should hold and the amount to invest in the
risk-free rate, we simply look to identify the the combination of stock and bonds that
will ensure that the value of the portfolio is at least 90 in the down state (where the
Sundaram & Das: Derivatives - Problems and Solutions . . . . . . . . . . . . . . . . . . . . . . . . . . . . . . . 150
stock price is $80). Let the position in stock be s, and the position in bonds be b. Then,
we need:
80s + 1.10 b = 90.
We also need one more condition, namely that the cost of this position be no more than
the $100 we have. So this leads to another equation:
100s +b = 100
Solving the two equations, we get
s =
2
3
, b =
100
3
We may also compute what outcome we have in the upper node now, when the stock
portfolio has thus been enhanced by portfolio insurance. This outcome is
130s + 1.10 b =
_
130
2
3
_
+
_
1.10
100
3
_
= 123.33
This is lower than the value of 130 that would have resulted from the all-stock position,
but this is the cost we pay for the downside protection.
The following questions relate to the concept of martingale probability measures.
20. What is a martingale measure? What is the role of the martingale measure in nance?
Answer: Let (X
t
) be a stochastic process. If the process is such that for any t, the
expectation of X

(for > t), conditional on all information F


t
available up to t, is
equal to X
t
, then the process X
t
is said to be a martingale. In notational terms, the
process (X
t
) is a martingale if for all t and for all k 0
E[X
t+k
| F
t
] = X
t
The expectations in this denition are obviously with respect to some probability measure
which denes the likelihood oft he various outcomes of (X
t
). That probability measure
is called a martingale measure for the process (X
t
).
Sundaram & Das: Derivatives - Problems and Solutions . . . . . . . . . . . . . . . . . . . . . . . . . . . . . . . 151
21. Does the delta of the option in the binomial tree depend on the risk-neutral probabilities?
Answer: Not in the one-period model. But in general, the delta at any node of the
binomial tree (where S is the stock price at that node) is given by
C
u
C
d
uS dS
where C
u
and C
d
are the values of the option after an up- and down-move respectively,
and u and d are the up- and down-move parameters for the stock price. In a multi-
period model, the values C
u
and C
d
will themselves have been derived from the values
one further period in the future (see Chapter 12 where this method of deriving prices
by backwards induction is described), so reect the risk-neutral probabilities at those
nodes. In this sense, yes, the deltas of options in binomial models will depend on the
risk-neutral probabilities along the tree.
22. In the binomial model, the up move of the stock is set by parameter u, i.e., the stock
goes from S at the start of the period to uS at the end of the period if it moves up.
Likewise, the down-move parameter for the stock is d. The value of 1 plus the interest
rate is specied as R. What is the no-arbitrage relationship between u, d, R? Explain
what happens if this relationship is violated.
Answer: The relationship between these three parameters is
d < R < u
If this were not the case, we must have R u > d or u > d R. In the former case,
the risk-free rate dominates the stock: it always provides as high a return and sometimes
strictly more. An arbitrage prot can be made by shorting the stock and investing the
proceeds at the risk-free rate. In the latter case, the stock dominates the risk-free asset:
there is no circumstance in which it does worse and circumstances in which it does
strictly better. An arbitrage prot can be made by borrowing at the risk-free rate and
investing in the stock.
Note too that the risk-neutral probability q in a binomial model is given by
q =
R d
u d
.
If R > u or R < d, q is no longer a probability, i.e., it no longer lies between 0 and 1.
Sundaram & Das: Derivatives - Problems and Solutions . . . . . . . . . . . . . . . . . . . . . . . . . . . . . . . 152
23. You are given the following one-period-ahead binomial outcomes for a stock, trading at
a current price of S (h is the length of one period measured in years):
_
S exp(

h) with prob q
S exp(

h) with prob 1 q
The continuously compounded interest rate is r. The time interval over which the stock
moves is h. Answer the following questions:
(a) What is a martingale?
(b) If the normalized price of the stock is a martingale, then what is the probability q?
(c) What is the variance of the continuously compounded return on the stock in this
scenario?
Answer:
(a) Martingales as a general mathematical concept were dened in Question 20 above.
(b) The normalized price of S is the price of S dividend by the value of the numeraire
asset. Taking the value of the risk-free asset to be one at the initial node, the
normalized price of S is S at the initial node and

S/e
rh
= e
rh

S after one period,
where the one-period-ahead price

S is given by

S =
_
e

h
S, with probability q
e

h
S, with probability (1 q)
For the normalized price of S to be a martingale, the expectation of its one-period-
ahead value must equal the current price. This means we must have
e
rh
S [qe

h
+ (1 q)e

h
] = S,
so
q =
e
rh
e

h
e

h
e

h
(c) The continuously compounded return on the stock in the upstate is given by
R
s
= ln(Se

h
/S) =

h
and in the downstate is
R
s
= ln(Se

h
/S) =

h
Sundaram & Das: Derivatives - Problems and Solutions . . . . . . . . . . . . . . . . . . . . . . . . . . . . . . . 153
Therefore, the expected stock return is:
E(R
s
) = q

h + (1 q)(

h) =

h(2q 1).
The expected return squared is
E(R
2
s
) = q
2
h + (1 q)
2
h =
2
h.
Therefore, the variance of the return is
V ar(R
s
) = E(R
2
s
) [E(R
s
)]
2
= 4q(1 q)
2
h.
Sundaram & Das: Derivatives - Problems and Solutions . . . . . . . . . . . . . . . . . . . . . . . . . . . . . . . 154
Chapter 12. Binomial Option Pricing
1. Keeping all other parameters the same, if the dividend rate on the stock increases, which
option depreciates less, the American call or the European call? Why?
Answer: Dividends have a negative impact on call values since they decrease the stock
price at maturity. With European options, since the option can only be exercised at
maturity, the full impact of the dividends is felt by the option holder. With American
options, the option can be exercised early before it goes ex-dividend, so the eect of the
dividends can be partially negated. Thus, an American call will drop less in value on
account of dividends than a European call.
2. What condition is required on the movement of stock prices for the binomial tree to be
recombining?
Answer: Let u and d denote the up and down moves in the rst period of a binomial tree,
and let (u
1
, d
1
) and (u
2
, d
2
) denote the up- and down-move pairs in the second period
following an up-move and a down-move in the rst period, respectively. Recombination
in the two-period tree requires that ud
1
= du
2
or that
u
2
d
1
=
u
d
(The same condition evidently holds for recombination in any two-period subtree of a
longer tree.) If the binomial tree has constant parameters (i.e., u and d are the same
at all nodes in the tree), as, this condition is always met. The Cox-Ross-Rubinstein
parametrization of the binomial tree (see Chapter 13) utilizes such a constant-parameter
tree, while the Implied Binomial Tree approach described in Chapter 16 constructs re-
combining trees in which the parameters are not constant from node to node.
3. Why does the payment of dividends usually render the binomial tree into a nonrecom-
bining one? What type of dividend causes the failure of recombination? What type of
dividends do not?
Answer: Cash dividends cause the binomial tree to be non-recombining. For example,
let S be the initial stock price in a two-period binomial tree, and let u and d denote the
up- and down-moves in the binomial tree. Let D be the cash dividend payment at the
end of one period. Then, the two possible ex-dividend prices at the end of one period
are uS D and dS D, respectively. Thus, an initial up move followed by a down
move results in the price
d (uS D) = udS dD
Sundaram & Das: Derivatives - Problems and Solutions . . . . . . . . . . . . . . . . . . . . . . . . . . . . . . . 155
at the end of two periods, while down followed by up results in the price
u (dS D) = udS uD
The rst of these is evidently greater than the second since u > d. As we have shown
in the chapter, however, proportional dividends do not aect recombination. With cash
dividends too, there are methods to split the stock price into two parts, a dividend
component and a remaining component, and to make the tree recombining, as explained
in the chapter.
4. Holding all else constant, if dividends increase, does the dierence between American
calls and puts increase or decrease? Why? What about the dierence between European
calls and puts?
Answer:
In general, both American and European call options fall in value on account of dividend
payments, though American calls may fall by less than European calls on account of
the ability to exercise early. In general, too, both American and European put options
rise in value on account of dividend payments since the payment of dividends lowers the
distribution of stock prices at maturity. Thus, the dierence in value between American
calls and puts will becomes smaller (less positive or more negative) as a consequence of
an increase in dividends, as will the dierence in value between European calls and puts.
5. How would you know from examining the risk-neutral probabilities on a binomial tree if
the model is free from arbitrage?
Answer: As explained in Chapter 11, it is necessary and sucient for the absence of
arbitrage in binomial models that a risk-neutral measure exists such that the probability
q of an up move at each node lies between 0 and 1.
6. Explain briey in a heuristic manner why option replication on a binomial tree is a
dynamic strategy.
Answer: The portfolio that replicates an option at any node captures the value of the
option at that node. As, for instance, a call moves deeper into-the-money, it is more
likely to get exercised, so its replicating portfolio more closely resembles a long position in
one unit of the stock and a borrowing of PV (K) for repayment at maturity T. Similarly,
as it moves more out of the money, its value increasingly resembles the null portfolio
that holds neither stock nor cash. These principles are reected in replication on the
binomial tree, as illustrated in Section 12.3.
Sundaram & Das: Derivatives - Problems and Solutions . . . . . . . . . . . . . . . . . . . . . . . . . . . . . . . 156
For example, consider any node S on the binomial tree and its two subsequent nodes
{S
u
, S
d
}; lets call this a triplet. We know that to replicate an option value at node
S, we must form a portfolio of the stock and bond at node S such that (a) the value of
the portfolio at node S coincides with the option value at S, and (b) the value of this
portfolio at nodes S
u
and S
d
coincides exactly with the values of the option at those two
nodes. However, the portfolio of stock and bonds will be dierent at each node since
the option values change as we move to another node on the same tree. For example, as
the stock price increases from S to S
u
, a call will generally be worth more as the option
has moved into-the-money, and so would have a higher delta. In order to replicate the
option, we would need to modify the holding of stock and bonds as prices evolve.
7. Explain what is meant by a self-nancing replicating strategy.
Answer: Option replication consists of dynamically maintaining a portfolio of the un-
derlying (e.g., stock) and cash/bonds (the risk-free asset) so as to mimic the value of
the option at all times. Since the option value changes with time, so does the com-
position of the replicating portfolio, so re-adjustment of the holdings in the portfolio
(rebalancing as it is called) becomes necessary to stay replicated. The rebalancing
is said to be self-nancing if not net injection or withdrawal of funds is caused by the
rebalancingfor example, if any additional investment in the risk-free asset is paid for
by the sale of stock. Without the self-nancing property, the initial cost of a replicating
portfolio cannot be taken as the fair value of an option, since further injections or
withdrawals of funds may be required to maintain the portfolio.
8. Suppose we used a trinomial tree with three replicating securities instead of a binomial
tree with two securities. Would the dynamic replication be self-nancing?
Answer: Yes, the replication will still be self-nancing. The property of self-nancing
has nothing to do with the number of branches on the tree. Mathematically speaking,
as long as the market is complete, i.e., we have enough securities to perfectly replicate
at each node (in a trinomial model, this means having three securities as the question
assumes), we will be able to obtain a self-nancing strategy.
9. Suppose you have two states of the world and two assets. The prices of both assets in
each of the two states is known. What conditions are needed for a derivative security
that is a function of the two assets to be replicable?
Answer: Some notation is needed to answer this question. Let the initial prices of the
two assets be S
1
and S
2
, respectively. Denote the two states by u and d, and suppose
the prices of the two assets in the two states are denoted by {S
u
1
, S
u
2
} and {S
d
1
, S
d
2
},
respectively. Finally, let D
u
and D
d
denote the value of the derivative at these two
Sundaram & Das: Derivatives - Problems and Solutions . . . . . . . . . . . . . . . . . . . . . . . . . . . . . . . 157
states. Suppose we consider a possible replicating portfolio consisting of X
1
of asset 1
and X
2
of asset 2. The portfolio replicates the derivative if
X
1
S
u
1
+X
2
S
u
2
= D
u
X
1
S
d
1
+X
2
S
d
2
= D
d
Replication is feasible when a solution to these equations exists. It is possible that no
solution may exist. (For example, suppose that S
u
1
= S
u
2
= 1.25, S
1
d
= S
2
d
= 0.80, and
D
u
= 10, D
d
= 0. Then, the rst equation gives us X
1
+ X
2
= 8, while the second
gives us X
1
+X
2
= 0, which are inconsistent.) Mathematically, a solution is guaranteed
to exist when the following matrix has full rank:
_
S
u
1
S
u
2
S
d
1
S
d
2
_
i.e., when S
u
1
S
d
2
S
u
2
S
d
1
= 0.
10. (Dicult) In a two-period binomial tree, let the volatility at any node be given by
= ln(u/d)
where u and d are as usual, respectively, the up-shift and down-shift values for stock
price moves on the tree. Given a starting stock price of $50, suggest one way to draw
a two-period recombining stock tree when the volatility of the rst period is = 0.20
and in the second period is 0.25.
Answer: As in the answer to Question 2, we adopt the following notation. Let u and d
denote the up and down moves in the rst period of a binomial tree, and let (u
1
, d
1
) and
(u
2
, d
2
) denote the up- and down-move pairs in the second period following an up-move
and a down-move in the rst period, respectively. This gives us a total of six unknowns:
((u, d), (u
1
, d
1
), (u
2
, d
2
)). Since we are given a volatility of 0.20 at the initial node and
0.25 after one period, we obtain three restrictions:
ln
_
u
d
_
= 0.20
ln
_
u
1
d
1
_
= 0.25
Sundaram & Das: Derivatives - Problems and Solutions . . . . . . . . . . . . . . . . . . . . . . . . . . . . . . . 158
ln
_
u
2
d
2
_
= 0.25
Moreover, since we need the tree to be recombining, an initial up-move followed by
a down move must result in the same price as an initial down move followed by an
up-move, i.e., we must have ud
1
= du
2
or
u
2
d
1
=
u
d
.
This gives us only four conditions in the six unknowns, so there are clearly many feasible
solutions. To pin down a specic solution, we add two more (somewhat arbitrary)
restrictions. First, suppose that in the rst period, we have u = 1/d:
ud = 1
Finally, suppose that the middle price after two periods is equal to the initial price of
the stock, i.e., that
ud
1
= 1
Solving these six equations gives us the six unknown values as (approximately)
u = u
2
= 1.1052, d = d
1
= 0.9048, u
1
= 1.1618, d
2
= 0.8607
This gives us the initial stock price as 50; the possible prices after one period as 55.26
and 45.24; and the possible prices after two periods as 64.20, 50, and 38.94.
11. You are given the following tree of stock prices. In addition, the rate of interest per
period is constant at 2%. Find the risk-neutral probabilities of the stock movements
from each node on the tree. Are these probabilities the same? If not, explain whether
the tree is a valid one.
Sundaram & Das: Derivatives - Problems and Solutions . . . . . . . . . . . . . . . . . . . . . . . . . . . . . . . 159
45
60
30
80
50
20
Answer: The tree has 3 nodes from which probabilities need to be computed. The rst
node has a stock price of 45 (we will denote the up-move probability from this node as
q). We apply the usual formula to determine the risk-neutral probability, i.e.,
q =
R d
u d
=
1.02 2/3
4/3 2/3
= 0.53.
Similarly, denote the risk-neutral probability following an initial up-move (to 60) by q
u
and following an initial down-move (to 30) by q
d
. Straightforward calculations reveal
that
q
u
=
1.02 5/6
8/6 5/6
= 0.3733.
q
d
=
1.02 2/3
5/3 2/3
= 0.3533.
Sundaram & Das: Derivatives - Problems and Solutions . . . . . . . . . . . . . . . . . . . . . . . . . . . . . . . 160
The risk-neutral probabilities at the three nodes are dierent, reecting the changing
values of u and d over the tree. There is nothing conceptually problematic about this;
the tree is still arbitrage-free and is a perfectly valid pricing tree.
12. On the tree given in the previous problem, price an American call and an American put.
Both options are assumed to be at strike $45.
Answer: We begin with the call option. At maturity the payos will be {35, 5, 0}
corresponding to stock prices {80, 50, 20}.
The upper node at time 1 will have a call option value of
[35q
u
+ 5(1 q
u
)]/1.02 = 15.88.
The lower node at time 1 will have a call option value of
[5q
d
+ 0(1 q
d
)]/1.02 = 1.732.
Using these two values we get the value of the call at time zero:
[15.88q + 1.732(1 q)]/1.02 = 9.05.
This is the price of the American call. Note that we do not need to check for early
exercise since the stock does not pay dividends.
We now proceed onto the put option. At maturity the payos are {0, 0, 25} correspond-
ing to stock prices {80, 50, 20}. If unexercised at that node, the value of the put at the
upper node is given by
[0q
u
+ 0(1 q
u
)]/1.02 = 0.
Since the exercise value is negative, the value of the American put at the upper node is
the value of leaving it unexercised which is zero.
If unexercised at the lower node, the value of the put at that node is
[0q
d
+ 25(1 q
d
)]/1.02 = 15.85.
Exercising the put early results in a value of 45 30 = 15, which is less than the value
of the put if unexercised. Hence, the value of the American put at this node is 15.85.
Using these two values we get the value of the put at time zero:
[0q + 15.85(1 q)]/1.02 = 7.30.
Sundaram & Das: Derivatives - Problems and Solutions . . . . . . . . . . . . . . . . . . . . . . . . . . . . . . . 161
13. Again, using the same tree as in the previous two questions, what is the delta of the call
and the put at times 0 and 1?
Answer: We apply the usual formulae (see Section 12.1) to get the deltas for the call
and the put at each node. At time zero, the delta of the call is
15.88 1.732
60 30
= 0.4717.
The delta of the put is
0 15.85
60 30
= 0.53.
At the upper node at time 1, the deltas are:
Call delta =
35 5
80 50
= 1
Put delta =
0 0
80 50
= 0
At the lower node at time 1, the deltas are:
Call delta =
5 0
50 20
= 0.1667
Put delta =
0 25
50 20
= 0.8333
Notice that the sum of the absolute values of all deltas at each node is always 1. Even
though this is a property of European options and the current options are American,
the property still holds because, as we saw in the answer to the previous question, the
options are never exercised early.
14. The initial stock price is $100. The stock moves up each period by a factor of 1.3
and down by a factor of 0.8. If the simple interest rate per period is 1%, what is the
risk-neutral probability of an up move in the stock price?
Draw the stock price tree for three periods and price an Europan call option for three
periods at strike $105.
Answer: The risk-neutral probability of an up move is
q =
R d
u d
=
1.01 0.8
1.3 0.8
= 0.42.
The stock tree will be as follows:
Sundaram & Das: Derivatives - Problems and Solutions . . . . . . . . . . . . . . . . . . . . . . . . . . . . . . . 162
Stock Price Tree
100.00 130.00 169.00 219.70
80.00 104.00 135.20
64.00 83.20
51.20
Using the usual arguments outlined in the text, we generate the tree for the evolution
of prices of the European call.
European Call Tree
17.245 34.258 65.040 114.700
5.222 12.558 30.200
0.000 0.000
0.000
The initial price of the European call is $17.245.
Remark These trees are to be read in the obvious way. From each node, the two prices
that result are either the next number in the same row (if an up-move occurs) or the
number immediately below that (if a down-move occurs). For example, in the stock
price tree, the initial price is 100 from which the price goes to 130 or 80. From the price
of 80, the two possible prices are 104 and 64; from 104, they are 135.20 and 83.20; and
so on.
The Call Tree is read similarly. The initial call value is 17.245. Following an up-move in
the stock price, the call value goes to 34.258, while following a down move it goes to
5.222. From 5.222, the possible prices in the next period are 12.558 (if the stock price
goes up) and 0 (if it goes down), etc.
15. The initial stock price is $100. The stock moves up each period by a factor of 1.3 and
down by a factor of 0.8. If the simple interest rate per period is 1%, nd the prices of
three period European and American puts, and state the early exercise premium amount.
Answer: As before the risk-neutral probability is 0.42 of an up-move, since the param-
eters are the same as that of the previous question. The stock tree is as follows (the
trees are to be read in the way mentioned in the Remark at the end of the answer to
Question 14):
Stock Price Tree
100.00 130.00 169.00 219.70
80.00 104.00 135.20
64.00 83.20
51.20
Sundaram & Das: Derivatives - Problems and Solutions . . . . . . . . . . . . . . . . . . . . . . . . . . . . . . . 163
Using standard backward recursion, we generate the tree for the European put.
European Put Tree
19.157 7.189 0.000 0.000
28.153 12.519 0.000
39.960 21.800
53.800
The initial price of the European put is $19.157.
The American call also gets priced using backward recursion except that we check at
each node whether it might be preferable to also exercise the put early. The tree for the
American put is as follows:
American Put Tree
19.500 7.189 0.000 0.000
28.750 12.519 0.000
41.000 21.800
53.800
The intial price of the American put is is $19.500. Comparing the trees for the American
and European puts we see that there is only one node at which early exercise is optimal.
This is the lowest price node after 2 periods. The stock price at this node is $64. The
continuation value of the option at this node is the same as that of the European option,
i.e., a value of $39.96. However, at a price of $64 and strike $105, if we exercise the
put, we get a value of $41, which is higher than its continuation value. Hence, it is
optimal to exercise at this node, and accordingly, the value of this node is set to $41,
which is higher than that of the European option.
Note also that all prior node values are accordingly increased, since the continuation
values for earlier nodes are impacted by the optimal exercise at the lowest node at the
end of the second period.
The value of early exercise is the dierence in value between the American and European
puts, i.e.,
19.500 19.157 = $0.343.
16. When there are no dividends, the early exercise of an American put depends on a trade-
o between insurance value (which comes from volatility) and time value (a function
of interest rates). Thus, for example, for a given level of volatility, early exercise of the
put becomes more likely if interest rates are higher. This question provides a numerical
illustration.
Sundaram & Das: Derivatives - Problems and Solutions . . . . . . . . . . . . . . . . . . . . . . . . . . . . . . . 164
Consider a two-period binomial model with u = 1.10 and d = 0.90. Suppose the initial
stock price is 100, and we are looking to price a two-period American put option with a
strike of K = 95.
(a) First, consider a low interest rate of R = 1.02. Show that early exercise of the
American put is never optimal in this case.
(b) Now consider a high interest rate of R = 1.05. Show that it now becomes
optimal to exercise the put early in some circumstances. What is the early exercise
premium in this case?
Answer: We are given S = 100, u = 1.10, d = 0.90, and K = 95. The stock price tree
that arises under these parameters is described in the gure below.

3
Q
Q
Q
Q
Q
Q
Q
Q
Qs

3
Q
Q
Q
Q
Q
Q
Q
Q
Qs

3
Q
Q
Q
Q
Q
Q
Q
Q
Qs
100
110
90
121
99
81
Given these stock prices and the strike of K = 95, the terminal option payos (i.e., the
payos if the option is not exercised till maturity) are
P
uu
= max{0, K u
2
S} = 0
P
ud
= max{0, K udS} = 0
P
du
= max{0, K duS} = 0
P
dd
= max{0, K d
2
S} = 14
Sundaram & Das: Derivatives - Problems and Solutions . . . . . . . . . . . . . . . . . . . . . . . . . . . . . . . 165
(a) When R = 1.02:
In this case, the risk-neutral probability q is
q =
R d
u d
=
1.02 0.90
1.10 0.90
= 0.60.
First consider the node uS = 110. The value from immediate exercise at the node
uS is (K uS) = 15. The value from not exercising is P
uu
= 0 (if the price
goes up again) or P
ud
= 0 (if the price goes down). Therefore the value from not
exercising is
P
u
= max{20, 0} = 0,
and the optimal decision is not to exercise at the node uS.
Now consider the node dS = 90. The value from immediate exercise at this node
is (K dS) = 5. The value from not exercising at this node is either P
du
= 0 (if
the price goes up), or P
dd
= 14 (if the price goes down again). Therefore, using
the risk-neutral probabilities and discounting, the value from not exercising is
1
1.02
[(0.60)(0) + (0.40)(14)] = 5.49.
It now follows that
P
d
= max{5, 5.49} = 5.49,
so that early exercise is not optimal at this point either.
To nd the initial value P of the put, note that immediate exercise at the node
S = 100 leads to a payo of max{K S, 0} = 5, while not exercising results in
a value of P
u
= 0 if the node uS results and P
d
= 5.49 if dS results. Thus, it is
optimal to not exercise early at S either, so the initial value of the put is
P =
1
1.02
[(0.60)(0) + (0.40)(5.49)] = 2.15.
Observe that, as stated in the question, early exercise is never optimal for this
put, so the value of this American put is also the value of an otherwise identical
European put, i.e., the early exercise premium is zero.
(b) When R = 1.05:
Now, the risk-neutral probability q is
q =
R d
u d
=
1.05 0.90
1.10 0.90
= 0.75.
At the node uS = 110, the analysis is identical to the case above, so early exercise
is not optimal and the value P
u
of the option at this node is zero.
Sundaram & Das: Derivatives - Problems and Solutions . . . . . . . . . . . . . . . . . . . . . . . . . . . . . . . 166
At the node dS = 90, the value from immediate exercise is (K dS) = 5. The
value from not exercising at this node is again either P
du
= 0 (if the price goes
up), or P
dd
= 14 (if the price goes down again). Therefore, using the risk-neutral
probabilities and discounting, the value from not exercising is
1
1.05
[(0.75)(0) + (0.25)(14)] = 3.33.
It now follows that
P
d
= max{5, 3.33} = 5,
and early exercise is optimal at this node.
To nd the initial value P of the put, note that immediate exercise at the node
S = 100 leads to a payo of max{K S, 0} = 5, while not exercising results
in a value of P
u
= 0 if the node uS results and P
d
= 5 if dS results. Thus, it is
optimal to not exercise early at S either, so the initial value of the put is
P =
1
1.05
[(0.75)(0) + (0.25)(5)] = 1.19.
Thus, early exercise does become important at r = 1.05 as stated in the question.
The value of an otherwise identical European put is seen to be
P
E
=
1
1.05
[(0.75)(0) + (0.25)(3.33)] = 0.79
so the early exercise premium is now 1.19 0.79 = 0.40.
17. Consider a two-period example with S = 100, u = 1.10, d = 0.90, R = 1.02, and a
dividend of $5 after one period. Is early exercise of a call optimal given these parameters?
Answer: Yes. This is the example in Section 12.7 of the book.
18. We repeat the previous question with higher volatility and interest rates and with lower
dividends. Consider a two-period binomial tree with the following parameters: S = 100,
u = 1.20, d = 0.80, and R = 1.10. Suppose also that a dividend of $2 is expected after
one period.
(a) Compute the risk-neutral probability in this world.
(b) Find the tree of prices of an American call option with a strike of 100 expiring in
two periods.
(c) What is the early-exercise premium?
Sundaram & Das: Derivatives - Problems and Solutions . . . . . . . . . . . . . . . . . . . . . . . . . . . . . . . 167
Answer: We are given S = 100, u = 1.20, and d = 0.80 and R = 1.10, and that a
dividend of $2 will be paid after one period. The resulting stock price tree and risk-
neutral probability are shown below.

@
@
@
@
@
@R

>
Z
Z
Z
Z
Z
Z~

>
Z
Z
Z
Z
Z
Z~
62.40
93.60
94.40
141.60
118
78 80
120
100
(a) The risk-neutral probability is
q =
1.10 0.80
1.20 0.80
= 0.75.
(b) To calculate the value of the option, we begin at the end of the tree. Conditional
on not having been exercised so far, the option values are:
C
uu
= max{141.60 100, 0} = 41.60
C
ud
= max{94.40 100, 0} = 0
C
du
= max{93.60 100, 0} = 0
C
dd
= max{77.40 100, 0} = 0
Now consider the option value at the node uS:
If the option is exercised at the cum-dividend point, the value is (120100) =
20.
If the option is exercised at the ex-dividend point, the value is (118100) = 18.
Sundaram & Das: Derivatives - Problems and Solutions . . . . . . . . . . . . . . . . . . . . . . . . . . . . . . . 168
If the option is left unexercised, then, in the next period it brings in either 41.60
(if the price goes up again) or zero (if it goes down). Using the risk-neutral
probability, the value of not exercising is
1
1.10
[(0.75)(41.60) + (0.25)(0)] = 28.36
Comparing these alternatives, it is immediate that the value of the option at the
node uS is 28.36 and it is not optimal to exercise the option at this point.
A simple argument establishes that it is not optimal to exercise early at the node
dS, and the value of the call at this node is zero.
Finally, the initial node. Since the option is at-the-money at this node, immediate
exercise cannot be optimal. After one period, the option is worth 28.36 (if the
price goes up), or zero (if the price goes down). Therefore, the initial value of the
option is
C =
1
1.10
_
3
4
(28.36) +
1
4
(0)
_
= 19.33.
(c) Since early exercise is never optimal for this call, the early exercise premium is zero,
and the European option value coincides with the American.
19. The payment of a dividend on the underlying stock increases the value of a put option
since it lowers the stock price distribution at maturity. This question provides a
numerical illustration.
Let a two-period binomial tree be given with the following parameters: S = 100, u =
1.10, d = 0.90, and R = 1.05. Consider a two-period American put option with a strike
of 90. Note that this put is quite deep out-of-the-money at inception.
(a) What is the value of the American put given these parameters?
(b) Now suppose a dividend of $4 is paid at the end of the rst period. What is the
new price of the put?
Answer: We are given S = 100, u = 1.10, d = 0.90 and R = 1.05. Thus, the
risk-neutral probability is
q =
1.05 0.90
1.10 0.90
= 0.75.
(a) The tree of stock prices in this case is shown in the gure below:
Sundaram & Das: Derivatives - Problems and Solutions . . . . . . . . . . . . . . . . . . . . . . . . . . . . . . . 169

3
Q
Q
Q
Q
Q
Q
Q
Q
Qs

3
Q
Q
Q
Q
Q
Q
Q
Q
Qs

3
Q
Q
Q
Q
Q
Q
Q
Q
Qs
100
110
90
121
99
81
To calculate the value of the option, we begin at the end of the tree. Conditional
on not having been exercised so far, the option values are:
P
uu
= max{90 121, 0} = 0
P
ud
= max{90 99, 0} = 0
P
du
= max{90 99, 0} = 0
P
dd
= max{90 81, 0} = 9
Now consider the option value at the node uS:
If the option is exercised at this point, the value is (90 110) = 20.
If the option is left unexercised, it brings in 0 for sure next period (since both
P
uu
and P
ud
are zero). Therefore, the value of leaving the option unexercised
is zero.
It follows that the value of the option at the node uS is zero, and the optimal
action at this node is to leave the option unexercised.
Next consider the node dS:
If the option is exercised early at this point, the value is (90 90) = 0.
If the option is left unexercised, it brings in either P
du
= 0 (if the price goes
up in the second period), or P
dd
= 9 (if the price goes down again). Since the
risk-neutral probability of an up move is 0.75, the value of leaving the option
unexercised is
1
1.05
_
3
4
(0) + (
1
4
(9)
_
= 2.14
Sundaram & Das: Derivatives - Problems and Solutions . . . . . . . . . . . . . . . . . . . . . . . . . . . . . . . 170
It follows from a comparison of these alternatives that the value of the option at
the node dS is 2.14, and the optimal action at this node is to not exercise the
option early at this point either.
Finally, the initial node. From what we have shown above, after one period, the
option is worth 0 (if the price goes up), or 2.14 (if the price goes down). Therefore,
the initial value of the option is
P =
1
1.05
_
3
4
(0) +
1
4
(2.14)
_
= 0.51
(b) In this case, a dividend of $4 will be paid after one period. The resulting stock
price tree is shown below.

@
@
@
@
@
@R

>
Z
Z
Z
Z
Z
Z~

>
Z
Z
Z
Z
Z
Z~
77.40
94.40
95.40
116.60
106
86 90
110
100
To calculate the value of the option, we begin at the end of the tree. Conditional
on not having been exercised so far, the option values are:
P
uu
= max{90 116.6, 0} = 0
P
ud
= max{90 95.4, 0} = 0
P
du
= max{90 84.6, 0} = 0
P
dd
= max{90 77.4, 0} = 12.60
Now consider the option value at the node uS:
If the option is exercised early at the cum-dividend point, the value is (90
110) = 20.
Sundaram & Das: Derivatives - Problems and Solutions . . . . . . . . . . . . . . . . . . . . . . . . . . . . . . . 171
If the option is exercised early at the ex-dividend point, the value is (90106) =
16.
If the option is left unexercised, it brings in 0 for sure next period (since both
P
uu
and P
ud
are zero). Therefore, the value of leaving the option unexercised
is zero.
It follows from comparing these three alternatives that the value of the option at
the node uS is zero, and the optimal action at this node is to leave the option
unexercised.
Next consider the node dS:
If the option is exercised early at the cum-dividend point, the value is (90
90) = 0.
If the option is exercised early at the ex-dividend point, the value is (9086) =
4.
If the option is left unexercised, it brings in either P
du
= 0 (if the price goes
up in the second period), or P
dd
= 12.60 (if the price goes down again). Since
the risk-neutral probability of an up move is 0.75, the value of leaving the
option unexercised is
1
1.05
_
3
4
(0) + (
1
4
(12.60)
_
= 3.0.
It follows from a comparison of these alternatives that the value of the option at
the node dS is 4.0, and the optimal action at this node is to exercise the option
early at the ex-dividend point.
Finally, the initial node. From what we have shown above, after one period, the
option is worth 0 (if the price goes up), or 4.0 (if the price goes down). Therefore,
the initial value of the option is
1
1.05
_
3
4
(0) +
1
4
(4.0)
_
= 0.952.
20. In the absence of dividends, the holder of a European call always benets from an
increase in maturity since the insurance value and time value of the call both increase.
However, for the holder of a European put in this case, insurance value increases but
time value decreases, so the put value could increase or decrease. In general, for a given
level of volatility, if interest rates are high, the time-value eect will outweigh the
insurance-value eect, so European put values will decrease as maturity increases; but
if interest rates are low, the insurance-value eect will dominate, so the put value will
increase. This question illustrates these arguments.
Consider a binomial model with parameters S = 100, u = 1.10, and d = 0.90, and a
European put with a strike of K = 100.
Sundaram & Das: Derivatives - Problems and Solutions . . . . . . . . . . . . . . . . . . . . . . . . . . . . . . . 172
(a) First, consider a high interest rate environment where R = 1.02 (1 plus the
interest rate). We can see that with these parameter values, a one-period put is
worth 3.92, but a two-period European put is worth only 3.38. The increase in
maturity hurts the put holder because the insurance-value eect is outweighed by
the time-value eect.
(b) Now consider a low interest-rate environment where R = 1.00. Show that in
this case, the one-period put is worth less than the two-period put.
Answer: Part (a) of this question was worked out in Chapters 11 (the one-period prob-
lem) and 12 (the two-period problem). For part (b), we are given the following input
information: S = 100, K = 100, u = 1.10, d = 0.90, R = 1.00. Thus, the risk-neutral
probability is
q =
R d
u d
=
1
2
.
A one-period put pays P
u
= 0 in state u and P
d
= 10 in state d; thus, its initial value is
P(1) =
1
R
[qP
u
+ (1 q)P
d
] = 5.
A two-period put pays P
uu
= 0 in the state u
2
S, P
ud
= 1 in the state udS, and P
dd
= 19
in the state d
2
S. Thus, its initial value is
P(2) =
1
R
2
_
q
2
P
uu
+ 2q(1 q)P
ud
+ (1 q)
2
P
dd

= 5.25.
Thus, the value of the two-period put is now more than that of the one-period put.
21. Consider a binomial tree model with u = 1.05 and d = 0.90. Suppose the per-period
interest rate is R = 1.02. Suppose the initial stock price is 100.
(a) What is the risk-neutral probability?
(b) Calculate the value of an American put option on the stock with a maturity of two
periods and a strike of 95.
(c) Compute the early-exercise premium.
Answer: We are given S = 100, u = 1.05, d = 0.90, and R = 1.02. The stock price
tree and risk-neutral probability given these parameters is described below. Since the
calculations have been described in many previous questions, we provide only summary
results here.
Sundaram & Das: Derivatives - Problems and Solutions . . . . . . . . . . . . . . . . . . . . . . . . . . . . . . . 173

3
Q
Q
Q
Q
Q
Q
Q
Q
Qs

3
Q
Q
Q
Q
Q
Q
Q
Q
Qs

3
Q
Q
Q
Q
Q
Q
Q
Q
Qs
100
105
90
110.25
94.50
81
(a) The risk-neutral probability is q = 0.80.
(b) Early exercise is optimal at the node dS = 90. The initial price of the American
put is P = 1.06.
(c) The European put costs 0.69. Therefore, the early exercise premium is 0.37.
22. The initial stock price is $50. The up move in the stock price is modulated by factor
u = 1.2, and the down move is d = 0.9. One dollar invested at the beginning of a
period returns $1.05 at the end of the period.
Draw a two-period stock price tree for this stock. Then price a two-period European
call with a strike price of 50.
Find the replicating portfolio at the initial node on the tree. Show that this replicating
portfolio does mimic the price of the call at both subsequent nodes at time 1 on the
option tree.
Answer: First we compute the risk-neutral probabilities on the tree:
q =
R d
u d
=
1.05 0.9
1.2 0.9
= 0.5.
The stock price tree for two periods is as follows (the trees are to be read in the way
mentioned in the Remark at the end of the answer to Question 14):
Sundaram & Das: Derivatives - Problems and Solutions . . . . . . . . . . . . . . . . . . . . . . . . . . . . . . . 174
Stock Tree
50.0 60.0 72.0
45.0 54.0
40.5
Using standard backward recursion, we get the following call option tree.
Call Tree
6.8028 12.3810 22.0000
1.9048 4.0000
0.0000
Hence, the call option value is $6.80.
The replicating portfolio at time zero for a call will comprise a long position in the stock
and short position in the risk-free asset. The stock position is simply the delta of the
option, i.e.,
=
12.3810 1.9048
60 45
= 0.69841.
The position in bonds (B) will satisfy the following pricing equation:
6.8028 = 50 +B = 50(0.69841) +B
Solving, we get B = 28.118. Let us check now that the replicating portfolio does
indeed replicate the option value at time one.
In the upper node, the value of the replicating portfolio will be
60 + 1.05B = 60(0.69841) + 1.05(28.118) = 12.381
Hence, this matches the value of the option at the upper node.
In the lower node, the value is:
45 + 1.05B = 1.9048,
which is exactly the price of the call.
Note: It is important to recognize here that since the replicating portfolio from time
zero is able to match call values at time 1, it is also able to fully nance the replicating
portfolio between times 1 and 2. This is why the replicating strategy is known as self-
nancing no further injection of money is required to mimic the option from node to
node.
Sundaram & Das: Derivatives - Problems and Solutions . . . . . . . . . . . . . . . . . . . . . . . . . . . . . . . 175
23. Suppose the initial price of the stock is $100. The binomial process has an upshift
u = 1.5 and a downshift d = 0.6 per period. The interest rate per period is assumed
to be zero. What is the risk-neutral probability that the stock nishes above a price of
$200 after six periods? What is the price of the six-period call at a strike of $200?
Answer: The risk-neutral probability of an up move is
1 0.6
1.5 0.6
=
0.4
0.9
= 0.4444.
The generic terminal value of the stock price will be given by the following formula:
S
T
= 100u
j
d
6j
where j denotes the number of upward moves made by the stock, and correspondingly,
(6 j) is the number of downward moves. The risk-neutral probability of having j up
moves is given by (see Appendix 12A):
Prob(j) =
_
6
j
_
0.4444
j
(1 0.4444)
6j
The following table lists the values of the terminal stock price S
T
and the corresponding
risk-neutral probabilities:
j S
T
Probability
6 1139.0625 0.007707347
5 455.625 0.0578051
4 182.25 0.180640937
3 72.9 0.301068228
2 29.16 0.282251463
1 11.664 0.141125732
0 4.6656 0.029401194
The risk-neutral probability that the stock ends up above a price of 200 is
0.0077 + 0.0578 = 0.0655
The value of a call option at strike 200 is
(1139.0625 200)0.0077 + (455.625 200)0.0578 = 22.01
Recall that interest rates are zero, so there is no discounting required.
Sundaram & Das: Derivatives - Problems and Solutions . . . . . . . . . . . . . . . . . . . . . . . . . . . . . . . 176
24. (Dicult) Using values for u = 1.03 and d = 0.98 and an initial stock price of $50,
compute and plot the nal risk-neutral probability distribution of the stock price after
100 periods. The interest rate is zero. What distribution does this remind you of?
Answer: The risk-neutral probability of an upmove is q = 0.4. The binomial probability
function used in the previous question may be applied here to determine the probability
of all the nal stock prices. The binomial tree (in returns) converges to the normal dis-
tribution, and the stock tree converges to the lognormal distribution, which is described
in detail in the next chapter. The plot is as follows:
-0.01
0
0.01
0.02
0.03
0.04
0.05
0.06
0.07
0.08
0.09
0 25 50 75 100 125 150
Stock Price
P
r
o
b
a
b
i
l
i
t
y
25. The price today of stock XYZ is $100. Each period on a stock binomial tree is of length
two months i.e., 0.1667 of a year. The annualized risk-free rate on a continuously
compounded basis is 5%. The annualized dividend rate on the stock is 2% continuously
compounded. The dividend is paid as a percentage of the stock value at the end of
period.
The up move (after adjusting for downward drift from the dividends) in the stock is driven
by the factor u = 1.167618, and the down move (also after the eect of dividends) is
modulated by d = 0.842289.
(a) What is the risk-neutral probability of an up move in the price?
Sundaram & Das: Derivatives - Problems and Solutions . . . . . . . . . . . . . . . . . . . . . . . . . . . . . . . 177
(b) Compute the stock tree for three periods (i.e., for a six-month horizon).
(c) Based on this stock tree, compute the value of the dividends paid at the end of
each period.
(d) Now present value the terminal prices of the stock tree. Weight each value by its
probability of occurring. What is the present value you get? Does this strike you
as strange? Why or why not?
(e) Price the European call option at a strike of 100 for a maturity of six months.
(f) Price the American call option at a strike of 100 for a maturity of six months.
Assume that if you exercise at a given node on the tree, you do not get the
dividends for that period, but only for subsequent periods.
(g) Price the European put. Same terms as the calls.
(h) Price the American put. Same terms as the calls.
Answer:
(a) The risk-neutral probability is
e
(0.050.02)0.1667
0.842289
1.167618 0.842289
= 0.500182.
(b) Using u and d, the stock tree is as follows (the trees are to be read in the way
mentioned in the Remark at the end of the answer to Question 14):
100.00 116.76 136.33 159.19
84.23 98.35 114.83
70.94 82.84
59.76
(c) We now compute the dividends paid at the end of each period. Since the dividends
are a fraction of the price at the end of the period, and are paid continuously
compounded, we have for example, the dividend at the last period at the topmost
node (based on a stock price of 159.19:
dividend = 159.19
_
e
0.020.1667
1
_
= 0.5315.
Similarly, we may compute the entire tree of dividends:
0.0000 0.3899 0.4552 0.5315
0.2812 0.3284 0.3834
0.2369 0.2766
0.1995
(d) We may present value the terminal stock prices by weighting each one by their
probabilities and discounting back on the tree to time zero. The tree representing
these values is as follows:
Sundaram & Das: Derivatives - Problems and Solutions . . . . . . . . . . . . . . . . . . . . . . . . . . . . . . . 178
99.01 115.99 135.88 159.19
83.67 98.02 114.83
70.71 82.84
59.76
So, the present stock price is not equal to $100 but only 99.01. Why is this? This
is so because the stock value today is equal to its expected value at the end of six
months discounted back to today, and the present value of all its dividends. We
left out the dividends in the above calculation, which is why the total value is only
99.01.
We need to add back the dividends to conrm that the stock is fairly valued.
Hence, we take the termnal stock prices, add on the dividends (see the dividend
tree above) and then discount back values, taking care at each stage to remember
to add on the dividends. The resultant tree is as follows:
100.00 117.15 136.79 159.72
84.51 98.68 115.22
71.18 83.11
59.96
We can see that dividends are added, and now, the stock price is indeed equal to
100. In fact we have just implemented the valuation concept that a stock price is
the present value of a stream of future dividends and terminal value.
(e) The tree for the European call is given below. We compute this using backward
recursion, and discounting in a continuous compounding way.
12.65 21.86 36.71 59.19
3.65 7.36 14.83
0.00 0.00
0.00
The European call is worth $12.65.
(f) Pricing an American call when dividends are non-zero requires careful handling.
At each node, we need to compare the value of holding the option for one more
period versus the value that may be obtained from immediate exercise. The value
of continuing to hold the option is just the present value of the next periods option
values on the tree.
The value of exercise at any node on the tree is the dierence between the stock
price and the exercise price plus the expected present value of all future dividends.
So, as a rst step, we need to compute the present value of future dividends at each
node of the tree. We develop a tree of these values by starting at the last period
and discounting cumulative future dividends to each node by backward recursion.
The resultant tree is as follows (the reader is advised to reproduce the same):
Sundaram & Das: Derivatives - Problems and Solutions . . . . . . . . . . . . . . . . . . . . . . . . . . . . . . . 179
0.9950 0.7758 0.4537 0.0000
0.5597 0.3273 0.0000
0.2361 0.0000
0.0000
We now build the American call option tree. This tree is built using the same
backward recursion procedure as for the European call, except that at each node
we compare the continuation value with the value of early exercise, which is:
Value of Early Exercise = S K + PV of expected future dividends
The tree is as follows:
12.67 21.89 36.78 59.19
3.65 7.36 14.83
0.00 0.00
0.00
The American call is worth $12.67.
There is but one node at which early exercise occurs, and this has been represented
in bold font in the above tableau. Here the continuation value is $36.71 (see the
European option tree), and the value of early exercise is
S K +PV (D) = 136.33 100.00 + 0.45 = 36.78 > 36.71.
Hence, we exercise early and capture the higher value.
Ultimately, this results in a higher value for the American call.
(g) Left as an exercise to the reader. The European puts value is $11.18.
(h) Left as an exercise to the reader. The American puts value is $11.26.
Sundaram & Das: Derivatives - Problems and Solutions . . . . . . . . . . . . . . . . . . . . . . . . . . . . . . . 180
Chapter 13. Implementing the Binomial Model
1. Suppose the distribution of S
t
is given to be lognormal:
log
_
S
t
S
_
N
_
t,
2
t
_
,
where = 0.10 and = 0.20. Given that S = 60, calculate 95% condence intervals
for the price of S
t
three months from today.
Answer: Since = 0.20, we have
2
= 0.04. Given that time is measured in years, we
have t = 1/4 = 0.25. Therefore, t = (0.10)(0.25) = 0.025, and
2
t = (0.04)(0.25) =
0.01. Using these values, we now have
ln
_
S
t
S
_
N(0.025, 0.01).
Note that the standard deviation of ln(S
t
/S) is given by

0.01 = 0.1. Since the 95%


condence interval for a normal distribution is given by
(mean 1.96 standard deviations, mean + 1.96 standard deviations),
it is the case that, with probability 0.95, ln(S
t
/S) will lie between
0.025 2

0.01 and 0.025 + 2

0.01,
that is, between
0.175 and 0.225.
Taking exponents on both sides, we see that with probability 0.95 that S
t
/S will lie
between
exp{0.171} = 0.84 and exp{0.221} = 1.25.
Since S is given to be 60, this means that with probability 0.95, S
t
will lie between
S 0.84 = 50.4 and S 1.25 = 75.
Sundaram & Das: Derivatives - Problems and Solutions . . . . . . . . . . . . . . . . . . . . . . . . . . . . . . . 181
2. Repeat Question 1 with = 0.10 and = 0.10.
Answer: The required condence interval in this case is the interval from 55.78 to 67.85
(i.e., with probability 0.95, S
t
will lie between 55.78 and 67.85). Since the procedure
by which this answer is obtained is similar to that employed in solving Question 1, the
details are omitted.
3. Suppose you wish to approximate the distribution of a stock price three months from
now using a binomial tree with 100 steps. Suppose also that the stock price distribution
is given to be lognormal with = 0.04 and = 0.40. What values would you use for
the parameters of the binomial model?
Answer: As explained in the chapter, one solution (the CRR solution) is to use
u = exp
_

_
T
n
_
, d =
1
u
, and p =
1
2
+
1
2

_
T
n
,
where and are as specied in the lognormal distribution, T is the horizon we are
interested in, and n is the number of steps we wish to use in the binomial tree. In the
current problem, we are given = 0.04 and = 0.40; n = 100, and T = three months
= 1/4 years. Therefore, we have
u = exp{(0.40)
_
(1/400)} = exp{0.02} = 1.0202
d =
1
u
= 0.9802
p =
1
2
+
_
1
2
__
0.04
0.40
_
_
1
400
= 0.5025.
4. Repeat Question 3 but with = 0.20 and = 0.20.
Answer: Using the same methodology as in solving Question 3, it can be seen that one
solution is to use u = 1.01, d = 0.99, and q = 0.525. The details are omitted.
5. Suppose the price S
t
of a stock follows a lognormal distribution with = 0.07 and
= 0.30. What are the expected simple returns on the stock over a three-month
horizon?
Sundaram & Das: Derivatives - Problems and Solutions . . . . . . . . . . . . . . . . . . . . . . . . . . . . . . . 182
Answer: Let T denote the terminal date, so T t = 3 months is the horizon. The
expected price of the stock after three months is given by the following expression:
E(S
T
) = S
t
exp[(T t) +
1
2

2
(T t)]
Hence, the expected simple return over the period is
E
_
S
T
S
t
_
= exp[(T t) +
1
2

2
(T t)]
= exp[0.07(3/12) + 0.5(0.3
2
)(3/12)]
= exp(0.02875) = 1.0292
i.e., 2.92%.
6. Assume the S&P 500 index follows a lognormal distribution with a volatility of 25%.
Suppose the expected simple returns on the index over a one-year horizon is 8%. What
is the value of the annual expected log-return?
Answer: We have that, given T = 1,
E
_
S
T
S
t
_
= exp[ +
1
2

2
] = exp[ + 0.5(0.25
2
)] = 1.08
Solving, we have
= ln(1.08) 0.5(0.25
2
) = 0.0457
The expected log return is much lower than the expected simple return.
7. If the continuously compounded return on a stock is normal, then why is the stock price
distribution lognormal?
Answer: When the return r for any given interval of time is normal, then the nal price
of the stock over an interval is S = S
0
e
r
, where S
0
is the initial price of the stock which
is a constant. By denition, if a random variable x is normal, then the variable y = e
x
is lognormal since lny = x. Hence, the stock price will be lognormal.
Sundaram & Das: Derivatives - Problems and Solutions . . . . . . . . . . . . . . . . . . . . . . . . . . . . . . . 183
8. If the continuously compounded return on a stock worth $1 currently for a preset interval
of time is distributed normally as follows: r N(,
2
) N(0.1, 0.2
2
), then what are
(a) the expected price of the stock after one interval and (b) the variance of the stock
price?
Answer: The expected price of the stock is
E(S) = e
+0.5
2
= e
0.1+0.5(0.04)
= 1.1275.
The variance of the stock price is
Var(S) = (e

2
1)e
2+
2
= (e
0.04
1)e
2(0.1)+0.04
= 0.051881.
Both answers are obtained by application of formulae for the moments of the lognormal
distribution which are provided in the chapter.
9. (Technical) Suppose that the continuously compounded returns in each period are nor-
mal, but are not always independent from period to period. Will the nal stock price
after all periods be lognormal?
Answer: We note that the return over many periods will be the sum of returns over the
individual periods. If the returns in each period are normal, then the sum of returns will
also be normal, even if the returns are correlated with each other. If the total return
over all periods is normal, then the nal stock price will also be lognormal, whether or
not the returns each period are correlated.
10. (Technical) In binomial tree models, as we shrink the time interval on the tree, will the
nal return distribution of the stock price always converge to the normal distribution?
Answer: In general, the Central Limit Theorem, one of the most important results
in statistics, says that the limiting distribution will indeed be normal. But there are
exceptional cases where binomial distributions may not converge to a normal. One is
where the Poisson Limit Theorem applies: letting n denote the number of steps and p
the probability of an up move in each step, Poisson Limit Theorem says that if n
and p 0 with np = , then the binomial probability converges to a Poisson limit. Note
that part of the condition is that p 0, so the theorem applies only if the distribution
is very skewed.
Sundaram & Das: Derivatives - Problems and Solutions . . . . . . . . . . . . . . . . . . . . . . . . . . . . . . . 184
11. For a binomial tree with the probability of 0.01 of an outcome of 1 and a probability
of 0.99 of an outcome of 0, what are the mean and variance of the payo? Intuitively,
what do you learn from this analysis?
Answer: The mean value of the payo is
0.01(1) + 0.99(0) = 0.01
The variance of the payo is
(1 0.01)
2
(0.01) + (0 0.01)
2
(0.99) = 0.99
2
(0.01) + 0.01
2
(0.99)
= (0.99)(0.01)[0.99 + 0.01]
= 0.0099 0.01
The mean and variance are equal (approximately) which is a characteristic of the Poisson
distribution. We have just shown in a non-rigorous manner that the skewed probability
function leads to a Poisson limit.
12. For a binomial tree with equity returns continuously compounded with = 0.2 and
interest rates quarterly compounded at annual rate r = 0.03, what is the upshift in
stock price, downshift, and the risk-neutral probability of the upshift if the interval on
the tree is quarterly?
Answer: We use the CRR solution. The up shift is
u = exp(0.2

0.25) = 1.1052.
The down shift will be d = 1/u = 0.90484.
The compounded value of $1 in three months will be R = 1 + 0.03/4. The probability
of an up shift will be:
q =
R d
u d
=
1 + 0.03/4 0.90484
1.1052 0.90484
= 0.51238
13. Suppose the annualized volatility of a stock is = 0.30. The mean return is = 0.10.
The risk-free rate is constant for all maturities at 2%. Letting the time interval h
increase in monthly increments (1/12 of a year), how does the risk-neutral probability
of an up move in the stock price change when using the CRR model? Why do we see
this pattern?
Sundaram & Das: Derivatives - Problems and Solutions . . . . . . . . . . . . . . . . . . . . . . . . . . . . . . . 185
Answer: The up movement in the stock given by the CRR model is equal to
u = exp(

h) = exp(0.3

h)
and d = 1/u. The risk-neutral probability of an up-move is given by
q =
e
rh
d
u d
Since u, d are also functions of h, we can dierentiate q by h and examine the sign of
the derivative, or we may just compute it numerically.
Notice that if we multiply the denominator and numerator of q by e

h
, then we have
q =
e
rh+

h
1
e
2

h
1
We can see that q increases in r. Also, holding r constant, whether q increases or
decreases in h, depends on r. For higher values of r we see that q increases in h, and
for lower values of r, q decreases in h. The reader may plot these outcomes to verify
this result. The observed patterns are solely mechanical consequences of the formula for
q in terms of , r and h.
14. If the standard deviation of daily stock returns is 2%, what is the volatility of annual
stock returns?
Answer: If = 0.02, then the variance is
2
= 0.02
2
= 0.0004. Since it is variance, not
standard deviation that scales linearly with time, the annual variance will be the daily
variance times the number of days in a year.
If we take the number of days to be trading days, then we get annual variance to be
252
2
= 252 0.0004 = 0.1008. Hence the annual standard deviation (volatility) is

0.1008 = 0.3174.
15. What is the key assumption that supports converting weekly standard deviation into an
annual standard deviation by multiplying it by

52?
Answer: The assumption needed is that returns from period to period are independent
of each other. Under this assumption the variance over many periods is simply the
sum of variances of each period. Hence, when variance scales linearly with time, the
standard deviation will scale with the square-root of time. However, if there is (say)
serial correlation between returns from period to period, then the total variance will not
Sundaram & Das: Derivatives - Problems and Solutions . . . . . . . . . . . . . . . . . . . . . . . . . . . . . . . 186
any more be equal to the sum of period variances. In this case, the standard deviation
will also not scale with the square-root of time.
In notational terms, if we have 52 weeks of returns, r
1
, r
2
, ..., r
52
, then, if they are
independent, the variance of the returns for the year is
Var(r
1
+r
2
+... +r
52
) = Var(r
1
) + Var(r
2
) + + Var(r
52
) = 52
w
where
w
is the variance of weekly returns. But if there is correlation in the returns from
week-to-week, then
Var(
52

t=1
r
t
) =
52

t=1
Var(r
t
) +
52

t=1
52

k=1
Cov(r
t
, r
k
|t = k)
The presence of the covariance terms means that the square-root rule for scaling the
standard deviation with time will not apply.
16. You are constructing a 100-period binomial tree to represent a 91-day ( 0.2493-year)
horizon. The risk-free rate for the given horizon is 4% in annualized continuously com-
pounded terms. The underlying asset has a volatility of 38%. What are the parameters
of the binomial tree if you use the JR solution? What is the risk-neutral probability in
the constructed tree?
Answer: The interval on the binomial tree will be h = 0.2493/100 = 0.002493. In the
Jarrow-Rudd model, the up shift parameter is
u = exp[(r
f
0.5
2
)h +

h] = exp[(0.04 0.5(0.38
2
))h + 0.38

h] = 1.01907.
Likewise the down move parameter is
d = exp[(r
f
0.5
2
)h

h] = exp[(0.04 0.5(0.38
2
))h 0.38

h] = 0.981127.
The risk-neutral probability on the tree is
q =
e
r
f
h
d
u d
=
e
0.04h
0.981127
1.01907 0.981127
= 0.5
Sundaram & Das: Derivatives - Problems and Solutions . . . . . . . . . . . . . . . . . . . . . . . . . . . . . . . 187
17. Rederive the risk-neutral probability in the JR model using general algebra. Is the
probability always exactly
1
2
?
Answer: Simple algebra shows that the risk-neutral probability in the JR model is
q =
exp[

h +
1
2

2
h] 1
exp(2

h) 1
We can see that this is independent of r, the risk-free rate of interest.
In addition, this probability is very close to 0.50, but is not always equal to 0.5. There
is of course a locus of points in {, h} for which p = 0.5 exactly. The values in the
previous problem provided one such case.
18. This problem will require a spreadsheet or programming eort. The initial stock price is
given to be $100. We wish to price European calls and puts with strike price $100. The
option maturity is T = 1 year and the risk-free rate of interest is 5% per annum. If the
volatility is = 0.40, then price the call and the put using the JR model. Assume you
use a binomial tree comprising n = 30 periods.
Answer: The basic values we we need to compute for the Jarrow-Rudd implementation
are as follows:
Time interval: h = 1/30 = 0.033333.
Up move: u = e
(r
f
0.5
2
)h+

h
= 1.074687.
Down move parameter: d = 0.928644.
Interest rate per time step: R = e
rh
= 1.001668.
Risk-neutral probability: q = 0.500016.
Building a tree and pricing a European call and put gives the following values. The call
is priced at $18.05, and the put is priced at $13.17.
19. Using the same parameters as in the problem above and the same JR tree, what are the
prices of American calls and puts?
Answer: The American call has the same value as the European one since there are no
dividends, i.e., $18.05.
The price of the American put is $13.73; note that there is an early exercise premium
over and above the value of the European put.
Sundaram & Das: Derivatives - Problems and Solutions . . . . . . . . . . . . . . . . . . . . . . . . . . . . . . . 188
20. In Problem 18, check that your solution satises put-call parity exactly.
Answer: The call price was $18.05 and the put price was $13.17. Put-call parity is as
follows:
C P = S PV (K)
Hence
C P = 18.05 13.17 = 4.88
which is the left hand side of the parity equation.
Now we check the right hand side of the parity equation:
S PV (K) = 100 100e
rT
= 100 100e
0.05
= 4.88.
We have veried that parity holds exactly.
21. Rework Problem 18 with exactly the same parameters but use the CRR model instead.
Compare your European put and call prices with those from the JR model. Also price
the options using the Black-Scholes model and compare those prices as well.
Answer: The solutions are as follows:
Option Jarrow-Rudd CRR Black-Scholes
Call 18.055 17.894 18.023
Put 13.178 13.017 13.146
22. You are given the following parameter values and are required to price calls using both
the JR model and the CRR model for dierent values of n, the number of periods on
the tree. The given values are: S = 100, K = 100, T = 1, = 0.4, and r = 0.05. For
varying values of n, running from 5 to 100, plot the values of the call option obtained
from the two models. How dierent are the convergence rates?
Answer: Implementing the models using a program, we generate values of the call for
dierent n = 5...100. The plots are presented below.
Sundaram & Das: Derivatives - Problems and Solutions . . . . . . . . . . . . . . . . . . . . . . . . . . . . . . . 189
Call-JR
17.2
17.4
17.6
17.8
18
18.2
18.4
18.6
18.8
5
1
1
1
7
2
3
2
9
3
5
4
1
4
7
5
3
5
9
6
5
7
1
7
7
8
3
8
9
9
5
Call-JR
Call-CRR
16.5
17
17.5
18
18.5
19
5
1
2
1
9
2
6
3
3
4
0
4
7
5
4
6
1
6
8
7
5
8
2
8
9
9
6
Call-CRR
23. Repeat the previous question for the CRR model only, but change the strike to $90, i.e.,
the call option is deep in-the-money. What happens to the plot? Then repeat this with
the strike equal to $110, i.e., when the option is out-of-the-money. What happens to
the plot?
Answer: The plot is as follows: for the ITM option, strike 90, we have
Sundaram & Das: Derivatives - Problems and Solutions . . . . . . . . . . . . . . . . . . . . . . . . . . . . . . . 190
Call-CRR
22.4
22.5
22.6
22.7
22.8
22.9
23
23.1
23.2
23.3
23.4
23.5
5
1
2
1
9
2
6
3
3
4
0
4
7
5
4
6
1
6
8
7
5
8
2
8
9
9
6
Call-CRR
And, for the OTM option, strike 110, we have
Call-CRR
13.4
13.5
13.6
13.7
13.8
13.9
14
14.1
14.2
14.3
14.4
14.5
5
1
2
1
9
2
6
3
3
4
0
4
7
5
4
6
1
6
8
7
5
8
2
8
9
9
6
Call-CRR
24. (Technical) Prepare program code in Visual Basic for Applications (VBA) in Excel (i.e.,
macro programming) to price options using a recursive implementation of the binomial
tree. This exercise is meant to give you some experience with recursive programming
structure.
Answer: The program code may be written very economically when done with recursion.
The VBA code is as follows:
Function bs(s, k, t, v, r, n, dt)
u = Exp(v * Sqr(dt))
d = 1 / u
RR = Exp(r * dt)
Sundaram & Das: Derivatives - Problems and Solutions . . . . . . . . . . . . . . . . . . . . . . . . . . . . . . . 191
q = (RR - d) / (u - d)
If n = 0 Then
bs = WorksheetFunction.Max(0, s - k)
Else
bs = (q * bs(u * s, k, t, v, r, n - 1, dt)
+ (1 - q) * bs(d * s, k, t, v, r, n - 1, dt)) / RR
End If
End Function
Note that this is not the most ecient implementation of the binomial tree when the tree
is recombining because the code process the tree as if it is non-recombining, meaning
computational eort is of the order O(2
n
) (exponential in n) and not of the order O(n
2
)
(polynomial in n), where n is the number of periods on the tree.
25. Is it possible to build a recombining tree if the interest rate is not constant, nor stochastic,
but a deterministic function of time, i.e., r(t)?
Answer: Yes. The CRR tree is built using only the volatility as an input, so if
is constant, the parameters u and d will be constant too, and the tree will recombine
regardless of the behavior of interest rates. Note, however, that the risk-neutral prob-
abilities of up moves will not be constant on the tree. If R
t
= exp(r
t
h) denotes the
risk-free rate per time-step at a node, then the risk-neutral probability q
t
at that node
is given by
q
t
=
R
t
d
u d
which varies with R
t
.
We leave it to the reader to answer this question if we are instead using the Jarrow-Rudd
solution to approximate the risk-neutral distribution of the stock price, i.e., with the drift
of the stock given by the appropriate generalization of (13.11).
Sundaram & Das: Derivatives - Problems and Solutions . . . . . . . . . . . . . . . . . . . . . . . . . . . . . . . 192
Chapter 14. Black & Scholes Model
1. Explain why the Black-Scholes model is inappropriate if the stock can gap.
Answer: A market gap or jump is a discontinuous price movement. The Black-Scholes
model assumes prices follow a geometric Brownian motion (GBM) process, and one of
the requirements of GBM is that realized price paths be continuous. So market gaps
are ruled out by assumption. An extension of the Black-Scholes framework to include
jumps is presented in Chapter 16.
2. Is assuming a constant (nonstochastic) interest rate in the Black-Scholes model a major
deciency of the model?
Answer: It depends. If the maturity of the option is short, then assuming interest rates
are constant is a reasonable assumption, as rates will typically not change too much in a
short period of time, and the option value from Black-Scholes will not be much dierent
from a model that accounts for stochastic interest rates.
However, for long maturities, this may not be the case. One reason is that if interest rates
are stochastic, the eect of discounting in the model may be mis-specied if we assume
interest rates are constant.Then there is the possibility that interest rate changes and
stock price changes may be correlated. For instance, with positive correlation between
changes in interest rates and stock prices, upward moves in stock prices that push the
call deeper intothe-money and so improve call values will also be associated with higher
interest rates that too push up call values. Hence, positive correlations may enhance
call value, an eect that is ignored by assuming constant interest rates.
3. The Black-Scholes model assumes constant volatility. How serious a shortcoming is this?
Answer: Empirically, it is well known that volatility does not remain constant over time
and demonstrates substantial variation at times. We consider extensions of Black-
Scholes that account for stochastic volatility in Chapter 16. We show there that when
volatility is stochastic option prices can dier substantially from those of the Black-
Scholes model. The driving factor is that when volatility is stochastic, the terminal
distribution of the stock price return is no longer normal; rather, it tends to have fat
tails (leptokurtic) and to exhibit skewness. These features match empirical behavior of
stock prices better than the Black-Scholes model. So, as a general matter, ignoring the
stochastic nature of volatility can be a serious shortcoming. Sophisticated models used
in practice commonly incorporate stochastic volatility.
4. Compute the three-month (T = 1/4) forward price F of a stock currently trading at $40
when the risk-free rate for this period is r = 4%. Then, set the strike price K = F and
Sundaram & Das: Derivatives - Problems and Solutions . . . . . . . . . . . . . . . . . . . . . . . . . . . . . . . 193
calculate call and put values from the Black-Scholes model if the volatility is = 0.4,
assuming the stock pays no dividends. What can you say about the call and put prices
you just computed?
Answer: Since there are no dividends, the forward price is obtained from PV (F) = S,
or
F = S exp(rT) = 40.402.
Options with a strike price of F are said to be at-the-money-forward or ATMF. Using the
Black-Scholes formula, we get the prices of three-month ATMF calls and puts to both
be 3.186. The reason that call and put prices are equal is because of put-call parity:
C P = S PV (K) (Put-Call Parity)
C P = S PV (F) (since K = F)
C P = S S = 0 (PV (F) = S by the forward-pricing condition)
5. (We repeat the previous question allowing for nonzero dividends). Assume a stock has
a dividend yield of d = 2%. Compute the three-month (T = 1/4) forward price F of a
stock currently trading at $40 when the risk-free rate for this period is r = 4%. Then,
set the strike price to K = F and calculate call and put values from the Black-Scholes
model if the volatility is = 0.4. What can you say about the call and put prices you
just computed?
Answer: The forward price is
F = S exp((r d)T) = 40.201.
Using the Black-Scholes equation we get that the call price is 3.170. Then, computing
the put price we get the value to be also 3.170. The reason that call and put prices are
equal is because the strike price is the forward price, and such options are said to be
struck at-the-money-forward or ATMF. The payment of dividends does not change
the result, and in fact the reader is encouraged to use the put-call parity relationship (see
Chapter 10 for the put-call parity result with continuous dividend yields) to demonstrate
this in this case as well.
6. Plot the price of a Black-Scholes call for a range of volatility from 5% to 40%. Use the
following parameters: S = 30, K = 33, T = 1/3, r = 0.03, and d = 0. Does the
function appear concave or convex?
Answer: The plot for the option prices is as follows:
Sundaram & Das: Derivatives - Problems and Solutions . . . . . . . . . . . . . . . . . . . . . . . . . . . . . . . 194
0
0.5
1
1.5
2
2.5
3
3.5
4
4.5
5
0
.
0
5
0
.
0
8
0
.
1
1
0
.
1
4
0
.
1
7
0
.
2
0
.
2
3
0
.
2
6
0
.
2
9
0
.
3
2
0
.
3
5
0
.
3
8
0
.
4
1
Volatility
O
p
t
i
o
n

p
r
i
c
e
Call
Put
The graphs indicate that call prices appear to be convex in volatility for this range of
parameter values.
7. Plot the price of a Black-Scholes call for declining maturity from three years to zero
years. Does the function appear concave or convex? Use the following parameters:
S = 30, K = 33, = 0.3, r = 0.03, and d = 0.
Answer: The plot for the option prices is as follows:
Sundaram & Das: Derivatives - Problems and Solutions . . . . . . . . . . . . . . . . . . . . . . . . . . . . . . . 195
0
1
2
3
4
5
6
7
3
2
.
7
5
2
.
5
2
.
2
5 2
1
.
7
5
1
.
5
1
.
2
5 1
0
.
7
5
0
.
5
0
.
2
5
0
.
0
1
Maturity
O
p
t
i
o
n

v
a
l
u
e
Call
Put
The prices of calls (and puts) appear concave in maturity over the given range. As
we discuss in Chapter 17, time decay accelerates as the option maturity nears, i.e., the
option loses value faster when maturity is short than when it is long.
8. Plot the price of a Black-Scholes call for a range of interest rates from 1% to 20%. Use
the following parameters: S = 30, K = 33, T = 3, = 0.3, and d = 0.
Answer: The plot for the option prices is as follows:
Sundaram & Das: Derivatives - Problems and Solutions . . . . . . . . . . . . . . . . . . . . . . . . . . . . . . . 196
0
2
4
6
8
10
12
14
0
.
0
1
0
.
0
3
0
.
0
5
0
.
0
7
0
.
0
9
0
.
1
1
0
.
1
3
0
.
1
5
0
.
1
7
0
.
1
9
Interest Rate
O
p
t
i
o
n

v
a
l
u
e
Call
Put
Call prices increases with interest rates, whereas put prices decrease. We discussed the
intuition for this in Chapter 10. Chapter 17 on the option greeks discusses the behavior
in more detail.
9. On December 1, the S&P 500 index (SPX) is trading at 1396.71. The prices of call
options on the index expiring on March 16 (i.e., in a bit over three months) are as
follows:
Strike K Call Prices
1300 116.80
1350 73.70
1400 41.00
Assuming the interest rate for that period is 4.88% and the annual dividend rate on
the SPX is 1.5%, compute the implied volatility for each of the SPX options using the
Black-Scholes formula. Are these volatilities the same? Explain.
Answer: Using the above inputs, we get the following implied volatilities:
Sundaram & Das: Derivatives - Problems and Solutions . . . . . . . . . . . . . . . . . . . . . . . . . . . . . . . 197
Strike K Call Implied Volatility (%)
1300 14.43
1350 12.43
1400 11.95
We can see that the implied volatilities are dierent for each strike. This is the well
known volatility skew: options with low strikes tend to exhibit greater implied volatility
than at-the-money options. Possible reasons for the skew were discussed in Section 14.8.
10. (Repeat for puts) On December 1, the S&P 500 index (SPX) is trading at 1396.71. The
prices of put options on the index expiring on March 16 (i.e., a little over three months)
are as follows:
Strike K Put Prices
1300 11.20
1350 17.30
1400 30.50
Assuming the interest rate for that period is 4.88%, and the annual dividend rate on
the SPX is 1.5%, compute the implied volatility for each of the options using the Black-
Scholes formula. Are these volatilities the same? Explain. Also, are these volatilities the
same as that obtained from the previous question? Should they be? Explain.
Answer: Using the above inputs, we get the following implied volatilities:
Strike K Call Implied Volatility (%)
1300 16.32
1350 13.78
1400 11.90
We can see that the implied volatilities are dierent for each strike. Again, this is just
the volatility skew that implied volatilities for puts with low strikes (corresponding to
out-of-the-money puts) are higher than those at-the-money.
Note that put-call parity implies that European calls and puts with the same strike and
maturity should have the same implied volatilities, but even though the SPX options
are European, the implied volatilities for puts obtained here diers from those for calls
obtained in the previous question. The dierence is likely driven by liquidity factors that
are ignored in the derivation of put-call parity and that make arbitrage strategies dicult
to implement.
Sundaram & Das: Derivatives - Problems and Solutions . . . . . . . . . . . . . . . . . . . . . . . . . . . . . . . 198
11. Show that the delta of an at-the-money European call option in the Black-Scholes model
is at least 1/2. What about the delta of an at-the-money put?
Answer: The delta of a call in the BlackScholes model is given by N(d
1
), where
d
1
=
1

T t
_
ln
_
S
K
_
+
_
r +
1
2

2
_
(T t)
_
, (2)
and the remaining notation is as usual. If the call is at-the-money, then we must have
S = K. This implies (S/K) = 1, so ln(S/K) = 0. Therefore,
d
1
=
1

T t
_
r +
1
2

2
_
(T t) =
1

_
r +
1
2

2
_

T t. (3)
Since each term on the right-hand side of this expression is positive, we must have
d
1
> 0. Recall that N(0) = 1/2 for a standard normal distribution. Therefore, d
1
> 0
implies N(d
1
) > 1/2.
The delta of a European put option in the BlackScholes world is N(d
1
). Since
N(d
1
) > 0.50 (as we showed above), and N(d
1
) + N(d
1
) = 1, we must have
N(d
1
) < 0.50. Therefore, N(d
1
) > 0.50, so the delta of the put lies between 0
and 1/2.
12. What happens to the delta of an at-the-money call as the time-to-maturity declines?
What about a put?
Answer: As noted in the answer to question 11, the delta of an at-the-money call
is given by N(d
1
) with d
1
specied by (3). The right-hand side of (3) decreases as
(T t) decreases, so d
1
decreases as maturity decreases. When d
1
decreases, N(d
1
)
also decreases. This means the delta of the call decreases as maturity approaches.
For a put, the delta is N(d
1
). As d
1
falls, so does N(d
1
). Thus, the delta of the
put also declines (i.e., it moves away from zero).
13. Let S = K = 100, = 0.25, and T t = 1 month. Create a spreadsheet to value a
call and a put for the following values of r:
(a) r = 0.08.
(b) r = 0.06.
(c) r = 0.04.
Answer: The answers we obtain are the following:
Sundaram & Das: Derivatives - Problems and Solutions . . . . . . . . . . . . . . . . . . . . . . . . . . . . . . . 199
(a) With S = K = 100, = 0.25, T t = 1 month, and r = 0.08:
d
1
= 0.128 d
2
= 0.056
C = 3.213 P = 2.549
(b) With S = K = 100, = 0.25, T t = 1 month, and r = 0.06:
d
1
= 0.105 d
2
= 0.033
C = 3.128 P = 2.629
(c) With S = K = 100, = 0.25, T t = 1 month, and r = 0.04:
d
1
= 0.082 d
2
= 0.010
C = 3.043 P = 2.710
14. Microsoft stock is currently trading at $24.35. Consider call and put options with a
strike of $25.00 expiring in 12 days (=0.0476 years). Suppose that the volatility of
Microsoft stock is 40% and that the interest rate is 3%. What are the Black-Scholes
prices of the call and the put? What are the option deltas?
Answer: Using S = 24.35, K = 25, r = 0.03, T = 0.0476, and = 0.40 in the Black-
Scholes formulae, we get C = 0.5859 and P = 1.200. The call delta is N(d
1
) = 0.404
and the put delta is 0.596.
15. GE stock is currently trading at $26.15. A call option with a strike of $25.00 and 12
days (= 0.0476 years) to expiry costs $1.56. Assuming an interest rate of 3%, what is
the implied volatility?
Answer: We are given S = 26.15, K = 25, T = 0.0476, r = 0.03 and C = 1.56.
The question is for what value of will the Black-Scholes formula yield the given call
price. Trial and error (or use of the Solver on Excel) shows that the implied volatility
is 0.376 or 37.6%.
16. The S&P 500 index is currently at 1101. A call option with a strike of 1075 and 17 days
(= 0.067 years) to maturity costs 36.20. Assume an interest rate of 3%. For simplicity,
assume also that the dividend yield on the index is zero.
(a) What is the implied volatility?
(b) If implied volatility went up to 20%, what would happen to the calls value?
Sundaram & Das: Derivatives - Problems and Solutions . . . . . . . . . . . . . . . . . . . . . . . . . . . . . . . 200
(c) If the other parameters remained the same, what would the option value be after
one week (i.e., with 12 trading days or 0.0476 years left to maturity)?
(d) Finally, how would your answer to part (a) change if the dividend yield were taken
to be 2% instead of zero?
Answer: We are given: S = 1101, K = 1075, T = 0.0685, and r = 0.03.
(a) The question is for what value of will the Black-Scholes call price equal 36.20.
Trial and error (or use of the Solver on Excel) shows that the required volatility is
about 16.75%.
(b) If, other things being equal, volatility went up to 20%, the call value would increase
to 39.35 or by about 8.70%.
(c) Suppose all the parameters were as given (S = 1101, K = 1075, r = 0.03, and
= 0.1675) and time-to-maturity declined by one week. Then, the option value
would fall to 33.28 or by about 7.7%.
(d) If we go back to the initial parameters but with a dividend yield of 2%, the implied
volatility of the call works out to around 17.9%.
17. The spot USD-EUR exchange rate is USD1.24/EUR. Consider a one-month (=0.083
years) put option on the EUR with a strike of USD1.25/EUR. Assume that the volatility
of the exchange rate is 12%, the one-month interest rate on the USD is 3.1%, and the
one-month interest rate on the EUR is 3.7%, both in continuously-compounded terms.
(a) What is the Black-Scholes price of the put?
(b) If you had written this put on EUR 10 million, what would you do to delta-hedge
your position?
Answer: We are given: S
0
= 1.24, K = 1.25, r = 0.031, r
f
= 0.037, T = 0.083, and
= 0.12.
(a) Substituting these values into the Black-Scholes formula, the put value works out
to 0.023.
(b) From the Black-Scholes formula, the delta of the put is 0.59, so if you write this
put on EUR 10 million, to delta hedge yourself you will have to short EUR (0.59
10) million, or EUR 5.9 million.
18. The spot USD-EUR exchange rate is USD1.50/EUR. Consider a six-month (= 0.5 years)
call option on the EUR with a strike of USD1.50/EUR. Suppose the volatility of the
exchange rate is 20%, the six-month interest rate on the USD is 1.5%, and the six-month
interest rate on the EUR is 2.5%, both in continuously-compounded terms.
Sundaram & Das: Derivatives - Problems and Solutions . . . . . . . . . . . . . . . . . . . . . . . . . . . . . . . 201
(a) What is the Black-Scholes price of the call?
(b) If you had written this call on EUR 100 million, what would you do to delta-hedge
your position?
Answer: We are given: S
0
= 1.5, K = 1.5, r = 0.015, r
f
= 0.025, T = 0.5, and
= 0.20.
(a) Substituting these values into the Black-Scholes formula, the call value works out
to 0.08.
(b) From the Black-Scholes formula, the delta of the call is 0.5077, so if you write
this call on EUR 100 million, to delta hedge yourself you will have to go long
EUR (0.5077 100) million, or EUR 50.77 million.
19. The spot USD-EUR exchange rate is USD1.50/EUR. Price a one-month straddle with
an at-the-money-forward (ATMF) strike. The ATMF strike price is dened to be that
value of K which equals the forward exchange rate for that maturity, i.e., for which
Ke
rT
= Se
qT
. Assume that the volatility of the exchange rate is 20%, the six-month
interest rate on the USD is 1.5%, and the six-month interest rate on the EUR is 2.5%,
both in continuously-compounded terms.
Answer: The ATMF strike is K = Se
(rq)T
= 1.49875. Using this and the remaining
values in the Black-Scholes formula, the call and put prices are each seen to 0.0345.
Hence the straddle costs 0.0690.
20. An option is said to be at-the-money-forward (ATMF) if the strike price equals the
forward price on the stock for that maturity. Assume there are no dividends, so the
ATMF strike K satises S
t
= PV (K) = e
r(Tt)
K. Show that the value of an ATMF
call in the Black-Scholes world is given by
S
t
[2 N(

d
1
) 1] (4)
where

d
1
= [

T t]/2.
Answer: The Black-Scholes formula for the price of a call in general is
C = S
t
N(d
1
) e
r(Tt)
K N(d
2
). (5)
where the parameters are as dened as usual, d
1
is described in expression (2) and
d
2
= d
1

T t. We will show that when the call is at-the-money-forward (i.e.,


when S
t
= e
r(Tt)
K), this expression specializes to
C = S
t
[2 N(

d
1
) 1] (6)
Sundaram & Das: Derivatives - Problems and Solutions . . . . . . . . . . . . . . . . . . . . . . . . . . . . . . . 202
where

d
1
=
1
2

T t. (7)
First, note that if S
t
= e
r(Tt)
K, then
S
t
K
= e
r(Tt)
= ln
_
S
t
K
_
= r(T t).
Substituting this in the denition of d
1
(expression (2) above), we have
d
1
=
1

T t
_
r(T t) +
_
r +
1
2

2
_
(T t)
_
=
1
2

T t, (8)
so d
1
=

d
1
, where

d
1
is given by (7). Moreover,
d
2
= d
1

T t =
1
2

T t. (9)
Thus, d
2
=

d
1
, so N(d
2
) = N(

d
1
) = 1 N(

d
1
). Substituting all this in the call
pricing formula (5), we get, as required.
C = S
t
N(

d
1
) e
r(Tt)
K [1 N(

d
1
)]
= S
t
[N(

d
1
) [1 N(

d
1
)]] (since S
t
= e
r(Tt)K
)
= S
t
[2N(

d
1
) 1].
21. Show that the at-the-money-forward call price (4) is approximately equal to
S
t
1

T t (10)
Remark: Expression (10) gives us a quick method for calculating the prices of ATMF
calls. Two interesting points about expression (10):
Sundaram & Das: Derivatives - Problems and Solutions . . . . . . . . . . . . . . . . . . . . . . . . . . . . . . . 203
(a) It depends on only three parameters (S
t
, , and T t) and the constant ; in
particular, the cumulative normal distribution function N() is not involved.
(b) It shows that the price of at-the-money-forward calls are approximately linear in .
These features make the formula above very easy to use in practice not only to obtain
prices of ATMF options, but also to obtain quick estimates of implied volatility of such
options. The next two questions illustrate these points.
Answer: The proof uses the idea of a Taylor-series expansion for N(

d
1
):
N(

d
1
) N(0) +N

(0) (

d
1
0)
Now, N(0) = 1/2 and N

(0) = 1/[

2], so
N(

d
1
)
1
2
+
1

d
1
.
Therefore,
2N(

d
1
) 1
1

2
2

d
1
=
1

T t.
So the price of an at-the-money-forward option is approximately given by
C = S
t
[2 N(

d
1
) 1] S
t
1

T t.
22. Using (10), identify the approximate price of an at-the-money forward call with the
following parameters:
(a) S = 50, T t = 1 month, and = 0.15.
(b) S = 70, T t = 2 month, and = 0.25.
Answer: We have
C S
t
1

T t.
(a) In this case,
C (50)
_
1

2
_
(0.15)
_
1/12 = 0.864.
Sundaram & Das: Derivatives - Problems and Solutions . . . . . . . . . . . . . . . . . . . . . . . . . . . . . . . 204
(b) Now:
C (70)
_
1

2
_
(0.25)
_
2/12 = 2.850.
23. Suppose an at-the-money forward call with one month to maturity is trading at a price
of C = 0.946 when the stock price is S
t
= 54.77.
(a) Using the approximation (10), what is the implied volatility on the call?
(b) What if the call were trading at C = 1.576 instead?
Answer: The price of an at-the-money forward call is, as we have just seen, approxi-
mately given by
C S
t
1

T t.
Given values for C, S
t
, and T t, therefore, the implied volatility of the at-the-money
forward call is approximately equal to
1
S
t

T t

2 C.
We are given that the call is at-the-money forward with C = 0.946, S
t
= 54.77, and
T t = 1 month
(a) If C = 0.946, the approximate implied volatility is
1
(54.77)
_
1/12

2 (0.946) = 0.15.
(b) If C = 1.576, the approximate implied volatility is given by
1
(54.77)
_
1/12

2 (1.576) = 0.25.
24. A stock index is currently at 858. A call option with a strike of 850 and 17 days (=
0.047 years) to maturity costs 23.50. Assume an interest rate of 3%. For simplicity,
assume also that the dividend yield on the index is zero.
(a) What is the implied volatility?
(b) If implied volatility went up to 28%, what would happen to the calls value?
Sundaram & Das: Derivatives - Problems and Solutions . . . . . . . . . . . . . . . . . . . . . . . . . . . . . . . 205
(c) If all the other parameters remained the same, what would the option value be
after one week (i.e., with 10 days or 0.027 years left to maturity)?
Answer: Using Solver, the answer to part (a) is = 0.25121. The answer to part (b) is
obtained by raising volatility to 0.28, and the call price is then 25.59. Finally, the answer
to part (c) is a call price of 18.83, given a volatility of 0.25121.
Sundaram & Das: Derivatives - Problems and Solutions . . . . . . . . . . . . . . . . . . . . . . . . . . . . . . . 206
Chapter 15. The Mathematics behind Black-Scholes
1. If x
t
= at + bW
t
where W
t
is a Wiener process and W
0
= 0, then write down the
equation in dierential form.
Answer: This is simply a matter of restating the equation in dierentials of all the
variables that are not constant together with the initial condition:
dx
t
= a dt +b dW
t
, x
0
= 0.
2. If x
t
= at + bW
t
, and y = e
x
(time subscripts suppressed), what is the dierential
process for y?
Answer: Let f(x, t) = e
x
. Then, y = f(x, t) and we have
f
x
(x, t) = e
x
= f(x, t), f
xx
(x, t) = e
x
= f(x, t), and f
t
(x, t) = 0
where, as usual, f
x
(x, t) is the partial derivative of f(x, t) with respect to x, etc. Using
these partial derivatives in Itos lemma, we have
dy
t
= f
x
(x, t)dx
t
+f
t
(x, t)dt +
1
2
f
xx
(x, t)b
2
dt
= f(x, t)[a dt +b dW
t
] + 0 dt +
1
2
f(x, t)b
2
dt
=
_
a +
1
2
b
2
_
y
t
dt +by
t
dW
t
Note that y follows a geometric Brownian motion. In particular, since x is normally
distributed, y is lognormally distributed.
3. In the previous question, what is the expected value of y at time t?
Answer: By denition, we have
y
t
= e
xt
Since the process (x

) is an arithmetic Brownian motion, we have


x
t
N
_
at, b
2
t
_
Sundaram & Das: Derivatives - Problems and Solutions . . . . . . . . . . . . . . . . . . . . . . . . . . . . . . . 207
As mentioned in Chapter 13, if we have a random variable X N(,
2
), then
E
_
e
X

= e
+
1
2

2
so we have
E[y
t
] = e
at+
1
2
b
2
t
4. If dr
t
= k( r
t
) dt + dW and P(r
t
, t) is a given function, then what is dP? (The
parameters k, , and are all positive constants.) Use Itos lemma.
Answer: Suppressing the arguments of P (i.e., writing P for P(r
t
, t)) and applying Itos
lemma, we obtain the following expression for dP.
dP =
P
r
dr
t
+
1
2

2
P
r
2

2
dt +
P
t
dt
=
_
P
r
k( r
t
) +
1
2

2
P
r
2

2
+
P
t
_
dt +
P
r
dW
t
5. If dx = a dt +b dW and y = ln(x), nd dy.
Answer: Again, its just a direct application of Itos lemma.
dy =
y
x
dx +
1
2

2
y
x
2
b
2
dt
=
1
x
dx +
1
2
(1/x
2
)b
2
dt
=
1
x
(a dt +b dW)
b
2
2x
2
dt
=
_
a
x

b
2
2x
2
_
+
b
x
dW
=
_
ae
y

b
2
2
e
2y
_
+be
y
dW
Sundaram & Das: Derivatives - Problems and Solutions . . . . . . . . . . . . . . . . . . . . . . . . . . . . . . . 208
6. Show that, in the Black-Scholes model, stock prices are lognormal.
Answer: We begin by noting that the underlying stochastic process in the Black-Scholes
model is a geometric Brownian motion of the following form:
dS
t
= S
t
dt +S
t
dW
t
It is easily checked (we have done this in Question 2 of this chapter already) that this
dierential form is obtained when S follows the process
S
t
= S
0
exp
_
(
1
2

2
)t +W
t
_
So
ln S
t
= ln S
0
+
_

1
2

2
_
t + W
t
The right-hand side is an arithmetic Brownian motion process, so for given t is distributed
normally with mean [ln S
0
+ (
2
/2)t] and variance
2
t. Thus, S
t
is lognormal.
7. (Requires Numerical Analysis)
(a) Write down the probability density function of the risk-neutral terminal distribution
of returns for stocks in the Black-Scholes model.
(b) Then write down the expression for the value of a call option on a stock in integral
(expectation) form under the risk-neutral probability measure.
(c) For the following parameter values, undertake the integration using Octave and
price the call option: S = 100, strike K = 102, volatility = 0.3, risk-free rate
r = 0.02, and maturity T = 0.5.
Assume there are no dividends.
Answer:
(a) The Black-Scholes model uses a geometric Brownian motion for the stock price.
In the risk-neutral world, the drift of this process is the risk-free rate:
dS
t
= rS
t
dt +S
t
dW
t
As we have seen in Questions 2 and 6, this implies the following lognormal time-T
distribution for the stock price:
S
T
= S
0
exp
__
r
1
2

2
_
T +

T
_
Sundaram & Das: Derivatives - Problems and Solutions . . . . . . . . . . . . . . . . . . . . . . . . . . . . . . . 209
where N(0, 1). Let R denote the log-returns over this period. Then
R = ln
_
S
T
S
0
_
=
_
r
1
2

2
_
T +

T
so
R N
__
r
1
2

2
_
T,
2
T
_
N(,
2
)
where
=
_
r
1
2

2
_
T,
2
=
2
T.
The probability density of R is
(R) =
1

2
exp
_

(R )
2
2
2
_
(b) Tthe stock price at maturity given a return R is
S
T
= S
0
e
R
The value of the call is the discounted risk-neutral expectation of its payos at T.
Since these payos are max{S
T
K, 0}, we obtain
Call price = e
rT
_
+

max
_
0, S
0
e
R
K
_
(R) dR
Computing this expression gives the call price. It is this expectation that is com-
puted in closed-form in Section 15.4 when we derive the Black-Scholes formula via
risk-neutral pricing.
(c) The following program code in Octave computes the expectation for the given
parameter values:
>> s=100; k=102; T=0.5; v=0.3; r=0.02;
>> mu = (r-0.5*v^2)*T
mu = -0.012500
>> gamma2 = v^2*T
gamma2 = 0.045000
>> R = [-2:0.005:2]; %range from -200% to +200% return
>> phiR = normal_pdf(R,mu,gamma2); %using the normal pdf of return
>> CALL = exp(-r*T)*sum(max(0,s*exp(R)-k).*phiR*0.005)
CALL = 8.0048
Sundaram & Das: Derivatives - Problems and Solutions . . . . . . . . . . . . . . . . . . . . . . . . . . . . . . . 210
8. Consider the usual Black-Scholes setting:
dS
t
= S
t
dt +S
t
dW
t
Suppose we have = 0.20 and = 0.40. What is the expected value of the stock price
after two years if the current price is $100? What is the standard deviation of the stock
price value after two years?
Answer: The Black-Scholes model involves lognormal price distributions, so we rst
recall properties of the lognormal distribution that were stated in Chapter 13. If x
N(m, s
2
) then y = e
x
is lognormal with mean
E(y) = e
m+
1
2
s
2
and variance
Var(y) = e
2m+s
2
(e
s
2
1)
Now, as we have seen in Questions 2 and 6, the time-T distribution of stock prices in
the Black-Scholes setting is given by
S
T
= S
0
exp
__

1
2

2
_
T +W
T
_
Let
X =
_

1
2

2
_
T +W
T
Then, of course, S
T
= S
0
e
X
and
X N
__

1
2

2
_
T,
2
T
_
So applying the properties of the lognormal, we get
E[S
T
] = S
0
E[e
X
] = S
0
e
T
= 100e
0.202
= 149.18.
Sundaram & Das: Derivatives - Problems and Solutions . . . . . . . . . . . . . . . . . . . . . . . . . . . . . . . 211
That is, in expectation, the stock price will be a little over 49% higher in two years. The
variance of the stock price after 2 years an also be computed using the properties of the
lognormal:
Var(S
T
) = S
2
0
Var[e
X
]
= S
2
0
e
[2(
1
2

2
)T+
2
T]
(e

2
T
1)
= S
2
0
e
2T
(e

2
T
1)
= 100
2
e
2(0.22)
(e
0.4
2
2
1)
= 8393.1
Hence, the standard deviation is 91.614.
9. Given that stock prices follow a risk-neutral geometric Brownian motion, i.e., dS =
rS dt + S dW, write down the volatility for a put options instantaneous return.
Denote the put as a function P(S, t).
Remark The instantaneous return on the put refers to the quantity dP/P. The
volatility of this term is the coecient on the Brownian process. For example, if dS
t
=
S
t
dt +
t
S
t
dW
t
for some (
t
), the volatility of the instantaneous return on S is
t
.
Answer: We rst use Itos lemma to identify the process followed by the put:
dP =
P
S
(rS dt +S dW) +
1
2

2
P
S
2

2
S
2
dt +
P
t
dt
=
_
P
S
rS +
1
2

2
P
S
2

2
S
2
+
P
t
_
dt +
P
S
S dW
So the volatility of the instantaneous return dP/P is given by

P
S

S
P
.
Speaking intuitively, why is this called the volatility of the instantaneous return? As
the expression for the dP process shows, the uncertainty in the evolution of put values
Sundaram & Das: Derivatives - Problems and Solutions . . . . . . . . . . . . . . . . . . . . . . . . . . . . . . . 212
comes entirely from the last term which contains the stochastic term dW. Since the
variance of dW
t
is dt, we may write the variance of dP/P as follows:
Var
_
dP
P
_
=
_

P
S
S
P
_
2
dt
Expressing this per unit time (dt = 1, i.e., in annualized terms), the standard deviation
of this expression is
Std Dev
_
dP
P
_
=

P
S

S
P
10. From the solution to the preceding question, what can you say about the instantaneous
volatility of a put option when the stock price increases?
Answer: This is tricky. As we saw in Question 9, the volatility of a put option has the
following form:
Volatility
_
dP
P
_
=

P
S

S
P
When the stock price increases, there are two competing eects. One the one hand, the
put moves more out-of-the-money (or becomes less in-the-money) so P/S, which
is its delta, decreases in absolute value. On the other hand, the term S/P increases
since (a) the numerator increases and (b) the denominator falls. So what is the net
eect?
To gauge this, note rst that, as we have seen in Chapters 14 and 15 (and as we formally
derive in the appendix to Chapter 17), the delta of the put in the Black-Scholes setting
of this question is just the term N(d
1
). So we obtain
Volatility
_
dP
P
_
= N(d
1
)
S
P
Dierentiating this expression with respect to S and simplifying, we see that the volatility
of the put increases when S increases (resp. decreases) if the following term is positive
(resp. negative):
N(d
1
) + [N(d
1
)]
2
S
P

1

T
N

(d
1
)
Sundaram & Das: Derivatives - Problems and Solutions . . . . . . . . . . . . . . . . . . . . . . . . . . . . . . . 213
where, for any x,
N

(x) =
1

2
exp
_

1
2
d
2
1
_
11. Intuitively, by inspection, in relation to the previous questions, what is the formula for
the volatility of a call? Does the volatility of a call increase or decrease as the stock
price increases?
Answer: Using Itos lemma again and proceeding as we did in Question 9, we see that
the instantaneous volatility of the call is
Volatility
_
dC
C
_
=

C
S

S
C
Since C/S is the delta of the call which is N(d
1
), we can write
Volatility
_
dC
C
_
= N(d
1
)
S
C
Dierentiating the right-hand side with respect to S, we see that the volatility increases
or decreases as S increases depending on whether the following term is positive or
negative:
N(d
1
) +
1

T
N

(d
1
) [N(d
1
)]
2
S
C
12. Suppose you start with the risk-neutral stochastic dierential equation for the stock,
which is
dS = rS dt +S dW
Note here that the drift is now the risk-free rate r. Suppose you want to price a derivative
security V (S, t), which is a function of the stock price and time.
(a) Write down the process for dV using Itos lemma.
(b) Take the expectation E(dV ).
(c) Under risk neutrality, what should this expectation be equal to?
Sundaram & Das: Derivatives - Problems and Solutions . . . . . . . . . . . . . . . . . . . . . . . . . . . . . . . 214
(d) Setting E(dV ) to the correct expected value, re-arrange the equation, and explain
your result.
Answer: The reader will see that the solution to this exercise provides one of the easiest
ways to derive the Black-Scholes partial dierential equation for valuing any derivative.
(a) Using Itos lemma, we get
dV =
_
V
S
rS +
1
2

2
V
S
2

2
S
2
+
V
t
_
dt +
V
S
S dW
(b) The expectation is as follows:
E(dV ) =
_
V
S
rS +
1
2

2
V
S
2

2
S
2
+
V
t
_
dt
(c) Under risk-neutrality, the expected change in the value of any asset must be at a
rate equal to the risk-free rate of return, so we have
E(dV ) = rV dt
(d) Equating the expressions for E(dV ) in (b) and (c) above we get the so-called
fundamental partial dierential equation
rV =
V
S
rS +
1
2

2
V
S
2

2
S
2
+
V
t
Given specic boundary conditions for V , this equation may then be solved for the
security we wish to value. For example, to value a call with strike K and maturity
T, take the boundary condition to be V (S
T
, T) = max{S
T
K, 0}.
13. Suppose x N(0, 1). Let a value K be given. Dene x
+
K
by x
+
K
= x I
xK
, where
I
xK
is, as usual, the indicator function that takes on the value 1 if x K and is zero
otherwise.
(a) Compute E[x
+
K
] symbolically.
(b) What might you imagine is the use of this calculation from an option pricing
standpoint?
Answer: This expectation is a standard one that arises in call option pricing. We need
to know the truncated expectation of a random variable.
Sundaram & Das: Derivatives - Problems and Solutions . . . . . . . . . . . . . . . . . . . . . . . . . . . . . . . 215
(a) The expectation may be taken directly by integration under the normal probability
function.
_

K
x (x) dx
where
(x) =
exp[
1
2
x
2
]

2
The resultant integral is:
_

K
_
x
exp[
1
2
x
2
]

2
_
dx =
1

2
exp
_

K
2
2
_
(b) Expectations of this sort arise routinely in option pricing problems. For example,
under risk-neutral pricing, the value of a call is the value of what one receives
(i.e., the stock) upon exercise minus the value of what one pays (i.e., the strike)
upon exercise. Each of these values can be calculated separately and then used
in conjunction to identify the value of the call. (This is, in fact, what we did in
this chapter in solving for the risk-neutral value of the call.) The rst of these
expectations is exactly akin to solving for the expectation of a truncated random
variable.
14. In this chapter, we developed the following approaches to solving the option pricing
problem:
(a) The PDE approach: In this method, we found that the call option value was the
solution to the following dierential equation:
rV =
V
S
rS +
1
2

2
V
S
2

2
S
2
+
V
t
subject to V (T) = max(0, S
T
K).
(b) The risk-neutral approach: In this method, we solved for the option price by taking
the following expectation (under the risk-neutral measure):
V = e
rT
E[V (T)]
The answer to both these methods was found to be the same. Is this always true?
Answer: Yes, the two methods are equivalent. The equivalence is known as the Feynman-
Kac formula; it represents the solution to given partial dierential equations as the
expectations of functions of stochastic processes.
Sundaram & Das: Derivatives - Problems and Solutions . . . . . . . . . . . . . . . . . . . . . . . . . . . . . . . 216
15. Suppose the beta of a stock is 1.2, and the stock price is S = 40. Let the volatility be
= 0.4, the risk-free rate be r = 0.04, and assume no dividends are paid. What is the
beta of a put option with maturity one year and strike K = 40?
Answer: The equation for the beta of a put option (see Section 15.5) is

P
=
S
P
S
S
P
We can nd the price of the put and its delta using the Black-Scholes model. Carrying
out the computations, we obtain

P
=
S
P
S
S
P
= 1.2 (0.3821)
40
5.4629
= 3.35724.
Note that the absolute value of the options beta is higher than that of the stock. This
is because the option is leveraged relative to the stock.
16. Suppose the beta of a stock is 1.2, and the stock price is S = 40. Let the volatility be
= 0.4, the risk-free rate be r = 0.04, and assume no dividends are paid. What is the
beta of a call option with maturity 1 year and strike K = 40?
Answer: The equation for the beta of a call option (see Section 15.5) is

C
=
S
C
S
S
C
We can nd the price of the call and its delta using the Black-Scholes model. Carrying
out the computations, we obtain

C
=
S
C
S
S
C
= 1.2 0.6179
40
7.0313
= 4.2182.
Note that the call beta is substantially higher than that of the stock reecting the
leverage built into the call.
Sundaram & Das: Derivatives - Problems and Solutions . . . . . . . . . . . . . . . . . . . . . . . . . . . . . . . 217
17. From the previous two questions, can you derive the relationship between the betas of
call, put, and stock?
Answer: First, let us state the relationship:

C
C
P
P =
S
S
If we plug the numbers in from the previous two questions, we will conrm this result:
[4.2182 7.0313] [3.3572 5.4629] = 1.2 40 = 48
Let us now derive this relation from rst principles. Recall the put-call parity relation:
C P = S PV (K)
Taking derivatives with respect to S, we obtain the relationship between the call and
put deltas that
C
S

P
S
= 1, ()
Now take the expression for the put beta,

P
=
S
P
S
S
P
and re-arrange it to get
P
S
=

P

S
P
S
, ()
Next, take the expression for call beta, and substitute step by step the preceding expres-
sions (*) and (**):

C
=
S
C
S
S
C
=
S
_
1 +
P
S
_
S
C
=
S
_
1 +

P

S
P
S
_
S
C
which when simplied gives precisely the above-stated relationship:

C
C
P
P =
S
S
Sundaram & Das: Derivatives - Problems and Solutions . . . . . . . . . . . . . . . . . . . . . . . . . . . . . . . 218
18. (Cash-or-Nothing Option) What is the value of an option that pays $100 if the stock
price exceeds a prespecied strike at maturity? Assume that the initial stock price is
$100, maturity is one year, volatility is 50%, and the strike is $110. Assume also that
the risk-free rate of interest is zero.
Answer: The value of this option is the present value of $100 times the risk-neutral
probability that the option ends up in the money. It is useful to recognize that the
risk-neutral probability of being in the money is nothing but the term N(d
2
) in the
Black-Scholes formula. We thus have:
V = 100 e
rT
Prob[S
T
K]
= 100 e
01
N(d
2
)
= 100 N
_
ln(S/K) + (r
2
/2)T

T
_
= 100 N
_
ln(100/110) (0.5
2
/2)
0.5
_
= 32.97439
Cash-or-nothing options are examined in greater detail in Chapter 18.
19. (Corridor Options) What is the price of an option that has a maturity of 60 days and
pays $1 for each day that the stock price lies in the range (50,60)? The current stock
price is S = 55, volatility = 0.4, interest rate r = 0.03, and dividends d = 0.
Answer: The value of a corridor option for 60 days is the sum of 60 single day range
options that pay o $1 if on each day through the life of the option. Each single range
option is valued as the dierence between a 50 strike cash-or-nothing call and a 60 strike
cash-or-nothing call. The value of a $1 notional cash-or-nothing call is given by:
Cash or Nothing Call = N(d
2
)e
rT
where d
2
=
ln(S/K)+(r
2
/2)T

T
.
The program code to implement the solution is as follows:
%Model to price a corridor option
upbarrier = 60;
dnbarrier = 50;
Sundaram & Das: Derivatives - Problems and Solutions . . . . . . . . . . . . . . . . . . . . . . . . . . . . . . . 219
S = 55;
sig = 0.7;
rf = 0.03;
d = 0;
%Note that this is just the difference of two cash or nothing options.
sumval = 0;
for i=1:60;
t = i/365;
d2 = (log(S/dnbarrier)+(rf+0.5*sig^2)*t)/(sig*sqrt(t))- sig*sqrt(t);
calldn = exp(-rf*t)*normal_cdf(d2);
d2 = (log(S/upbarrier)+(rf+0.5*sig^2)*t)/(sig*sqrt(t))- sig*sqrt(t);
callup = exp(-rf*t)*normal_cdf(d2);
sumval = sumval + calldn - callup;
end;
fprintf(Corridor = %10.6f \n,sumval);
Running the program gives the following result for the parameter values embedded in
the code:
octave:1> corridor
Corridor = 37.277493
We also reran the same problem with = 0.7, i.e., a higher volatility.
octave:2> corridor
Corridor = 24.622648
As expected the option price for the corridor falls, because at higher volatility, the stock
exits the range more often.
20. (Extension of Previous Question) Consider an option that is the same as the above
except that the option pays o $1 for each day only when the stock is outside the range
(50,60). What is the price of this option?
Answer: Here the option is simply given each day by the sum of a cash-or-nothing call
at strike 60 and a cash-or-nothing put at strike 50. The program code for this option is
as follows:
%Model to price a corridor option
upbarrier = 60;
Sundaram & Das: Derivatives - Problems and Solutions . . . . . . . . . . . . . . . . . . . . . . . . . . . . . . . 220
dnbarrier = 50;
S = 55;
sig = 0.4;
rf = 0.03;
d = 0;
%Note that this is just the difference of two cash or nothing options.
sumval = 0;
for i=1:60;
t = i/365;
d2 = (log(S/upbarrier)+(rf+0.5*sig^2)*t)/(sig*sqrt(t)) - sig*sqrt(t);
callup = exp(-rf*t)*normal_cdf(d2);
d2 = (log(S/dnbarrier)+(rf+0.5*sig^2)*t)/(sig*sqrt(t)) - sig*sqrt(t);
putdn = exp(-rf*t)*normal_cdf(-d2);
sumval = sumval + callup + putdn;
end;
fprintf(Corridor = %10.6f \n,sumval);
The solution is:
octave:4> corridor2
Corridor = 22.572345
One sees that this is a simple digital strangle option.
These options were very useful at the time of the convergence of European nations
to the euro standard. Companies hedged their exposure to the risk of a country not
maintaining its currency within the prescribed bounds of the European Monetary Union
by buying contracts that paid o if a currency stayed outside a given range. The pricing
of those options was related to the pricing demonstrated above.
21. Which is higher, the expected return on a stock or that of a call option on a stock?
Assume the CAPM model governs returns in the real world.
Answer: In a CAPM world, the instantaneous returns on the call and the stock may be
expressed in terms of their betas as
E
_
dS
t
S
t
_
= (r +
S
r
m
) dt
E
_
dC
C
_
= (r +
C
r
m
) dt
Sundaram & Das: Derivatives - Problems and Solutions . . . . . . . . . . . . . . . . . . . . . . . . . . . . . . . 221
where
S
,
C
represent the betas of the stock and the call, r is the risk-free rate, and
r
m
is the excess expected return on the market portfolio (i.e., the expected return on
the market portfolio less the risk-free rate). So clearly, the expected return on the call
is greater as long as
C
>
S
. This is an inequality that always holds since the call is
akin to a levered position in the stock. Formally, as noted in Section 15.5, we have

C
=
S
C
C
S

S
so
C
>
S
as long as
C
S
>
C
S
.
It can be shown that this inequality must always hold as a consequence of the convexity
of C in the stock price S. In the specic context of the Black-Scholes model, this is
easy to see: the term C/S is the delta of the call, which is N(d
1
), so writing in the
full expression for the call price, we obtain

S
=
SN(d
1
)
SN(d
1
) PV (K)N(d
2
)
> 1.
Sundaram & Das: Derivatives - Problems and Solutions . . . . . . . . . . . . . . . . . . . . . . . . . . . . . . . 222
Chapter 16. Options Modeling: Beyond Black-Scholes
Many of the questions in this chapter are dicult as the material of this chapter is based
on advanced topics. Some questions are more dicult than others.
1. What are the shortcomings of the geometric Brownian motion for stock prices that
underlies the Black-Scholes option pricing model? For each deciency, state what mod-
ication to the model is likely to provide an improvement.
Answer: There are many shortcomings that are addressed with modeling innovations
discussed in Chapter 16.
The Black-Scholes model assumes that stock prices evolve continuously, i.e.,
there are no market gaps. However, casual observation suggests that markets
do gap, typically in response to unexpected bad or good information. To capture
this feature, the geometric Brownian motion model may be enhanced with a jump
process for stocks as described in Section 16.2.
The Black-Scholes model assumes volatility is constant, but volatility in practice
changes frequently and sometimes sharply. One way to address this shortcoming
is to make volatility itself volatile, i.e., augment the model with a stochastic
process for volatility. Stochastic volatility models are the subject of Section 16.3.
The Black-Scholes model assumes returns are i.i.d. and volatility is constant, but
empirically, returns and volatility exhibit serial correlation especially over small time
intervals. This means, for example, that if the stock moved up in the previous few
minutes, it is more likely to move up again. Stochastic volatility models can be
used to address this issue, as can the class of GARCH models described in Section
16.4.
2. (Dicult) In a jump model for returns, what determines the skewness and kurtosis of
the statistical distribution?
Answer: Section 16.2 in the text presents formulae (see equations (16.8)-(16.9)) for
the skewness and kurtosis, respectively, of jump diusions. From these equations, one
may derive the impact of , , and on skewness and kurtosis of stock returns. The
discussion below aims to provide an intuitive explanation of the behavior.
There are three components to a jump process: the jump arrival rate, the mean size of
the jump, and the variance of the jump.
Consider rst, the arrival rate of jumps (denoted in the chapter). As this rate increases,
the number of outliers that are large in size relative to the size of movement generated
by the continuous (diusion) part of the stock movement also increases. The tails of
the distribution become fatter, which increases kurtosis. However, if is too large, and
Sundaram & Das: Derivatives - Problems and Solutions . . . . . . . . . . . . . . . . . . . . . . . . . . . . . . . 223
jumps become frequent, then the jumps are no longer outliers, as all observations are
large. In this situation, kurtosis may not increase, and instead may decline with further
increases in . Put crudely, kurtosis is high when jumps are rare, and is lower when
jumps are frequent.
Next consider the mean size of the jump (). This parameter determines the sign of
the skewness of the distribution. If the mean jump is negative we get a negative skew,
and if positive we get a positive skew. The reason is obvious. Empirical data on stock
returns supports the assertion that the mean jump size is negative.
Third, the variance of the jump size (
2
) relative to the diusion variance (
2
) injects
additional fatness in the tails. As the variance increases relative to the size of the normal
variance of the stock return, kurtosis of stock returns increases.
3. In a model of stock returns with geometric Browian motion, where the volatility is
assumed to be stochastic, what determines the type of skewness that emerges? What
determines the kurtosis?
Answer: Stochastic volatility models are discussed in detail in Section 16.3, and closed-
form expressions for the skewness and kurtosis of returns under such processes are
provided in Das and Sundaram (1999). Here, we look to provide an intuitive description
of the answers.
When the volatility of stock returns is changing it makes the tails of the distrbution
fatter. Hence kurtosis comes from the fact that volatility has volatility. To understand
why, consider the following simple thought experiment. Suppose volatility is sometimes
low and sometimes very high. When volatility is low, most of the returns are small
and hence, this gives many observations tightly clustered around the mean stock return.
When volatility switches to being very high, there is a preponderance of observations
far away from the mean, which generates many outliers relative to the observations at
the mean when volatility is low. Outliers relative to normal observations implies more
kurtosis.
What about skewness? In a stochastic volatility model, skewness depends on the cor-
relation of volatility with stock return. If the correlation is positive, then when returns
are high, volatility is high, resulting in more positive-side outliers, and thus, positive
skewness. Note also, negative moves have smaller volatility, resulting in less skew on
the negative side, again resulting in positive skewness overall. Similarly, when when
stock returns are negatively related to volatility, negative skewness results in returns.
Empirically, such negative correlation appears to be the prevalent form.
4. Suppose you want to model stock returns r as being driven purely by jumps. The jumps
arrive at rate = 0.1 per time interval. When they do arise, they are normally distributed
Sundaram & Das: Derivatives - Problems and Solutions . . . . . . . . . . . . . . . . . . . . . . . . . . . . . . . 224
with mean = 0.05 and variance
2
= 0.50
2
. What is the variance, skewness, and
kurtosis of returns?
Answer: Formulae for these quantities are provided in Das and Sundaram (1999) and
repeated in the chapter as equations (16.7)-(16.9). Applying these formulae, we obtain:
Variance(r) = [
2
+
2
] = 0.1((0.05)
2
+ 0.50
2
) = 0.02525.
Hence the standard deviation is

0.02525 = 0.1589.
Skewness(r) =
(
3
+ 3
2
)
V ar(r)
1.5
=
0.1((0.05)
3
+ 3(0.05)(0.5
2
))
0.02525
1.5
= 0.93774
Since the mean of the jump is negative, we get negative skewness.
Kurtosis(r) =
(
4
+ 6
2

2
+ 3
4
)
V ar(r)
2
=
0.1(0.05
4
+ 6(0.05
2
)(0.5
2
) + 3(0.5
4
))
0.02525
2
= 28.82
This is very high. Kurtosis of the normal distribution is only 3.
5. What is the option smile? Why does it arise from fat-tailed stock return distributions?
Answer: The option smile (or the option skew) is a plot of implied volatilities against
strikes for a given maturity. Section 14.8 describes the option smile and the role of fat
tails (or leptokurtosis) in creating the smile.
6. A skewed implied volatility smile occurs more often than a symmetric smile in equity
markets. Why? What model feature is needed to generate this skew?
Answer: A noted property of stock returns is that negative outliers tend to be bigger
than positive ones, often thought of as crash risk. Reecting this, out-of-the-money
put options will be more expensive than implied volatilities estimated from at-the-money
options would suggest. In turn, this makes implied volatilities at lower strikes (corre-
sponding to OTM puts) greater than those at higher strikes, which is precisely the
volatility skew. For more details, see Section 14.8.
How might we model this feature? There are two (conceptually) straightforward ways.
One is to add a jump component to the model with a negative mean. The other is to
Sundaram & Das: Derivatives - Problems and Solutions . . . . . . . . . . . . . . . . . . . . . . . . . . . . . . . 225
augment the model by making volatility stochastic and introducing negative correlation
between the returns and volatility processes. See Sections 16.2 and 16.3.
7. Suppose the S&P index options demonstrate a left-skewed smile. You are an options
trader and believe that the smile is steeper than it should be because the market has
overestimated the extent of crash risk. You believe that the market will correct its view
within the next month. What options trading strategy would you adopt?
Answer: If, in your view, the skew is excessively steep today, it means that the market
has overpriced deep out-of-the-money put options relative to the prices of those at-
the-money. One possible strategy to adopt then is to sell deep OTM puts and buy
at-the-money puts.
8. Suppose the index option smile is symmetric, but you expect it to steepen on both sides.
What option strategy would you adopt?
Answer: Extending the logic of the previous question, in this case, we can buy deep
OTM puts and calls (corresponding to low and high strikes, respectively), and sell ATM
calls and puts.
9. (Requires Writing Code) Using the following parameters, price call options for a range
of seven strike prices with the Merton jump model.
S = 100
K = {70, 80, 90, 100, 110, 120, 130}
T = 0.5 years
rf = 3%
sigma = 0.30
mu = -0.05
gamma = 0.50
lambda = 0.5
Now with the seven option prices (one for each strike price), nd out what the implied
volatility is in the Black-Scholes model. You will need to write program code to nd the
implied volatility.
Once you have the seven corresponding implied volatilities, plot them against the strike
prices. What shape does your options smile have?
Answer: The Merton 1976 model needs to be programmed to give option prices using
the parameters above. The following is the Octave program code:
Sundaram & Das: Derivatives - Problems and Solutions . . . . . . . . . . . . . . . . . . . . . . . . . . . . . . . 226
%Program to compute option values in Mertons 1976 jump-diffusion model
%BASIC INPUTS
%s0 %Initial stock price
%X %Strike price
%r; %risk-free rate
%sig; %Diffusion coefficient
%mu %Mean jump
%gam %Jump standard deviation
%lambda %Jump arrival frequency per year
%t %Option maturity in years
function u = merton76(s0,X,t,sig,r,lambda,mu,gam)
%INTERMEDIATE VALUES
g = exp(mu + 0.5*gam^2);
xi = lambda*(1+g);
%COMPUTE OPTION VALUE
optval = 0;
totpoiss = 0;
for k=0:20; %Assuming that k=20 is large enough for interval t
eta = sqrt(sig^2 + k*gam^2/t);
rf = r - lambda*g + k*log(1+g)/t;
c_bms = bms73(s0,X,t,eta,rf,0,1);
optval = optval + c_bms*exp(-xi*t)*(xi*t)^k/prod(1:k);
totpoiss = totpoiss + exp(-xi*t)*(xi*t)^k/prod(1:k);
end;
%printf(totpoiss = %10.4f \n,totpoiss);
u = optval/totpoiss;
Next, we also need a program to compute implied volatilities and the Octave code for
this is:
%Program to price a call option using Merton 76 model
% and then use BS 73 to back out the implied vol
function u = jump_ivol(s0,X,t,sig,r,lambda,mu,gam);
jcall = merton76(s0,X,t,sig,r,lambda,mu,gam);
diff = 9999;
ivol = sqrt(sig^2 + lambda*gam^2);
dv = 0.0001;
Sundaram & Das: Derivatives - Problems and Solutions . . . . . . . . . . . . . . . . . . . . . . . . . . . . . . . 227
while diff > 0.0001;
bspr = bms73(s0,X,t,ivol,r,0,1);
ivol = ivol + (jcall - bspr)/(bspr1 - bspr)*dv;
diff = abs(jcall - bspr);
end;
u = ivol;
Finally, we write a small program to loop over the dierent strikes and obtain the implied
volatilities:
%Program to call the jump model and determine the smile.
s0 = 100;
K = [70 80 90 100 110 120 130];
T = 0.5;
rf = 0.03;
sig = 0.30;
mu = -0.15;
gam = 0.30;
lam = 0.25;
for i=1:length(K);
X = K(i);
impvol = jump_ivol(s0,X,T,sig,rf,lam,mu,gam);
fprintf(X= %10.6f IV= %10.6f \n,X,impvol);
end;
The results are:
octave:1> jump_main
X= 70.000000 IV= 0.361915
X= 80.000000 IV= 0.343751
X= 90.000000 IV= 0.333770
X= 100.000000 IV= 0.328545
X= 110.000000 IV= 0.326096
X= 120.000000 IV= 0.325475
X= 130.000000 IV= 0.326276
Implied volatility declines monotonically as the strike price increases, but nally curls up
a little when the strike equals 130. This behavior of implied volatility is known as the
volatility smile or skew (or, sometimes, as a volatility smirk).
Sundaram & Das: Derivatives - Problems and Solutions . . . . . . . . . . . . . . . . . . . . . . . . . . . . . . . 228
10. (Requires Writing Code) Write a program to simulate monthly returns for two years
from a process where returns r are drawn from a normal distribution with mean 10%
and standard deviation
t
, which follows the risk-neutral process:

t+1
=
t
e
x
, x N(0, 1)
The initial stock price is $100 and the initial
0
= 0.15. Each month the stock price
grows as follows:
S
t+1
= S
t
e
r
t
, r
t
N(0.10,
t
)
(a) Price call options for strikes: 90,100,110 with = 0.1. Assume the interest rate is
zero.
(b) Now set = 0 and reprice the options for these strikes. Compare your results with
those in (a) and comment.
Answer: The Octave program for this problem is as follows:
%Program to simulate simple SV model and get option prices
s0 = 100;
sig0 = 0.15;
mu = 0.10;
n = 24;
m = 10000;
eta = 0.1
s = s0*ones(m,1);
sig = sig0*ones(m,1);
for i=1:n;
sig = sig.*exp(eta*randn(m,1));
r = randn(m,1).*sig + mu;
s = s.*exp(r);
end;
call_90 = mean(max(0,s-90))
call_100 = mean(max(0,s-100))
call_110 = mean(max(0,s-110))
eta = 0.0
s = s0*ones(m,1);
sig = sig0*ones(m,1);
for i=1:n;
Sundaram & Das: Derivatives - Problems and Solutions . . . . . . . . . . . . . . . . . . . . . . . . . . . . . . . 229
sig = sig.*exp(eta*randn(m,1));
r = randn(m,1).*sig + mu;
s = s.*exp(r);
end;
call_90 = mean(max(0,s-90))
call_100 = mean(max(0,s-100))
call_110 = mean(max(0,s-110))
The output (prices) are as follows:
octave:4> simple_sv_hw
eta = 0.10000
call_90 = 1548.7
call_100 = 1538.8
call_110 = 1528.9
eta = 0
call_90 = 1362.6
call_100 = 1352.6
call_110 = 1342.6
When we switch o the volatility of volatility (), option prices decline. As shown in
Section 16.3 of the chapter, if volatility is volatile, the tails of the return distribution of
the underlying become fatter. Given the asymmetry in option payos, fatter tails will
lead to higher option prices.
11. (Requires Writing Code) Write a program to implement the Derman-Kani model for n
periods. The inputs are the current stock price and a volatility surface. Your output will
be the Derman-Kani tree of stock prices.
Answer: This is left as an exercise for the reader. The program may be tested by
replicating the values from the example in the chapter.
12. For a negatively skewed stock return process, what GARCH model would you use? Why?
Answer: One model that accommodates this feature is the asymmetric GARCH model of
Glosten, Jagannathan and Runkle (GJR). See Section 16.4 of the chapter, in particular,
equation (16.25). The variance equation in this model contains an indicator variable
that takes a value of 1 when the stock return is negative, thereby injecting additional
downside variance, and resulting in greater negative skewness.
Sundaram & Das: Derivatives - Problems and Solutions . . . . . . . . . . . . . . . . . . . . . . . . . . . . . . . 230
13. (a) What happens empirically to the option smile with increasing maturity? (b) How
is the smirk typically dierent from the smile? (c) Which markets are characterized by
smiles, and which ones display smirks? (d) What is the volatility surface?
Answer:
(a) The option smile appears to atten out with maturity, and this is sometimes cited
as evidence that excess kurtosis dissipates with maturity. However, Foresi and Wu
(2005, Journal of Derivatives) use an extensive database across several countries
and provide compelling evidence that the smile does not, in fact, atten appreciably
even at very long maturities.
(b) Kurtosis in returns results in a symmetric volatility smile when skewness is zero.
The presence of non-zero skewness results in a smile that is more pronounced on
side. For example, if there is negative skewness, the smile will curl up more on the
left side of the implied volatility graph, appearing as a smirk. The dierence in a
smile and a smirk therefore, is the presence of skewness.
(c) Equity index option smiles are skewed. Currency option smiles tend to be symmet-
ric. Options on individual equities also tend to exhibit some degree of symmetry.
(d) The volatility surface is a plot of the implied volatility function obtained across
various strikes (K) and maturities (T). It is a three-dimensional plot.
14. (Requires Writing Code) Does put-call parity hold in the extended Black-Scholes models?
Explain.
Answer: Yes, put-call parity is a feature of options that is independent of the stochastic
process followed by the asset underlying the option. Recall from Chapter 10 that the
derivation of put-call parity relied only on no-arbitrage arguments. Hence, enhancing
models by adding jumps and stochastic volatility does not invalidate the put-call parity
result.
15. (Requires Writing Code) Can GARCH models develop an option smile? Simulate option
prices (puts and calls) for a maturity of a half year and an initial stock price of $50; let
the initial volatility equal 30% per annum. Choose various strike prices and parameter
values for the volatility process such that you are able to generate a left skew of implied
volatility where the implieds are generated from the Black-Scholes model after prices are
generated by the GARCH model.
Answer: This is left as an exercise to the reader. Instructors: this is best assigned for
a homework exercise so that students can get some experience programming GARCH
option-pricing models. Programming hints may be obtained from Chapter 36 on Monte
Carlo simulation, where a sample program is provided for equity option simulation with
Sundaram & Das: Derivatives - Problems and Solutions . . . . . . . . . . . . . . . . . . . . . . . . . . . . . . . 231
GARCH processes. Program code in Octave for computing implied volatility was pro-
vided previously in the answer to Question 9.
16. What is the leverage eect? How does it impact option prices? How would you account
for the leverage eect in a stochastic volatility equity option model? In a jump-diusion
model?
Answer: The leverage eect is the name given to an empirical phenomenon that
negative returns tend to be correlated with higher volatility and vice versa. The moniker
comes from a possible explanation of the phenomenon. For a levered company, when
equity prices fall, the debt-equity ratio rises, so returns to equity become riskier and
volatility rises. Conversely, when equity prices rise, the debt-equity ratio falls, making
equity safer and less volatile.
The impact of the leverage eect is that returns tend to exhibit negative skewness, so
the volatility smile is left-skewed with OTM puts having higher implied volatilities than
ATM options or OTM calls.
A leverage-eect-like reaction (i.e., a left-skewed volatility smile) can be generated in
stochastic volatility models by making the returns and volatility processes negatively
correlated. In a jump-diusion model, this may be achieved by setting the jump mean
to be negative (and suitably large).
17. (Requires Writing Code) Using Octave, write a short recursive program to implement
the Cox-Ross-Rubinstein model. Run this out six periods. Use the following parameters:
initial stock price of $100, strike price of $101, risk-free rate of 5%, volatility is 25% per
annum, and maturity of half-year.
(a) Make sure that the program provides reasonably accurate prices by checking your
results against the Black-Scholes formula (the prices will be within the ballpark of the
correct prices even though there are very few periods in the model). Use both puts and
calls in your validation. Report your results.
(b) Extend the program to allow for an extra negative jump in stock returns of 20%
per jump. This jump occurs with probability of 5% (risk-neutral). (Now you have three
branches emanating from each node.) For the calls and puts reported in the previous
question, also report the prices from the jump-enhanced model. What can you say from
your comparisons about the eect of jumps?
(c) Now extend the basic program in (a) to incorporate switching volatility. This is a
simple volatility process where the volatility can take just one of two values, i.e., 10% or
40%, with equal risk-neutral probability. Volatility is not correlated with the stock price
movement. With this addition, there will now be four branches emanating from each
Sundaram & Das: Derivatives - Problems and Solutions . . . . . . . . . . . . . . . . . . . . . . . . . . . . . . . 232
node. What can you say from your comparisons about the eect of stochastic volatility?
Start with the initial volatility of 25% and then let it switch between 10% and 40%.
(d) Run your program from the previous question with volatility at levels 20% and 30%.
Start with initial volatility of 25%. How do prices change in comparison? Explain why.
Note: Make sure that in each question, you set up the risk-neutral probabilities correctly.
You will need to calculate it dierently for each of the subparts of this problem.
Answer:
(a) We may write the CRR model in recursive form. The Octave program to do this
is as follows:
%Recursive program to price options
function u = crr_rec(s,k,t,v,rf,pc,n);
if n==0;
if pc==1; optval=max(0,s-k); end;
if pc==0; optval=max(0,k-s); end;
else
h = t/n;
u = exp(v*sqrt(h));
d = exp(-v*sqrt(h));
r = exp(rf*h);
q = (r-d)/(u-d);
optval = (q*crr_rec(s*u,k,t-h,v,rf,pc,n-1)+ ...
(1-q)*crr_rec(s*d,k,t-h,v,rf,pc,n-1))/r;
end;
u = optval;
In order to compare this program with the Black-Scholes model, we ran the fol-
lowing implementation of Black-Scholes:
%Black-Merton-Scholes 1973 model
%s: stock price
%x: exercise price
%t: maturity
%v: volatility
%r: risk-free interest rate
%q: dividend rate
%pc: flag, call=1, put=0
function u = bms73(s,x,t,v,r,q,pc);
d1 = (log(s/x)+(r-q+v^2/2)*t)/(v*sqrt(t));
d2 = d1 - v*sqrt(t);
Sundaram & Das: Derivatives - Problems and Solutions . . . . . . . . . . . . . . . . . . . . . . . . . . . . . . . 233
if pc==1;
u = s*exp(-q*t)*normal_cdf(d1) - x*exp(-r*t)*normal_cdf(d2);
else
u = -s*exp(-q*t)*normal_cdf(-d1) + x*exp(-r*t)*normal_cdf(-d2);
end;
We ran the two programs to get the following results:
octave:10> bms73(100,101,0.5,0.25,0.05,0,1)
ans = 7.7627
octave:11> bms73(100,101,0.5,0.25,0.05,0,0)
ans = 6.2690
octave:12> crr_rec(100,101,0.5,0.25,0.05,1,6)
ans = 7.6193
octave:13> crr_rec(100,101,0.5,0.25,0.05,0,6)
ans = 6.1256
We see that the two models give results that are reasonably close, especially
given the very few steps on our tree.
(b) Each period is one month. The jump comprises a down move of 20% in one month
(a time interval on the tree of h = 1/12). We now assume that the stock price can
take three possible values from the previous price S. We follow the CRR model
with a simple extension.
Up shift: uS = S exp(h), with probability q.
Down shift: dS = S exp(h) with probability (1 q)..
Jump: JS = S exp(0.20) with probability 0.05.
Under risk-neutrality, we must have the following forward pricing condition:
S exp(rh) = 0.95[quS + (1 q)dS] + 0.05JS
Simplifying we nd that
q =
e
rh
0.95d 0.05J
0.95(u d)
The modied recursive program is as follows:
%Recursive program to price options
function u = crr_rec(s,k,t,v,rf,J,pJ,pc,n);
if n==0;
if pc==1; optval=max(0,s-k); end;
if pc==0; optval=max(0,k-s); end;
else
Sundaram & Das: Derivatives - Problems and Solutions . . . . . . . . . . . . . . . . . . . . . . . . . . . . . . . 234
h = t/n;
u = exp(v*sqrt(h));
d = exp(-v*sqrt(h));
r = exp(rf*h);
q = (r-(1-pJ)*d-pJ*exp(J))/((1-pJ)*(u-d));
optval = ((1-pJ)*q*crr_rec_jump(s*u,k,t-h,v,rf,J,pJ,pc,n-1) + ...
(1-pJ)*(1-q)*crr_rec_jump(s*d,k,t-h,v,rf,J,pJ,pc,n-1) + ...
pJ*crr_rec_jump(s*exp(J),k,t-h,v,rf,J,pJ,pc,n-1))/r;
end;
u = optval;
The program returns the following values:
octave:14> crr_rec_jump(100,101,0.5,0.25,0.05,-0.2,0.05,1,6)
ans = 8.6254
octave:15> crr_rec_jump(100,101,0.5,0.25,0.05,-0.2,0.05,0,6)
ans = 7.1317
We can see that the prices of puts and calls have increased. This is because of the
extra volatility injected from the jump component.
(c) In this model, to implement stochastic volatility, we grow a tree where there are
four branches emanating from each node. Since the stock price can move up
and down, and so can the volatility, we have 2 2 = 4 possible outcomes. We
implement the recursive pricing program as follows:
%Recursive program to price options with stochastic volatility
function u = crr_rec_sv(s,k,t,v0,v1,v2,rf,pc,n);
if n==0;
if pc==1; optval=max(0,s-k); end;
if pc==0; optval=max(0,k-s); end;
else
h = t/n;
u = exp(v0*sqrt(h));
d = exp(-v0*sqrt(h));
r = exp(rf*h);
q = (r-d)/(u-d);
optval = 0.5*(q*crr_rec(s*u,k,t-h,v1,rf,pc,n-1)+ ...
(1-q)*crr_rec(s*d,k,t-h,v1,rf,pc,n-1) + ...
q*crr_rec(s*u,k,t-h,v2,rf,pc,n-1)+ ...
(1-q)*crr_rec(s*d,k,t-h,v2,rf,pc,n-1))/r;
end;
u = optval;
The results for the call and put respectively are:
Sundaram & Das: Derivatives - Problems and Solutions . . . . . . . . . . . . . . . . . . . . . . . . . . . . . . . 235
octave:3> crr_rec_sv(100,101,0.5,0.25,0.1,0.4,0.05,1,6)
ans = 8.1163
octave:4> crr_rec_sv(100,101,0.5,0.25,0.1,0.4,0.05,0,6)
ans = 6.6226
The prices are higher than that of the case (a) in which we assumed constant
volatility.
(d) Re-running the model with switching volatility of 20% and 30% gives:
octave:6> crr_rec_sv(100,101,0.5,0.25,0.2,0.3,0.05,1,6)
ans = 7.6888
octave:7> crr_rec_sv(100,101,0.5,0.25,0.2,0.3,0.05,0,6)
ans = 6.1951
Since the volatility of volatility has declined somewhat, the options are worth less
than in the previous question.
18. State at least three reasons why the Black-Scholes model has warranted extension in the
past decades. What is the impact of these extensions on the stock return distribution.
What impact does this have on the prices of calls and puts?
Answer: There are many reasons for the chosen extensions of the standard Black-Scholes
model presented in Chapter 16: (i) Continuous stock return distributions are not Gaus-
sian. (ii) Stock prices jump. (iii) The volatility of stock returns changes over the life
of the option. (iv) Terminal stock return distributions are dierent depending on the
horizon that is examined.
These features of stock dynamics imply that empirical stock returns are skewed, a feature
not accommodated in the Black-Scholes model. Also, returns evidence more kurtosis
than in the Gaussian model. The skewness and kurtosis depends on the horizon of
returns, i.e., the maturity of the option.
All extensions are aimed at nding the best modication of Black-Scholes so as to match
skewness and kurtosis in returns for most of the options horizons.
19. In the jump-diusion model, what parameter needs to be set to develop a sharp negative
smirk (asymmetric skew)? What is your answer to this question in the case of the
stochastic volatility model?
Answer: (i) In a jump-diusion model, a left-sided smirk comes from negative skewness
in the return distribution. To inject negative skewness from jumps, the mean of the
jump size should be negative.
(ii) To achieve a negatively skewed return distribution in a stochastic volatility model,
the correlation between shocks to the stock return and shocks to volatility must be
Sundaram & Das: Derivatives - Problems and Solutions . . . . . . . . . . . . . . . . . . . . . . . . . . . . . . . 236
negative. Hence, when volatility is high, larger negative moves in the stock are more
likely, thereby leading to a negatively skewed return distribution.
20. The stock price of Microsoft (MSFT) on December 22 is $25.50. The traded option
prices for calls and puts maturing on January 21, and February 18 (i.e., in one and two
months) are as follows:
Maturity (T) Strike (K) Calls Puts
January 21 27 1.65 3.15
February 18 25 2.45 1.80
27.5 1.51 3.36
30 0.83 NA
Assume the interest rate is zero.
Use this information to build a two-period implied binomial tree that ts this set of
options as best as possible. Each period will be for one calendar month.
Your tree will have an initial stock price node and two periods thereafter. Since the tree
is recombining, you will have altogether 6 nodes on your tree. At each node, show the
stock price, and the branching probability. State any assumptions you make clearly.
Answer: We begin with the rst period. Let the stock price in the up-node after one
period be S
u
(with probability q) and in the down-node S
d
. We need to solve for these
three quantities, and to do so, we have the following three equations.
Forward pricing equation: 25.5 = q S
u
+ (1 q) S
d
.
Call pricing equation: q(S
u
27) = 1.65.
Put pricing equation: (1 q)(27 S
d
) = 3.15.
Solving these three equations gives the following:
q = 0.55, S
u
= 30, S
d
= 20
We move on to the second period. Here we have to nd three stock prices: S
uu
, S
ud
, S
dd
,
and two probabilities, q
u
from the node with stock price S
u
and q
d
from the node with
stock price S
d
. To solve for these, we have ve equations, one for each of the options
maturing in February:
Strike25 call-pricing equation:
2.45 = max(0, S
uu
25) q
u
0.55
+max(0, S
ud
25)[(1 q
u
) 0.55 +q
d
0.45]
+max(0, S
dd
25)(1 q
d
) 0.45
Sundaram & Das: Derivatives - Problems and Solutions . . . . . . . . . . . . . . . . . . . . . . . . . . . . . . . 237
Strike27.5 call-pricing equation:
1.51 = max(0, S
uu
27.5) q
u
0.55
+max(0, S
ud
27.5)[(1 q
u
) 0.55 +q
d
0.45]
+max(0, S
dd
27.5)(1 q
d
) 0.45
Strike30 call-pricing equation:
0.83 = max(0, S
uu
30) q
u
0.55
+max(0, S
ud
30)[(1 q
u
) 0.55 +q
d
0.45]
+max(0, S
dd
30)(1 q
d
) 0.45
Strike25 put-pricing equation:
1.80 = max(0, 25 S
uu
) q
u
0.55
+max(0, 25 S
ud
)[(1 q
u
) 0.55 +q
d
0.45]
+max(0, 25 S
dd
)(1 q
d
) 0.45
Strike27.5 put-pricing equation:
3.36 = max(0, 27.5 S
uu
) q
u
0.55
+max(0, 27.5 S
ud
)[(1 q
u
) 0.55 +q
d
0.45]
+max(0, 27.5 S
dd
)(1 q
d
) 0.45
Solving these ve equations results in the solution:
q
u
= 0.50, q
d
= 0.50, S
uu
= 33, S
ud
= 25.5, S
dd
= 17
21. One of the early extensions to the Black-Scholes model was the constant elasticity of
variance (CEV) model for equities. The CEV model assumes the following form of
stochastic process for the stock price:
dS = S dt +S

dZ
where the parameters are dened as usual except that 0 < 1 is the CEV parameter.
(a) What parameter value for results in the Black-Scholes model?
(b) As declines, does the riskiness of the stock increase or decrease?
(c) Explain the linkage of this model to the leverage eect.
Answer:
Sundaram & Das: Derivatives - Problems and Solutions . . . . . . . . . . . . . . . . . . . . . . . . . . . . . . . 238
(a) When = 1, the process becomes a geometric Brownian motion, and this gives
the underlying model for the Black-Scholes equation.
(b) As declines, the return distribution deviates from the Gaussian. Dividing both
sides by S, we get:
dS
S
= dt +S
1
dZ
Hence the local volatility of returns is approximated by
v = S
1
Note that
dv
d
= S
1
ln(S) > 0, if S > 1
and it is usually the case that S > 1. Hence, as increases, riskiness as measured
by the second moment increases as well. So as declines, second moment risk
declines too.
(c) However, skewness in return is non-zero when < 1. As the stock price declines,
risk increases because the volatility increases when < 1, and this leads to negative
skewness in stock returns. Note that
dv
dS
= ( 1)S
2
< 0
Hence as S declines, local volatility increases. This also implies that S and v are
negatively correlated, resulting in negative skewness in the stock return distribution.
This is the reason why the CEV process may be favored in modeling stocks of
rms with higher levels of default risk because, as S declines, the leverage eect
is enhanced.
Sundaram & Das: Derivatives - Problems and Solutions . . . . . . . . . . . . . . . . . . . . . . . . . . . . . . . 239
Chapter 17. Sensitivity Analysis: Greeks
1. What is the sign of the delta of (a) a call and (b) a put?
Answer: Since a call option increases in value with the stock price, its delta (math-
ematically, the rst derivative of the call with respect to the stock price) is positive.
Similarly, the delta of a put is negative because put values decline with increases in the
stock price. See Figure 17.1 where the range of call and put deltas is evident.
2. What is the sign of the gamma of (a) calls and (b) puts?
Answer: Gamma is the second derivative of option value with respect to the stock price.
Since both calls and puts are convex in the stock price, their second derivatives will be
positive. Equivalently, since both have deltas that increase with the stock price (call
deltas towards +1, put deltas towards 0), the gammas are positive. See Section 17.4
and Figure 17.3 in the chapter for more details.
3. What is the sign of the theta for (a) calls and (b) puts?
Answer: For American options, the theta of calls and puts is negative because more
time to maturity is unambiguously good (if you dont want it, you can always exercise
early and throw away the extra time). So American option values decrease as maturity
nears.
With European options, a change in the time to maturity aects the option value through
the time value and insurance value components. For European calls (on non-dividend
paying stocks), as maturity shortens, both time value and insurance value decrease,
hence theta is negative. That is, short-dated options are worth less than longer-dated
ones. But if there are dividends, and particularly if dividends are large, it is possible
that the short-dated options could be worth more as the payout impact of the longer
maturity could overwhelm the positive time-value and insurance-vale eects.
For European puts, as maturity shortens, time value increases and insurance value de-
creases. If time value increases more than insurance value decreases, theta will instead
be positive. This typically occurs in the case of deep in-the-money puts, but generally
speaking, in most other cases, theta is negative. See Figure 17.7 in the chapter for a
graphical depiction.
Section 17.5 of the chapter provides more details.
4. What is the sign of the vega for (a) calls and (b) puts?
Answer: As volatility increases, all options increase in value. Hence, vega is always
positive. See Section 17.6 for details. In particular, notice that the expression for vega
in equation (17.32) is always non-negative.
Sundaram & Das: Derivatives - Problems and Solutions . . . . . . . . . . . . . . . . . . . . . . . . . . . . . . . 240
5. What is the sign of the rho for (a) calls and (b) puts?
Answer: For American and European calls, as interest rates rise, the value of the options
increase. Hence rho is positive. For American and European puts, as interest rates rise,
option values decline. Hence rho is negative. See Section 17.7 of the chapter for details.
Figures 17.14 and 17.15 depict the change in option value as interest rates change for
calls and puts, respectively.
6. You are given two puts on the same stock but with strikes K
1
and K
2
. If their individual
gammas are
1
and
2
, what is the gamma of the portfolio consisting of both options?
Answer: Gamma is additive. The gamma of the portfolio is the sum of the gammas of
the two options. Mathematically, this just says that the second-derivative of the sum of
two functions is the sum of their second derivatives. See Section 17.8 for details.
7. Why is the gamma of an ATM option that is about to expire large in magnitude?
Answer: If the option is ATM and expires in a very short period of time, there are two
polar outcomes: (i) in the short remaining time to maturity, the option may move to
be in ITM, in which case it will have positive delta (in fact, delta close to +1 since the
option is close to maturity), or (ii) the option may become OTM and will have delta
that is close to zero.
This means that when the option is ATM and maturity is drawing near, small changes
in the stock price result in the delta swinging from zero to +1. Since gamma measures
the sensitivity of the delta, gamma becomes large.
8. If the delta of a European call is 0.6, what is the delta of the European put for the same
strike and maturity?
Answer: Start with put-call parity.
C P = S PV (K)
which leads to

P
= 1
Hence,

P
=
C
1 = 0.6 1 = 0.4
Sundaram & Das: Derivatives - Problems and Solutions . . . . . . . . . . . . . . . . . . . . . . . . . . . . . . . 241
9. Can you derive the delta of a European call from the delta of the corresponding European
put with no knowledge of the strike price?
Answer: Yes. For any given common strike and maturity, it is true that for European
options
(call) (put) = 1.
So from knowledge of one delta, one can calculate the other one regardless of the
(common) strike and maturity.
10. Is it possible to determine the strike price of a pair of a call and a put on the same stock
with the same strike if you know the deltas of both the call and the put? Assume the
options are European. [Note: You are not given the option pricing model underlying the
option prices and deltas].
Answer: No, there is no way to identify the strike K from just knowledge of the option
deltas. In particular, put-call parity is of no use: it enables us to identify a relation
between the option deltas regardless of the strike price, so does not enable going from
the deltas to the strike price that produced them.
11. If the gamma of a European call is 0.03, what is the gamma of the corresponding
European put at the same strike and maturity?
Answer: It follows form put-call parity (see Section 17.4) that the gammas of otherwise
identical puts and calls coincide, so the European put in this question would also have
a gamma of 0.03.
12. In the Black-Scholes model, what is higher, the delta of a one-year call or that of a two-
year call on the same stock if the respective strikes are at-the-money forward? Show the
result algebraically and then discuss the intuition.
Answer: At-the-money forward means the strike price is set equal to the forward price
of the stock. Assuming for simplicity that there are no dividends, the forward price for
a given time-to-maturity is that strike K such that S = PV (K) = e
r
K. A simple
calculation shows that at this strike price, the delta of the call is N(

d
1
) where

d
1
=
1


1
2

2
=
1
2

.
The right-hand side increases with , so it follows that the delta of the ATMF call with
the longer maturity will be higher.
Sundaram & Das: Derivatives - Problems and Solutions . . . . . . . . . . . . . . . . . . . . . . . . . . . . . . . 242
13. Does your answer to the preceding question change if the call is deep out-of-the-money
(OTM)? What about if it is deep in-the-money (ITM)?
Answer: Appendix 14A oered intuitive arguments for why, in general, one would expect
the delta of a deep OTM call to increase as maturity increases, and that of a deep
ITM call to decrease as maturity increases. The appendix also provided closed-form
representations in the Black-Scholes setting for the behavior of the delta as the time
to maturity T changes. Specically, since the Black-Scholes delta of a call is given by

c
= N(d
1
), the sensitivity of the delta with respect to the time to maturity is given
by

= N

(d
1
)
d
1

Since N

(d
1
) > 0, the sensitivity of delta to maturity depends on d
1
/. Dierentiating
d
1
with respect to , we obtain

1
2
3/2
_
ln
_
S
K
__
+
1
2

_
r +
1
2

2
_
or what is the same thing,
1
2

ln
_
S
K
_
+
_
r +
1
2

2
__
.
Since the rst term 1/(2

) is always positive, the sign of d


1
/ depends on the
term in parenthesis, i.e.,

ln
_
S
K
_
+
_
r +
1
2

2
_
When S < K, it is the case that ln(S/K) < 0, so the entire expression above is always
positive. This means that d
1
, and therefore the call delta, always increase with maturity
for OTM call options.
For ITM call options, the behavior is a little more complex. Fix a value of S > K.
Then, ln(S/K) > 0. For close to zero, the term

ln
_
S
K
_
becomes negative and very large in magnitude, overwhelming the eect of the positive
term (r +
2
/2). This means that for suitably short maturities, d
1
/ will be negative,
so d
1
, and therefore the call delta, will decrease as maturity increases.
Sundaram & Das: Derivatives - Problems and Solutions . . . . . . . . . . . . . . . . . . . . . . . . . . . . . . . 243
But as becomes very large, the term

ln
_
S
K
_
goes to zero, while the second term (r +
2
/2) is strictly positive, so at long maturities,
the situation reverses with d
1
, and the call delta, increasing with .
To illustrate, we take the deep OTM case where S = 70, K = 100, = 0.30, and
r = 0.03. We compute the call delta as varies from 0.1 years to 5 years. The resulting
sequence of option deltas is (read across row-by-row as maturity increases):
0.00012 0.00544 0.02099 0.04256 0.06621 0.08992 0.11278 0.13444 0.15480
0.17389 0.19177 0.20855 0.22431 0.23916 0.25317 0.26641 0.27897 0.29090
0.30225 0.31308 0.32342 0.33332 0.34281 0.35192 0.36068 0.36911 0.37723
0.38507 0.39265 0.39998 0.40707 0.41394 0.42061 0.42708 0.43337 0.43949
0.44543 0.45123 0.45687 0.46237 0.46774 0.47298 0.47809 0.48309 0.48798
0.49276 0.49744 0.50202 0.50650 0.51090
The option delta increases monotonically in maturity.
Next, we take the deep ITM case where S = 130, K = 100, = 0.30, and r = 0.03.
We compute the call delta as varies from 0.1 years to 5 years. The resulting sequence
of option deltas is (read across row-by-row as maturity increases):
0.99778 0.98065 0.95851 0.93833 0.92126 0.90703 0.89516 0.88520 0.87678
0.86961 0.86347 0.85819 0.85362 0.84965 0.84620 0.84318 0.84055 0.83824
0.83623 0.83446 0.83292 0.83158 0.83041 0.82940 0.82854 0.82779 0.82717
0.82664 0.82621 0.82586 0.82559 0.82539 0.82525 0.82517 0.82515 0.82518
0.82525 0.82536 0.82551 0.82569 0.82591 0.82616 0.82643 0.82673 0.82706
0.82740 0.82777 0.82815 0.82856 0.82898
Here, delta declines with maturity, but then increases again at longer-dated maturities.
It is U-shaped.
14. In the Black-Scholes model, how does the delta of an ATM call change as volatility
increases?
Answer: The delta of a call is given by N(d
1
). When S = K,, we have
d
1
=
1

T t

_
r +
1
2

2
_
(T t) =
_
r

+
1
2

_

T t.
Sundaram & Das: Derivatives - Problems and Solutions . . . . . . . . . . . . . . . . . . . . . . . . . . . . . . . 244
We have
d
1

=
_

2
+
1
2
_

T t
If
2
> 2r, then d
1
/ > 0, which means d
1
and the call delta increase with , while
if
2
< 2r, then d
1
/ < 0, which means d
1
and the call delta decrease with .
15. Intuitively speaking, how does the delta of a deep OTM call change as volatility in-
creases?
Answer: See Appendix 14 A. A deep OTM call is almost worthless and hence has a low
delta. As volatility increases, the chance of the option ending up ITM increases as well,
as there is an increased chance of the option swinging back to becoming ITM. Hence
the delta of the call increases. See Appendix 14 A.
16. Intuitively speaking, how does the delta of a deep ITM call change as volatility increases?
Answer: See Appendix 14 A. A deep ITM call is worth a lot and hence has a high delta.
As volatility increases, the chance of the option ending up OTM increases. Hence the
delta of the call decreases.
17. For an at-the-money-forward (ATMF) call in the Black-Scholes model, how does
c
change when interest rates rise?
Answer: The delta of a call is given by N(d
1
) where
d
1
=
1

T t
ln
_
S
K
_
+
1

_
r +
1
2

2
_
(T t)
Taking the derivative with respect to r, we obtain
d
1
r
=
1

T t > 0
So d
1
, and the delta, increase with r. This is true for all calls, not just those that are
ATMF.
Sundaram & Das: Derivatives - Problems and Solutions . . . . . . . . . . . . . . . . . . . . . . . . . . . . . . . 245
18. Intuitively speaking, how does the delta of a call option change for constant strike K
when interest rates rise?
Answer: See the answer to the previous question or Appendix 14A. The delta increases
when interest rates increase. Intuitively, an increase in interest rates increases the risk-
neutral drift of the stock price pushing it more into-the-money.
19. Intuitively speaking, how does the delta of a put (at xed strike) change when interest
rates rise?
Answer: From put-call parity, we have (call) (put) = +1. As shown in the answer
to Question 17 or in Appendix 14A, call deltas increase as interest rates increase, so
the put deltas must also increase (i.e., become less negative and move towards zero).
Intuitively, an increase in interest rates increases the risk-neutral drift of the stock price
pushing the puts more out-of-the-money and reducing the put delta in absolute value.
20. For European options, what is the relationship of the interest rate sensitivity of the call
delta to that of the put delta?
Answer: Given, as derived before, that

P
= 1
we may dierentiate the expression with respect to the interest rate r to get

C
r


P
r
= 0
Hence call and put deltas have equal sensivity to changes in interest rates.
21. How does the gamma of an OTM option behave as we approach maturity? What about
the gamma of an ITM option? Derive the answers in the context of the Black-Scholes
model and explain the intuition.
Answer: Intuitively speaking, as the option goes deep in-the-money, its price becomes
almost linear in the stock price (slope of +1 for calls and 1 for puts), and when it goes
deep out-of-the-money, its price approaches zero and the pricing function is almost a at
straight line. As maturity approaches, this behavior gets exaggerated. For xed S and
K, an option that is OTM becomes even more OTM as maturity approaches because
there is less time now to push the option back into the money. Similarly, an option that
is ITM similarly becomes more ITM as maturity approaches. This means the gamma of
both ITM and OTM options should go to zero as maturity approaches.
Sundaram & Das: Derivatives - Problems and Solutions . . . . . . . . . . . . . . . . . . . . . . . . . . . . . . . 246
To see this formally in the context of Black-Scholes, note that the gamma of both calls
and puts is given by
=
1
S

(d
1
)
where
N

(d
1
) =
1

2
exp
_

1
2
d
2
1
_
.
and is the remaining time to maturity. As shown in Section 14.5 (and as is not dicult
to check directly), as 0, it is the case that d
1
+ for in-the-money calls and
d
1
+ for out-of-the-money calls, so for both in-the-money and out-of-the-money
options, N

(d
1
) 0. Moreover, because the pace at which e

1
2
d
2
1
0 is faster than
the pace at which the term 1/(S

) blows up, the gamma of the option goes to zero


as 0.
22. How does the vega of an OTM option behave as we approach maturity? What about
the vega of an ITM option? Derive the answers in the context of the Black-Scholes
model and explain the intuition.
Answer: We note that the vega of an option is as follows:
V
C
= V
P
=

S N

(d
1
)
where is the remaining time to maturity. As mentioned in the answer to the previous
question, N

(d
1
) 0 as the time-to-maturity goes to zero for both ITM and OTM
options, so clearly vega goes to zero as maturity approaches for both ITM and OTM
options. The intuition is simply that the insurance value of the call or put depends on
how much time volatility is given to play out. When time to maturity is very short,
volatility has much smaller impact.
23. Gamma is often thought of as representing a view on volatility while vega measures
the dollar impact on option values of a change in volatility. Do they measure the same
thing? Is it possible for gamma to increase and vega to decrease simultaneously?
Answer: The formulae for the gamma and the vega in the Black-Scholes setting are

C
=
P
=
1
S
t

T t
N

(d
1
),
Sundaram & Das: Derivatives - Problems and Solutions . . . . . . . . . . . . . . . . . . . . . . . . . . . . . . . 247
and
V
C
= V
P
=

T t S
t
N

(d
1
).
Gamma is associated with a view on volatility (particularly, jump-risk) because if you
are long the option and delta-hedged, you benet from the curvature regardless of the
direction in which the price movessee Figure 17.4. And, of course, the higher the
gamma, the more the curvature, and so the greater your benet from large price swings.
Whereas gamma measures the eect of curvature (for a given level of volatility), vega
measures the dollar price impact of a change in volatility, holding all the other parameters
constant. Hence, they do not measure the same thing. However, they are connected
because the greater the volatility, the more the eect of gamma, since curvature plays
a relatively greater role in price changes.
Is it possible for gamma to increase while vega decreases, and vice-versa? Yes. Take an
at-the-money option (S = K). As maturity approaches, the gamma of the option goes
to innity while the vega of the option goes to zero. We leave it to the reader to verify
these statements.
24. How does the theta of an OTM option behave as we approach maturity? What about
the theta of an ITM option? Derive the answers in the context of the Black-Scholes
model and explain the intuition.
Answer: Formulae for the thetas of calls and puts are given in Section 17.5. We have:

C
=
1
2

T t
S
t
N

(d
1
) re
r(Tt)
KN(d
2
).

P
=
1
2

T t
S
t
N

(d
1
) +re
r(Tt)
KN(d
2
).
Consider the call. First consider the OTM case (S
t
< K). In this case, d
1
, so
the rst term in the theta
1
2

T t
S
t
N

(d
1
)
goes to zero as T t 0. (Recall that N

(d
1
) = (1/

2) exp{
1
2
d
2
1
}.) Since
d
1
, we also have d
2
, so the second term re
r(Tt)
KN(d
2
) also goes to
zero as we approach maturity. Hence, the theta goes to zero in this case.
Sundaram & Das: Derivatives - Problems and Solutions . . . . . . . . . . . . . . . . . . . . . . . . . . . . . . . 248
Next consider the in-the-money case (S
t
> K). In this case, d
1
, d
2
+as T t 0.
So while the rst term
1
2

T t
S
t
N

(d
1
)
goes to zero, the second term re
r(Tt)
KN(d
2
) converges to rK. Hence, the theta of
the call in this case converges to rK as maturity approaches.
Finally, consider the ATM case S
t
= K. In this case, we have
d
1
=
1

_
r +
1
2

2
_

T t
d
2
=
1

_
r
1
2

2
_

T t
so d
1
, d
2
0 as T t 0. This means the rst term in the call theta blows up and

C
as maturity approaches.
The behavior of the put theta can be derived from the behavior of the call theta using
put-call parity. In particular, the theta of out-of-the-money puts (S
t
> K) converges to
0 as maturity approaches, that of in-the-money puts (S
t
< K) converges to +rK, and
that of ATM puts (S
t
= K) goes to . The details are left to the reader.
25. Suppose a stock is currently trading at 100. An at-the-money call with a maturity of
three months has the following price and greeks:
C = 5.598
= 0.565
= 0.032
= 12.385
V = 19.685
= 12.71
(a) If the stock price moves to S = 101, what is the predicted new option price (using
the delta alone)?
(b) If the stock price moves to S = 101, what is the predicted new call delta?
(c) Repeat these questions assuming the stock price moves to 98 instead.
(d) If the stock price registers a large jump increase to 120, what is the new call value
predicted by the delta alone? By the delta and gamma combined?
Sundaram & Das: Derivatives - Problems and Solutions . . . . . . . . . . . . . . . . . . . . . . . . . . . . . . . 249
(e) Go back to the original parameters. If the time-to-maturity falls by 0.01, what is
the new call value predicted by the theta?
(f) Repeat the last question if the time-to-maturity falls by 0.05.
(g) Go back to the original parameters. If the volatility increases by 1%, what is the
predicted new value of the call? What if volatility fell by 2%?
(h) Go back to the original parameters. If interest rates should rise by 50 basis points,
what is the new call value predicted by the rho?
Answer: We have the following interpretation of the various sensitivity parameters, which
we are to exploit in answering the questions:
The is 0.565 and the is 0.032. This means:
(a) For a small change of (say) $dS in the stock price, the option price will
change by dS = (0.565) dS.
(b) For a large change dS, we must employ a curvature correction. The option
price will change by [ dS] +
1
2
(dS)
2
.
For a small change of (say) $dS in the stock price, the option delta will change
by dS = (0.032) dS.
The is 12.385. Therefore, for a small change of dt in the time-to-maturity,
the value of the option will change by dt = (12.385) dt.
The vega V is 19.685. Therefore, for a small change of d in the volatility, the
value of the option will change by V d = (19.685) d.
The is 12.71. Therefore, for a small change of dr in interest rates, the value
of the option will change by dr = (12.71) dr.
(a) If S changes to 101, then dS = +1. Thus, the change in the option price predicted
by the delta is
dS = (0.565)(+1) = +0.565.
Since the original option price is 5.598, the predicted new price is
C = 5.598 + 0.565 = 6.163.
(b) Again, dS = +1. The change in the delta is given by (dS) = (0.032)(+1) =
0.032. Thus, the predicted new delta is
0.565 + 0.032 = 0.597.
(c) Now dS = 2. So the change in the call price predicted by the delta is
dS = (0.565)(2) = 1.130.
Sundaram & Das: Derivatives - Problems and Solutions . . . . . . . . . . . . . . . . . . . . . . . . . . . . . . . 250
Since the original option price is 5.598, the predicted new price is
C = 5.598 1.130 = 4.468.
Similarly, the change in the delta is now given by (dS) = (0.032)(2) = 0.064.
Thus, the predicted new delta is
0.565 0.064 = 0.501.
(d) If there is a large increase in the price, we must use a curvature correction, since
using the delta alone will lead to large errors. In this problem, dS = +20. If we
used the delta alone, we would predict a change in the call price of
dS = (0.565)(+20) = +11.30.
So the predicted new price on this basis would be
5.598 + 11.30 = 16.898.
However, if we used a gamma correction, the predicted change in the price would
be
dS +
1
2
(dS)
2
= (0.565)(+20) +
1
2
(0.032) (20)
2
= +17.70.
Thus, the predicted new price on this basis would be
5.598 + 17.70 = 23.298.
Notice the large underestimate produced by the delta.
(e) If dt = 0.01, the theta predicts call values will change by
dt = (12.385)(0.01) = 0.124
Thus, the predicted new call value is
5.598 0.124 = 5.474.
(f) If dt = 0.05, the theta predicts call values will change by
dt = (12.385)(0.05) = 0.619
Thus, the predicted new call value is
5.598 0.619 = 4.979.
(g) If d = +0.01, the vega predicts call values will change by
V d = (19.685)(+0.01) = +0.197
Thus, the predicted new call value is
5.598 + 0.197 = 5.795.
Sundaram & Das: Derivatives - Problems and Solutions . . . . . . . . . . . . . . . . . . . . . . . . . . . . . . . 251
If d = 0.02, the vega predicts call values will change by
V d = (19.685)(0.02) = 0.394
Thus, the predicted new call value is
5.598 0.394 = 5.204.
(h) If dr = +0.005, the rho predicts call values will change by
dr = (12.71)(+0.005) = +0.064
Thus, the predicted new call value is
5.598 + 0.064 = 5.662.
26. A stock is currently trading at 55. You hold a portfolio of the following instruments:
Long 200 shares of stock.
Long 200 puts with a strike of 50 and maturity of three months.
Short 200 calls with a strike of 60 and maturity of three months.
You are given the following information:
Instrument Price Delta Gamma Vega Theta Rho
Call with K = 50 6.321 0.823 0.038 7.152 5.522 9.730
Put with K = 50 0.700 0.177 0.038 7.152 3.053 2.615
Call with K = 55 3.079 0.565 0.057 10.827 6.812 6.993
Put with K = 55 2.396 0.435 0.057 10.827 4.096 6.586
Call with K = 60 1.210 0.297 0.050 9.515 5.513 3.779
Put with K = 60 5.465 0.703 0.050 9.515 2.551 11.035
(a) What is the current value of your portfolio?
(b) What is the delta of your portfolio? the gamma? the vega? the theta? the rho?
(c) Suppose you want to make your portfolio gamma neutral. What is the cost of
achieving this using the 55-strike call? What is the theta of your new position?
(d) What is the cost if you used the 55-strike put? What is the theta of the new
position?
Sundaram & Das: Derivatives - Problems and Solutions . . . . . . . . . . . . . . . . . . . . . . . . . . . . . . . 252
Answer: Let m denote the number of positions in the underlying, n
1
denote the position
in the 50-strike puts, and n
2
denote the position in the 60-strike calls. Since we are long
in the stock and the puts and short the calls, we have
m = 200 n
1
= 200 n
2
= 200
(a) The stock is trading at 55, the 50-strike put is worth 0.700 and the 60-strike call
is worth 1.210. Thus, the value of the position is
V = m (55) +n
1
(0.700) +n
2
(1.210) = 10, 898.
(b) The position greeks may be calculated using the greeks of the options and the
stock:

Pos
= m (+1) +n
1
(0.177) +n
2
(+0.297) = 105.20

Pos
= m (0) +n
1
(0.038) +n
2
(0.050) = 2.40
V
Pos
= m (0) +n
1
(7.152) +n
2
(9.515) = 472.6

Pos
= m (0) +n
1
(3.053) +n
2
(5.513) = +492.0

Pos
= m (0) +n
1
(2.615) +n
2
(3.779) = 1278.4
(c) From the previous answer, the position gamma is 2.40. The option we are to use
to help neutralize this quantity is the 55-strike call which has a gamma of 0.057.
Thus, the number of the 55-strike calls required to make the gamma zero (denoted,
say, n
3
) is
n
3
=
2.40
0.057
= +42.1
or a long position in approximately 42 calls. Since each of these calls costs 3.079,
the total cost of achieving gamma neutrality in this way is
42 3.079 = 129.318
Finally, the theta of each new call is 6.812. Thus, the new position theta is

Pos
= 492 (6.812)(42) = 205.896.
(d) If we make the position gamma neutral using the 55-strike put instead, the number
of puts we would need is exactly the same since the gamma of the 55-strike call
and the 55-strike put coincide. Since each of these puts costs 2.396, the total cost
of achieving gamma neutrality using the puts is
42 2.396 = 100.632
Since each of the puts has a theta of 4.096, the new position theta in this case
would be

Pos
= 492 (4.096)(42) = 319.968
Sundaram & Das: Derivatives - Problems and Solutions . . . . . . . . . . . . . . . . . . . . . . . . . . . . . . . 253
27. Using the same information as in Question 26, calculate the following quantities:
(a) The delta and gamma of a covered call portfolio with K = 55 (i.e., a portfolio
where you are long the stock and short a call with a strike of 55).
(b) The delta and gamma of a protective put portfolio with K = 50 (long the stock
and long a put with a strike of 50).
(c) The delta and gamma of a bull spread using calls with strikes of 55 and 60 (long
a 55-strike call, short a 60-strike call).
(d) The delta and gamma of a buttery spread using calls with strikes of 50, 55, and
60 (long a 50-strike call, long a 60-strike call, and short two 55-strike calls).
(e) The delta and gamma of a collar with strikes 50 and 60 (long position in the stock,
long a 50-strike put, short a 60-strike call).
Answer: Where the portfolio in question includes the stock, we use the facts that the
delta of the stock is 1 and the gamma of the stock is 0.
(a) The covered call portfolio (long stock, short 55-strike call): The delta of this
portfolio is
1 0.565 = 0.435
and the gamma of the portfolio is
0 0.057 = 0.057
With a positive delta and a negative gamma, the portfolio resembles a short position
in the 55-strike put optionwhich is, of course, exactly what it is! By put-call
parity, S C is equal to PV (K) P, so the covered call is equivalent to an
investment and a short position in a put.
(b) The protective put portfolio (long stock, long 50-strike put): The delta of this
portfolio is
1 0.177 = +0.823
and its gamma is
0 + 0.038 = +0.038
The delta and gamma are identical to that of a 55-strike call, which is again
unsurprising. By put-call parity, P + S is equal to C + PV (K), so the protective
put is equivalent to a long call plus an investment.
Sundaram & Das: Derivatives - Problems and Solutions . . . . . . . . . . . . . . . . . . . . . . . . . . . . . . . 254
(c) The bull spread using calls (long 55-strike call, short 60-strike call): The delta of
this portfolio is
0.565 0.297 = 0.268
and the gamma of the portfolio is
0.057 0.050 = 0.007
The positive delta indicates that the portfolio is bullish on price movements but
the relatively small delta and gamma indicate the bullishness is a limited one (as
the payo diagram shows).
(d) The buttery spread using calls (long 50-strike call, long 60-strike call, short two
55-strike calls): The delta of this portfolio is
0.823 + 0.297 2 (0.565) = 0.010
and its gamma is
0.038 + 0.050 2 (0.057) = 0.026
The delta close to zero and the negative gamma indicate this is a short volatility
position. This is, of course, conrmed by the payo diagram for this spread: the
payo at maturity is highest when the stock price is at 55 and declines with price
moves in either direction.
(e) The collar with the underling (long stock, long 50-strike put, short 60-strike call):
The delta of this portfolio is
1 0.177 0.297 = 0.526
and its gamma is
0 + 0.038 0.050 = 0.012
From put-call parity, it can be seen that this position is identical to an investment
plus a bull spread using the 50-strike call and the 60-strike call. The position is
bullish and will benet from volatility, but since the upside is also capped (as can
be seen from the payo diagram), the benets are limited.
28. You hold a portfolio that is short 2,000 puts, each with a delta of 0.63. What would
you do to delta-hedge the portfolio?
Answer: You would sell 2000 0.63 = 1260 shares of the stock. Since the delta of a
put is negative, a long put would be hedged by buying stock. The converse, a short
position in puts would be hedged by selling stock.
Sundaram & Das: Derivatives - Problems and Solutions . . . . . . . . . . . . . . . . . . . . . . . . . . . . . . . 255
29. A stock has a volatility of 40%. An at-the-money call on the stock has a vega of +21.45.
By how much will the call value change if volatility falls to 39%? Assuming the options
are European, what about the corresponding put?
Answer: The change in call value equals the vega multiplied by the change in volatility,
i.e., 0.01 21.45 = 0.2145.
The corresponding put also declines by exactly the same amount. This result follows
from put-call parity. Note that put-call parity is: C P = S PV (K); since the right-
hand side of the equation is unaected by changes in volatility, changes in volatility must
impact calls and puts equally.
30. The theta of a put with 23 days left to maturity is 17.50. Other things being equal,
by how much does the value of the put change if a day passes?
Answer: Assuming that the change of one day is denoted as t = 1/365, then the
value of the put decreases by P = 17.50 t = 0.0479.
31. You hold two types of calls and two types of puts on a given stock. The deltas and
gammas of the respective types are (+0.40, +0.03), (+0.55, +0.036), (0.63, +0.028)
and (0.40, +0.032). You have a long position in 1,000 of the rst type of call, a short
position in 500 of the second type of call, a long position in 1,000 of the rst type of
put, and a short position in 500 of the second type of put.
(a) What is the aggregate delta of your portfolio? the aggregate gamma?
(b) Suppose you decide to gamma hedge your portfolio using only the rst type of call.
What is the resulting delta of the new portfolio? What position in the underlying
is now required to create a delta-neutral gamma-neutral portfolio?
Answer: The aggregate delta is the sum of the deltas of the individual options:
Aggregate Delta = (1000)(0.40) + (500)(0.55)
+(1000)(0.63) + (500)(0.40) = 305
The aggregate gamma is the sum of the deltas of the individual options:
Aggregate Gamma = (1000)(0.030) + (500)(0.036)
+(1000)(0.028) + (500)(0.032) = 24
To gamma hedge the combined position we need to sell calls of the rst type because
the aggregate gamma is +24. The number of calls needed to be sold is 24/0.03 = 800
calls.
Sundaram & Das: Derivatives - Problems and Solutions . . . . . . . . . . . . . . . . . . . . . . . . . . . . . . . 256
The additional sale of 800 calls of the rst type injects an additional delta of 800
0.40 = 320. This increases the existing aggregate delta of 305 and results in a new
aggregate delta of 625. To oset this, 625 shares of the underlying need to be bought
to result in a zero-delta position.
32. You hold a portfolio that is short 800 calls, each with a rho of +21.50, and long 800
puts, each with a rho of 16.70. By how much does your portfolio value change if
interest rates move down by 20 basis points?
Answer: The aggregate rho of the position is equal to
Aggregate Rho = (800)(21.50) + (800)(16.70) = 30560
When interest rates move down by 20 basis points, the portfolio changes in value by
(30560)(0.0020) = 61.12
33. A stock is currently trading at $22.50. The delta of an at-the-money call on the stock
is +0.56 and the gamma is +0.035. If the stock price were to change to $22.25, by how
much would the call price change (using the delta alone)? What is the approximate new
value of the call delta?
Answer: Using the delta alone the call price would change by
(+0.56)(0.25) = 0.14
Hence, the call price would decline by 14 cents.
The delta would change by (0.035)(0.25) = 0.00875. So the new delta would be
0.56 0.00875 = 0.55125.
Sundaram & Das: Derivatives - Problems and Solutions . . . . . . . . . . . . . . . . . . . . . . . . . . . . . . . 257
Chapter 18. Exotic Options I: Path-Independent Options
1. What is the sign of the delta of a call option on a put option? Why? What about a put
on a call?
Answer: Consider rst, a call written on a put option. The put increases (resp. de-
creases) in value when the stock underlying the put decreases (resp. increases) in value,
so we have:
Stock Price = Put Value = Call-on-Put
and
Stock Price = Put Value = Call-on-Put
Since the stock price moves in the opposite direction to the call-on-put values, the delta
of the call-on-put (i.e., its sensitivity to changes in the stock price) is negative.
By an analogous argument, it can be established that the delta of a put on a call is also
negative; that of a put on a put is positive; and that of a call on a call is positive.
2. Why does a call on a put cost less than the put?
Answer: A call on a put gives you the right to buy the put. The right to buy the put
cannot cost more than the put itself without creating an arbitrage opportunity.
3. Find the price of a binary cash-or-nothing put option in a binomial tree with the following
parameters: S = 100, u = 1.10, d = 0.90, R = 1.02, and K = 100. Assume that the
binary pays a at amount of $10 if S
T
100, and nothing otherwise.
Answer: The risk-neutral probability given these parameters is:
q =
R d
u d
= 0.60.
The terminal prices on the binomial tree are:
u
2
S = 121
udS = 99
duS = 99
d
2
S = 81
Sundaram & Das: Derivatives - Problems and Solutions . . . . . . . . . . . . . . . . . . . . . . . . . . . . . . . 258
The binary put pays +10 if S
T
100 and nothing otherwise. Thus, its payos at
maturity are:
P
b
uu
= 0
P
b
ud
= 10
P
b
du
= 10
P
b
dd
= 10
Using the risk-neutral probabilities, the values of the put at the nodes uS and dS are,
respectively,
P
b
u
=
1
1.02
[(0.60)(0) + (0.40)(10)] = 3.92.
P
b
d
=
1
1.02
[(0.60)(10) + (0.40)(10)] = 9.80
Thus, the initial price of the option is
P
b
=
1
1.02
[(0.60)(3.92) + (0.40)(9.80)] = 6.15
4. Consider a digital call option, i.e., one that pays a dollar if at maturity, the stock price
S
T
is greater than the strike K.
(a) What is the sign of the delta of this option?
(b) When will the delta of this option be the highest?
Answer:
(a) As with any call, for a digital call also it is the case that higher stock prices increase
option value, so the delta will always be positive.
(b) The delta of the option will be the highest when the the change in option value
for a small change in stock price is the highest. This happens when the option
is near-the-money. When the option is deep in-the-money, it is nearly guaranteed
its xed payo of $1, so it hardly changes in value for small changes in the stock
price, while deep out-of-the-money, a small change in the stock price still leaves the
option deep out-of-the-money; the delta is small in both cases. Near-the-money,
a small change in the stoock price can make a big dierence to the likelihood
of nishing in-the-money (and receiving $1) or nishing out-of-the-money (and
receiving zero), so the delta will be high.
Sundaram & Das: Derivatives - Problems and Solutions . . . . . . . . . . . . . . . . . . . . . . . . . . . . . . . 259
5. You are given a three-period binomial tree with the following parameters: S = 100,
R = 1.02, u = 1.10, and d = 0.90. Consider a claim whose payo at maturity is given
by S
max
S
min
where S
max
and S
min
are, respectively, the highest and lowest stock
prices observed during the options life (including the initial price of S = 100). What is
the initial price of this claim?
Answer: The risk-neutral probability in this model is
q =
R d
u d
= 0.60.
Over three periods, there are 8 possible price paths: uuu, uud, udu, udd, duu, dud,
ddu, and ddd. For each of these price paths, the maximum and minimum prices, the
dierences between the maxima and minima, and the risk-neutral probabilities of each
path are provided in the table below:
Path S
max
S
min
S
max
S
min
RNP
uuu 133.1 100.0 33.1 0.216
uud 121.0 100.0 21.0 0.144
udu 110.0 99.0 11.0 0.144
udd 110.0 89.1 20.9 0.096
duu 108.9 90.0 18.9 0.144
dud 100.0 89.1 10.9 0.096
ddu 100.0 81.0 19.0 0.096
ddd 100.0 72.9 27.1 0.064
The initial value of the claim is just the expected payo under the risk-neutral probability
discounted back to the begining of the tree. Carrying out this computation, this value
is seen to be approximately X = 19.87.
6. Are ordinary American-style options path-independent?
Answer: Yes. A path-dependent option is one where the value of the option at any
given node on the option-pricing tree (if a tree is used) depends on the path taken to
reach that node. In the case of the standard American option, the decision to exercise
early does not depend on the path by which one arrived at the current stock price, but
only on the level of the price and the remaining time to maturity, which determines the
future value of the option.
Sundaram & Das: Derivatives - Problems and Solutions . . . . . . . . . . . . . . . . . . . . . . . . . . . . . . . 260
Intuitively, the decision to exercise an option early depends on whether the value from
exercise at that point in time is greater or less than the value of not exercising, i.e., the
expected future value of the option, also known as the continuation value. The value
of not exercising depends on the future possible evolution of the stock price and future
times at which early exercise may be optimal. It does not depend in any way on the
path taken by stock prices to reach the current price. Therefore, American options are
not path-dependent.
7. Consider a stock with current price S = 50 whose price process can be represented by a
binomial tree with parameters u = 1.221 and d = 0.819. Suppose the per-period gross
interest rate is R = 1.005.
(a) Find the value of a two-period European put option with a strike of K = 50.
(b) Using backwards induction on the tree, nd the value of a forward start put option
that comes to life in one period, is European, has a further life of two periods, and
will be at-the-money when it comes to life.
(c) Verify that your answers to parts (a) and (b) coincide.
(d) Suppose the puts had been American. What are the answers to parts (a) and (b)?
Do they still coincide?
Answer: The pricing of options on binomial trees is detailed in Chapter 13. We will use
the concepts from that chapter to compute the answers here.
Remark: The trees in the solution here are depicted as they would be on a spreadsheet.
From each cell, the one to the immediate right is the up-node and the one to the
right and one down is the down-node. Therefore, in the tree below, a price of 50
is followed by a price of 61.05 if the stock moves up and a price of 40.95 if the stock
moves down.
(a) First, we will price the European put option with strike K = 50. The stock tree is
as follows:
50.000 61.050 74.542
40.950 50.000
33.538
The risk-neutral probability of moving up on the tree is
q =
1.005 0.819
1.221 0.819
= 0.4627
And then superimposing the option payos on this tree for the European put leads
to the following tree for the option:
Sundaram & Das: Derivatives - Problems and Solutions . . . . . . . . . . . . . . . . . . . . . . . . . . . . . . . 261
4.705 0.000 0.000
8.801 0.000
16.462
Hence, the price of the put is 4.705.
(b) To value the forward start put option at the upper node at the end of the rst
period, we need the two period tree starting from that point, where the price is
61.05 and the strike will be at the money:
61.050 74.542 91.016
50.000 61.050
40.950
The forward start put option-value tree will be:
5.745 0.000 0.000
10.746 0.000
20.1
Hence, at the end of the rst period, at the upper node, the forward start option
is worth 5.745. One can easily show (this is the homogeneity property discussed in
Chapter 18, Section 18.2) that the value of the forward start option at the lower
node will bear an exact proportionate relationship to the value of the stock at the
lower node, and hence, we do not need to draw the entire tree but can price it
directly as follows:
Forward start Option at lower node = 40.95 5.745/61.05 = 3.853
Now, we can price the forward start option at time zero by discounting as usual.
Forward start Option at time zero = [5.745q + 3.853(1 q)]/1.005 = 4.705
which is the same as the price of the European put.
(c) As shown the prices to parts (a) and (b) do coincide.
(d) If the forward start option is American, then the same still holds. We leave the
reader to undertake detailed calculations, but we note that the price of the forward
start American option in the upper node after one period will be 5.908, and in the
lower node will be 3.963. Hence the price of the American forward start option at
time zero will be:
Fwd start American Option at time zero = [5.908q + 3.963(1 q)]/1.005 = 4.838
The tree for the American put option is as follows:
4.838 0.000 0.000
9.050 0.000
16.462
Sundaram & Das: Derivatives - Problems and Solutions . . . . . . . . . . . . . . . . . . . . . . . . . . . . . . . 262
which shows that the relationship found earlier for European options holds for
American options as well.
8. Consider a stock currently trading at S = 80 whose price evolution can be represented
by a binomial tree with parameters u = 1.226 and d = 0.815. Suppose the per-period
gross rate of interest is R = 1.005.
(a) Price a one-period call option on the tree with a strike of K = 76.
(b) Using backwards induction, nd the price of a forward start call option that comes
to life in one period, has a further life of one period, and has a strike equal to 95%
of the stock price when it comes to life. Verify that it is the same as your answer
to (a).
(c) Find the initial delta of the forward start.
(d) Now assume that the initial stock price is S = 1. Assuming the same parameters
for the binomial tree, nd the price of a one-period call with strike K = 0.95. How
does this price compare to the delta you identied in part (c)? Why?
Answer:
(a) The risk-neutral probability of an up move on the tree is
q =
1.005 0.815
1.226 0.815
= 0.4623
The stock tree underlying the call option is
80 98.080
65.200
The option tree for the call will be:
10.157 22.080
0.000
Hence, the one period call is priced at 10.157
(b) The two period tree will be as follows:
80 98.080 120.246
65.200 79.935
53.138
Using the upper subtree after one period, the pricing tree of the forward start
option at a strike of 95% of the initial price is as follows:
Sundaram & Das: Derivatives - Problems and Solutions . . . . . . . . . . . . . . . . . . . . . . . . . . . . . . . 263
12.452 27.070
0.000
Likewise the lower subtree for the forward start option is as follows:
8.278 17.995
0.000
Then, the initial price of the forward start call is:
12.452q + 8.278(1 q)
1.005
= 10.157
which is the same as the price of the European call option.
(c) The initial delta of the forward start option is
12.452 8.278
98.080 65.200
= 0.127
(d) If we priced the European call with an initial stock price of $1, then by simple
scaling the price would be
10.157
80
= 0.127
which is the same as the delta of the call computed in part (c).
9. This question deals with a Sprint-like repricing situation (see Section 18.2 for details of
the Sprint scheme). Assume that the current stock price is S = 24, the volatility of the
stock price is 45%, and the risk-free rate is 4%. Use the Black-Scholes model to answer
the following questions.
(a) Consider an option with a strike price of K = 32 and six years left to maturity.
Ignoring dividends, would you trade it in for a forward start call specied as in
Sprints scheme?
(b) What if the option had only one year to maturity?
Answer: Under Sprints scheme (see Section 18.2), if the original call had a maturity
date of T, the replacement forward-start option comes to life in 6 months, but also has
a maturity date of T, so the life of the forward start option (measured from the time it
comes to life) is 6 months less than the life of the original option.
Sundaram & Das: Derivatives - Problems and Solutions . . . . . . . . . . . . . . . . . . . . . . . . . . . . . . . 264
(a) The current call option may be valued using the Black-Scholes equation, and works
out to be $ 9.71.
The price of a forward start that (a) comes to life in six months, (b) has a remaining
life of 5.5 years at that point, and (c) will be at-the-money when it comes to life,
is just the value of an at-the-money call that comes to life right away and has a
life of 5.5 years. This works out to 11.24. Hence, it is clearly advantageous to
exchange the call in for the forward start one.
(b) If the current option has only one year left to maturity, it has a Black-Scholes
value of 2.204. The forward start oered in its place then has the same value as an
at-the-money call option with a maturity of 6 months, which has a value of 3.247.
It is still preferable to take the oer of the forward-start in place of the existing
option.
10. A stock is currently trading at $24. Assume that its volatility is 35% and the term-
structure of interest rates is at at 6%.
(a) What is the price of a forward start call option with T

= 1 year, = 1 year,
and = 1.10? Note that T

is maturity of the forward start period, and is


the maturity of the option once started. Also, is the strike multiplier, i.e., strike
K = S
T
.
(b) What is the delta of this option?
Answer:
(a) The forward start option has the same price as the current one-year call option
with strike of 24 1.1, and using the Black-Scholes formula gives a price of $2.98.
(b) The delta of the option is the value of the same option when the initial price is
$1, which is the ratio of the call price to that of the underlying stock price, i.e.,
2.98/24 = 0.124.
11. Consider the binomial tree of Figure 1. Suppose that the per-period interest rate is
R = 1.02.
(a) Show that the price of a call on a put in this model with a strike of k = 4 and a
maturity of one period is 1.58, where the underlying put has a strike of 100 and a
maturity of 2 periods.
(b) Show that the delta of the call on the put in the binomial example is 0.202. (Use
the usual formula for a binomial delta.)
Sundaram & Das: Derivatives - Problems and Solutions . . . . . . . . . . . . . . . . . . . . . . . . . . . . . . . 265
Figure 1: Stock Price Tree for Binomial Illustrations
121
100
90
110
99
81
(c) Verify that a position consisting of a short position in the option and a short position
in 0.202 units of the stock is perfectly risk-less over the compound options one-
period life.
Answer:
(a) When we price the two-period put on the binomial tree we get the following values
at the end of period two:
P
uu
= 0 P
ud
= 1 P
dd
= 19
The risk-neutral probability of an up move on the tree is
q =
1.02 0.90
1.10 0.90
= 0.6
Discounting the terminal put values back to the nodes at the end of period one
gives the following put values:
P
u
= 0.392, P
d
= 8.039
Given these put values, the call-on-put payos at the nodes at the end of period
one are:
C-on-P
u
= 0, C-on-P
d
= 4.039
And weighting these values by the risk-neutral probabilities and discounting them
back to time zero gives the value of the call on the put:
C-on-P
0
=
0 0.6 + 4.039 0.4
1.02
= 1.58
Sundaram & Das: Derivatives - Problems and Solutions . . . . . . . . . . . . . . . . . . . . . . . . . . . . . . . 266
(b) The delta of the call on the put is:

ConP
=
0 4.039
110 90
= 0.202
(c) We compute the value of a short C-on-P option and short 0.202 units of stock in
both up and down states after one period:
In the up-state we have
0 0.202 110 = 22.22
In the down-state we have
4.039 0.202 90 = 22.22
Since the portfolio gives the same value irrespective of whether the stock goes up
or down, the position is riskless.
12. Describe a put-call parity relationship for compound options, i.e., one that connects
calls-on-calls to puts-on-calls (or calls-on-puts to puts-on-puts).
Answer: We note that a put-call parity relationship holds no matter what the underlying
for a European option may be, and no matter what the terminal distribution of the
underlying. Hence, the fact that the underlying in this question is an option does not
invalidate the standard put-call parity relation. We write it as follows:
Call-on-Call Put-on-Call = Underlying Call PV (Strike of Call-on-Call).
13. Consider a stock currently trading at $45. Suppose its price evolution can be represented
by a binomial tree with u = 1.05 and d = 0.95. The risk-less rate per period is R = 1.01.
Calculate the following:
(a) The price of a two-period European call option with K = 43.
(b) The price of a one-period call on the two-period call with a strike of K = 2.
(c) The price of a one-period put on the two-period call with a strike of K = 0.50.
(d) The price of a forward start option beginning in one period with a further life of
one period that will be at-the-money when it starts.
Answer: Exercise for the reader. The answers are: (a) 3.22; (b) 1.59; (c) 0.0; (d) 1.34.
Sundaram & Das: Derivatives - Problems and Solutions . . . . . . . . . . . . . . . . . . . . . . . . . . . . . . . 267
14. Consider a two-period cash-or-nothing binary option in the binomial example of Figure 1
with K = 90 and M = 10. Assume that the per-period interest rate is R = 1.02.
(a) Show that the initial value of the option is C
C-or-N
= 8.07.
(b) What is the value of an asset-or-nothing option in this case?
(c) What is the value of a vanilla option? Verify that the following equation holds.
C
A-or-N

K
M
C
C-or-N
= C
Answer:
(a) First compute the risk-neutral probability of an up-move in the stock price. This is
q =
1.02 0.90
1.10 0.90
= 0.6
Next, at the end of the second period, there are three nodes, and the option
values in the three nodes, starting from the uppermost, are {10, 10, 0} respectively.
Discounting these values back to time zero, weighting by the correct probabilities,
gives the price of the C-or-N option, which is 8.07.
(b) For the A-or-N option, at the end of the second period, the option values in the
three nodes, starting from the uppermost, are {121, 99, 0} respectively. Discount-
ing these values back to time zero, weighting by the correct probabilities, gives the
price of the A-or-N option, which is 87.54.
(c) The plain vanilla option is also easily priced on the tree for K = 90 and the
resultant price is 14.88. We can now check the parity relationship:
A-or-N
K
M
C-or-N = 87.54
90
10
8.07 = 14.88
which is the price of the plain vanilla call option. What we have just shown is that
the standard call option may be decomposed into two digital options, an asset-or-
nothing option and a cash-or-nothing option, the dierence of which makes up the
vanilla call.
15. Consider a cash-or-nothing option in the binomial tree setting of Figure 1 with K = 90
and M = 10. Let
b
denote the initial delta of the option,
b
u
the delta following an up
move in the stock price, and
b
d
the delta following a down move. Show that
b
u
<
b
even though the option has moved into-the-money.
Answer: It can be shown that
b
u
= 0 and
b
d
= 0.56. The details of the computation
are left to the reader. Although the option moves more into-the-money when the
Sundaram & Das: Derivatives - Problems and Solutions . . . . . . . . . . . . . . . . . . . . . . . . . . . . . . . 268
stock price goes up in the rst period, the delta declines for the reasons explained in
Section 18.3; in particular, in this example, once the node uS has been reached, this
digital call is guaranteed to nish in-the-money (and to receive a at payo of M = 10)
no matter what happens next period.
16. Let C
C-or-N
denote a cash-or-nothing option that pays M is S
T
K. Let P
C-or-N
be
a put version of this cash-or-nothing option, i.e., it pays M if S
T
< K and nothing
otherwise. What is the relation between the prices of these two options?
Answer: If you purchase both options, then you will get M for sure. Hence, the following
relationship holds:
C
CorN
+P
CorN
= PV (M)
17. Consider two-period European options with a strike of 100 in the binomial model of
Figure 1. Assume the per-period interest rate is R = 1.02.
(a) Find the value of a straddle in this model.
(b) Find the value of a chooser where the holder must decide between the call and put
at the end of one period.
(c) Why is the dierence in values between the straddle and chooser so small?
(d) What are the deltas of the straddle and the chooser?
Answer: As before, we lay out the trees for the stock price and the option using the
spreadsheet format as described in Question 7.
(a) First, we determine the risk-neutral probability of an up move on the binomial tree.
q =
1.02 0.9
1.1 0.9
= 0.6
In order to price a straddle, we need to sum the values of a call and a put. The
option pricing tree for the call is as follows:
7.27 12.35 21.00
0 0
0
and the tree for the put is as follows:
3.38 0.39 0
8.04 1
19
Sundaram & Das: Derivatives - Problems and Solutions . . . . . . . . . . . . . . . . . . . . . . . . . . . . . . . 269
Hence, the price of the straddle is the sum of the call and put, i.e., 10.65.
(b) The value of a chooser option takes the value of the call at time 1 in the upper
node, and the value of a put in the lower node. The price at time 0 will be given
by the usual risk-neutral pricing rule:
Chooser =
12.35q + 8.04(1 q)
1.02
= 10.42
(c) The main dierence between a chooser and a straddle, in general, is that the
chooser involves an irrevocable choice between the call and the put on the choice
date, whereas in the straddle, one can keep both the call and the put all the way
to maturity. So the dierence in values is driven by the consequences of making
the wrong choice on the choice date, i.e., making a choice that, ex-post, turns
out to be the wrong one.
In this example, there is only one such wrong choice possible: when the node uS
occurs, the chooser-holder will choose the call, but if the price now moves down to
the node udS, the call nishes out-of-the-money while the put would have nished
in-the-money by $1. Since the consequence of the wrong choice is only $1 and
the (risk-neutral) probability of the node udS is only 0.24, the present value of the
consequence of the wrong choice is just
1
1.02
2
[0.24 1] = 0.23.
(d) The delta of the straddle and chooser are, respectively,
Straddle delta =
(12.35 + 0.39) (0 + 8.04)
110 90
= 0.24
Chooser delta =
12.35 8.04
110 90
= 0.22
18. Price a chooser option using the Black-Scholes formula with the following inputs: S =
100, K = 100, the maturity at which the option holder has to opt for a call or a put is
= 1 year, the nal maturity of the option is T = 2 years, risk-free rate r = 0.10, and
dividends = 0.03.
Answer: The formula for pricing the chooser is:
V
ch
= C(S, K, T) +e
(T)
P(S, Ke
(r)(T)
, )
Applying the formula, we get the value of the chooser option to be
21.7239 + 0.9704 5.6220 = 27.1795.
Sundaram & Das: Derivatives - Problems and Solutions . . . . . . . . . . . . . . . . . . . . . . . . . . . . . . . 270
19. Using the same input values as in the previous question, compute the value of the
straddle. Compare the price of the straddle with that of the chooser. Which is greater?
Why?
Answer: The straddle price is
V
straddle
= C(S, K, T) +P(S, K, T) = 21.7239 + 9.4205 = 31.1444
The straddle is worth more because it allows the holder to keep both call and put options
alive until maturity T. In the chooser, after maturity , the holder will retain only one
of the options.
20. You are asked to price a quanto option on the DAX index. The DAX is currently trading
at a value of 5000. Price a one-year maturity ATM option on one unit of the DAX given
that the current exchange rate is $0.8/e. The volatility of the DAX is 50%, and that of
the exchange rate is 20%. The correlation between the DAX return and exchange rate
is +0.25. The US risk-free rate is 1%, and the euro interest rate is 2%. Dividends on
the DAX are 1%.
Answer: The formula for quanto options is given in the chapter. The Octave program
code is as follows:
%Quanto Pricing Model
%S_0f: Foreign stock index
%X_0: Exchange rate, dollars per FCY
%sig_s: index volatility
%T: maturity
%r: domestic interest rate
%r_f: foreign interest rate
%delta_f: index dividend
%rho: correlation between index and fx rate
%sig_x: exchange rate volatility
%K: strike in FCY
function u=quanto(S_0f,X_0,K,sig_s,T,r,r_f,delta_f,rho,sig_x);
delta_A = delta_f + r - r_f - rho*sig_x*sig_s;
Gam1 = (log(S_0f/K) +
(-delta_f + r_f + rho*sig_s*sig_x + 0.5*sig_s^2)*T)/(sig_s*sqrt(T));
Gam2 = Gam1 - sig_s*sqrt(T);
u = X_0*(exp(-delta_A*T)*S_0f*normcdf(Gam1) - K*exp(-r*T)*normcdf(Gam2));
Application of the model results in a quanto value of $868.16.
Sundaram & Das: Derivatives - Problems and Solutions . . . . . . . . . . . . . . . . . . . . . . . . . . . . . . . 271
octave-3.0.1:25> quanto(5000,0.8,5000,0.50,1,0.01,0.02,0.01,0.25,0.20)
ans = 868.16
21. Using the same parameters as in the previous question, price the quanto when the
correlation between the DAX stock index and the $/eexchange rate is 0.25 instead of
+0.25. What happens to the price of the quanto? Explain.
Answer: Applying the program from above, we get a quanto value of $746.84.
octave-3.0.1:26> quanto(5000,0.8,5000,0.50,1,0.01,0.02,0.01,-0.25,0.20)
ans = 746.84
The value of the quanto has dropped when the stock index and exchange rate are
negatively correlated versus when they are positively correlated. Why?
This quanto call increases in value when the DAX increases, or when the euro appreciates
versus the dollar. When the DAX and exchange rate (in dollars per euro) are positively
correlated, both increase together on average, thereby pushing up the value of the quanto
option.
When the DAX and dollars per euro are negatively correlated, on average, a gain in the
DAX will be oset by a decline in the exchange rate, and vice-versa. Therefore, the
quanto is worth less when the DAX and exchange rate are negatively correlated than
when they are positively correlated.
22. Consider an option that pays the holder the amount
max{(S
T
K)
2
, 0} (11)
at maturity where S
T
is the terminal price of the stock and K the options strike price.
Such an option is one example of a power option. Consider a binomial tree with initial
stock price S = 60 and parameters u = 1.20 and d = 0.833. Suppose the per-period
gross rate of interest is R = 1.01. Value a power option of the form (11) that has a
maturity of two periods and a strike of K = 70.
Answer: Remark: The trees in the solution here are depicted as they would be on a
spreadsheet. From each cell, the one to the immediate right is the up-node and the one
to the right and one down is the down-node.
The risk-neutral tree probability is
q =
1.01 0.833
1.2 0.833
= 0.4823
The stock tree is as follows:
Sundaram & Das: Derivatives - Problems and Solutions . . . . . . . . . . . . . . . . . . . . . . . . . . . . . . . 272
60.00 72.00 86.40
49.98 59.98
41.63
based on which the option tree is
321.94 179.94 268.96
460.44 100.48
804.67
Hence, the price of the power option is $321.94.
23. Another example of a power option is one that pays the amount
max{S
2
T
K
2
, 0} (12)
(a) Show that the payos of this option may be written in terms of (11) and a vanilla
call with strike K. [Hint: Expand the term (S
T
K)
2
and rearrange.]
(b) Price the option (12) using the same binomial tree as in Question 23.
Answer:
(a) The following shows the required relation between the power option and other
options:
max[(S
T
K)
2
, 0] = max[S
2
T
2KS
T
+K
2
, 0]
= max[(S
2
T
K
2
) 2K(S
T
K), 0]
= max(S
2
T
K
2
, 0) 2K max(S
T
K, 0)
That the last two lines are equivalent is left for the reader to verify. This shows
that the nal payo is the dierence between the payos to the power option and
2K units of a vanilla call.
(b) The tree for pricing the option using the same numbers as in the previous question
is as follows:
584.86 1224.80 2564.96
0 0
0
So, the price of the option is $584.86.
Sundaram & Das: Derivatives - Problems and Solutions . . . . . . . . . . . . . . . . . . . . . . . . . . . . . . . 273
24. Consider a Black-Scholes setting with volatility and dividend yield q. Show that a
closed-form expression for the price of a power option of the form (12) is obtained
simply by using the Black-Scholes vanilla call option formula with
(a) A strike of K
2
.
(b) A volatilty of 2.
(c) A dividend yield of 2q (r +
2
).
Answer: We start with the stochastic process for the stock price S:
dS = (r q)S dt +S dZ
We are interested in an option on the variable Y = S
2
with strike K
2
. Using Itos
lemma, we have that
dY = 2S[(r q)S dt +S dZ] +
1
2
2
2
S
2
dt
dY
Y
= [r (2q (r +
2
))] dt + 2 dZ
which shows that the volatility is 2 and the dividend is 2q (r +
2
).
25. This question generalizes Question 24. Consider a power option with payo
max{S

T
M, 0} (13)
If we take = 2 and M = K
2
, this is (12). Consider a Black-Scholes setting with
volatility and dividend yield q. Show that the closed-form solution for this options
price is the vanilla call price with
(a) A strike of M.
(b) A volatility of .
(c) A dividend yield of q ( 1)(r +
1
2

2
).
Answer: We start with the stochastic process for the stock price S:
dS = (r q)S dt +S dZ
Sundaram & Das: Derivatives - Problems and Solutions . . . . . . . . . . . . . . . . . . . . . . . . . . . . . . . 274
We are interested in an option on the variable Y = S

with strike M. Using Itos


lemma, we have that
dY = S
1
[(r q)S dt +S dZ] +
1
2
( 1)S
2

2
S
2
dt
dY
Y
= [r (q ( 1)(r +
1
2

2
))] dt + dZ
which shows that the volatility is and the dividend is (q ( 1)(r +
1
2

2
)).
Note that setting = 2 provides the result from the previous question.
26. A corridor option is one that pays a xed sum M if the price at maturity lies between
two specied levels K
1
and K
2
but nothing otherwise. Consider a two-period binomial
tree with parameters u = 1.10 and d = 0.91. Suppose the initial stock price is S = 100
and R = 1.01.
(a) Find the initial value and the value at all points on the tree of a corridor option
that pays M = 10 if the price lies between K
1
= 90 and K
2
= 110 (both prices
inclusive), and nothing otherwise.
(b) Find the deltas of the corridor option at all points in the tree. Intuitively, why is
the delta positive in parts of the tree and negative in others?
Answer: Remark: The trees in the solution here are depicted as they would be on a
spreadsheet. From each cell, the one to the immediate right is the up-node and the one
to the right and one down is the down-node.
(a) The risk-neutral probability of an up move on the tree is:
q =
1.01 0.91
1.10 0.91
= 0.5263
The stock tree is as follows:
100.00 110.00 121.00
91.00 100.10
82.81
based on which the option tree is
4.89 4.69 0
5.21 10.00
0
(b) The deltas are:
Sundaram & Das: Derivatives - Problems and Solutions . . . . . . . . . . . . . . . . . . . . . . . . . . . . . . . 275
-0.0274 -0.4785
0.5784
The sign of the delta depends on whether an up move in the stock price moves the stock
to the center of the range (K
1
, K
2
), in which case the value of the option increases,
or to the edge of the range in which case the value decreases. When the price nears
the edges of the range, there is a heightened probability of no payothis reduces the
delta.
27. Find a closed-form expression for the price of a corridor option in a Black-Scholes setting.
[Hint: Show that a corridor is just a combination of two binary cash-or-nothing options.]
Answer: Given a range (K
1
, K
2
), the corridor option value is given by the following
expression:
C
CorN
(K
1
) C
CorN
(K
2
) = K
1
e
rt
N[d
2
(K
1
)] K
2
e
rt
N[d
2
(K
2
)]
where d
2
(K) is the value of the Black-Scholes d
2
function for a strike level of K.
28. Given the following parameters, price a gap call option: S = 100, K = 100, T = 1,
= 0.3, r = 0.10, and M = 90. Dividends are = 0.02. Remember, K is the strike
price of both calls, and M is the payo for the cash-or-nothing call if it nishes in the
money.
Answer: A gap call option is the dierence between an asset-or-nothing (A-or-N) call
and a cash-or-nothing (C-or-N) call. We price each one in turn.
C
AorN
= e
T
SN(d
1
) = 64.84
C
CorN
= e
rT
MN(d
2
) = 44.50
The price of the gap option is
Gap Call = C
AorN
C
CorN
= 20.34
29. Let the following Black-Scholes parameters be given: S = 100, K = 100, T = 1,
= 0.3, r = 0.10, and = 0.02. Consider a pay-later option (see Section 18.3 for the
denition of the option). Find the payo M of the cash-or-nothing call in this pay-later
option. Remember, K is the strike of both calls.
Sundaram & Das: Derivatives - Problems and Solutions . . . . . . . . . . . . . . . . . . . . . . . . . . . . . . . 276
Answer: A pay-later option is a gap option where the payo M is set such that the
initial price of the option is zero. Therefore we solve for M such that
Gap Call = C
AorN
C
CorN
= 0
Using the formulae for the aset-or-nothing and cash-or-nothing calls, we are to nd M
such that
e
T
S N(d
1
) e
rT
M N(d
2
) = 0
Solving this equation, we get the value of M = 131.15.
30. Consider a Margrabe exchange option. Suppose the initial prices of the two stocks
are S
1
= S
2
= 100 and
1
= 0.40. Suppose also that the returns on the stocks are
uncorrelated. Assume no dividends.
(a) Using the closed-form expressions for the price of these options, identify the price
of the exchange option when
2
= 0,
2
= 0.20,
2
= 0.40, and
2
= 0.60.
(b) Is there a trend in the price? Intuitively, why is this the case?
Answer: (a) The following program code in Octave implements the Margrabe exchange
option:
%Margabe Exchange Option
%s: stock prices
%t: maturity
%v: volatilities
%rho: correlation of returns
%q: dividends
function u = margrabe(s1,s2,t,v1,v2,rho,q1,q2);
v = sqrt(v1^2 + v2^2 - 2*rho*v1*v2);
d1 = (log(s1/s2)+(q2-q1+0.5*v^2)*t)/(v*sqrt(t));
d2 = d1 - v*sqrt(t);
u = exp(-q1*t)*s1*normal_cdf(d1) - exp(-q2*t)*s2*normal_cdf(d2);
Running this program for the dierent values of volatility of the second stock results in
the following:
Sundaram & Das: Derivatives - Problems and Solutions . . . . . . . . . . . . . . . . . . . . . . . . . . . . . . . 277
octave:1> margrabe(100,100,1,0.4,0,0,0,0)
ans = 15.852
octave:2> margrabe(100,100,1,0.4,0.2,0,0,0)
ans = 17.694
octave:3> margrabe(100,100,1,0.4,0.4,0,0,0)
ans = 22.270
octave:4> margrabe(100,100,1,0.4,0.6,0,0,0)
ans = 28.157
(b) As the volatility of one stock increases, holding the volatility of the other constant,
the volatility of the ratio of stock prices also increases. The Margrabe option is based
on the payo of an underlying that is the ratio of two stock prices. So when the ratio
becomes more volatile, the option value increases.
31. What is the value of a one-year option on the maximum of two assets when both assets
are trading at $100 each, their volatilities are 50% and 40%, and the correlation between
their returns is +0.30? Both assets pay no dividends.
Answer: The following program may be used to compute the option value:
%Max od 2 assets Exchange Option
%s: stock prices
%t: maturity
%v: volatilities
%rho: correlation of returns
%q: dividends
function u = max2assets(s1,s2,t,v1,v2,rho,q1,q2);
v = sqrt(v1^2 + v2^2 - 2*rho*v1*v2);
d1 = (log(s1/s2)+(q2-q1+0.5*v^2)*t)/(v*sqrt(t));
minus_d2 = (log(s2/s1)+(q1-q2+0.5*v^2)*t)/(v*sqrt(t));
u = exp(-q1*t)*s1*normcdf(d1) + exp(-q2*t)*s2*normcdf(minus_d2);
Running this function gives a value of $121.23.
octave-3.0.1:33> max2assets(100,100,1,0.5,0.4,0.3,0,0)
ans = 121.23
Sundaram & Das: Derivatives - Problems and Solutions . . . . . . . . . . . . . . . . . . . . . . . . . . . . . . . 278
32. What is the value of a one-year option on the maximum of two assets when both assets
are trading at $100 each, their volatilities are 50% and 40%, and the correlation between
their returns is 0.30? Both assets pay no dividends. Compare the value against the
case where the correlation is positive +0.30. Explain your result intuitively.
Answer: The value of the option is
octave-3.0.1:34> max2assets(100,100,1,0.5,0.4,-0.3,0,0)
ans = 128.41
When the correlation is +0.30, the option is worth $121.23. Hence, negative correlation
makes this option more valuable.
Intuitively, this is so because negative correlation implies that when one asset is dropping
in value the other is increasing. Since the option is on the maximum of two assets,
negative correlation means that at least one asset is likely to be rising in value. In
contrast, when correlation is positive, both assets may be falling together. Therefore,
under negative correlation, the number of states of the world in which at least one asset
has rising value will be larger, resulting in greater option value.
33. What is the value of a one-year option on the minimum of two assets when both assets
are trading at $100 each, their volatilities are 50% and 40%, and the correlation between
their returns is +0.30? Both assets pay no dividends.
Answer: We apply the simple formula:
V
min
= S
1
+S
2
V
max
The price of V
max
is 121.23 (see the answer to the previous question). Hence the option
value is
V
min
= 100 + 100 121.23 = 78.773
as seen here:
octave-3.0.1:35> 100 + 100 - max2assets(100,100,1,0.5,0.4,0.3,0,0)
ans = 78.773
Sundaram & Das: Derivatives - Problems and Solutions . . . . . . . . . . . . . . . . . . . . . . . . . . . . . . . 279
Chapter 19. Exotic Options II: Path-Dependent Options
1. What makes an exotic option path-dependent? Think of an example of such an option
that is not covered in this chapter. Explain why you consider it path-dependent.
Answer: An option becomes path dependent if its payo at exercise time depends on
not just the value of the underlying at that point but on the path taken by the underlying
to reach that point.
An example of a path-dependent option not considered in the chapter is the Japanese
reset convertible described in Chapter 21. This option resembles a call in which, if the
price drops suciently as to cross a trigger level, the option holder receives > 1
units of the option for each unit held. (See Chapter 21 for a more precise denition.)
For example, suppose the initial strike is K = 100, the trigger level is H = 80, and
= 1.25. If
the price at maturity is 110 and this price was reached without the trigger level
ever being breached, the option holder receives a payo of 10.
if the price at maturity is 110 and this price is reached after the trigger level was
breached, the option holder holds 1.25 calls, so receives a payo of 12.50.
Of course you may also dream up other examples.
2. If an option is path-dependent, do we need to use a nonrecombining binomial tree for
pricing it? That is, do we need to evaluate the option payo along each path of stock
prices separately rather than being able to price it by backwards recursion using the
original stock price tree?
Answer: Since a path-dependent option is characterized by payos at maturity that
depend on the path taken to arrive at that node, it would appear that we do indeed
need to treat each path separately (so, for example, we cannot draw a recombining
tree of the evolution of option values). Interestingly, and somewhat surprisingly, this
is not always the case. There exist path-dependent options which may be priced using
backwards recursion; what is needed is that during backward recursion, we are able to
impose path-dependence at each node without the need to follow the entire path from
the beginning.
One example is the knock-out barrier option. To price these options, we can use a
backwards recursion procedure in which we set the option value at nodes that lie beyond
the barrier to zero (or to their rebate value if this is positive). For a specic illustration of
the pricing procedure, see the answer to Question 21. We note however, that backwards
induction cannot be applied on a knock-in option because we need to follow the path
forward until it strikes the activation barrier.
Sundaram & Das: Derivatives - Problems and Solutions . . . . . . . . . . . . . . . . . . . . . . . . . . . . . . . 280
3. Are American options path-dependent? Why or why not?
Answer: No, they are not. See Question 6 in the previous chapter.
4. Why are Asian options popular? State some uses of Asian options.
Answer: Asian options are options on the average of the underlying over a pre-specied
period of time. Here are two features that make them useful (see Section 19.3 for
details).
Asian options are useful for hedging exposure to the average, i.e., repeated
exposure over a prolonged period of time. In particular, Asian calls oer a cheaper
way of protecting against exposure to the average than do vanilla calls.
Asian options guard against the eect of price spikes close to maturity. As a
consequence, they are less vulnerable to price manipulation than vanilla options.
5. Is an Asian option cheaper or more expensive than a plain vanilla option? Why? Is this
always true?
Answer: There are two eects we need to be aware of. First the volatility eect. Since
the Asian option is an option on an average, and the volatility of an average is lower
than that of the variable itself, the option premium will be lower on this count. This is
true for both calls and puts.
The second eect is the growth rate eect. Averaging the stock price slows down the
growth rate of the stock. Since the growth rate is dampened, this eect too pushes
down the value of Asian calls relative to vanilla calls. But the lower growth rate benets
the holders of puts who want lower prices, so increases the value of Asian puts relative
to vanilla puts.
Netting the two eects, Asian calls always cost less than vanilla calls, but Asian puts
could cost more or less than vanilla puts depending on which eect dominates.
6. Explain the dierence between an Asian average-price option and an Asian average-strike
option.
Answer: Let S
ave
denote the average price of the underlying over the life of the option.
In an Asian average-price option, the holder of a call would receive
max{S
ave
K, 0}
Sundaram & Das: Derivatives - Problems and Solutions . . . . . . . . . . . . . . . . . . . . . . . . . . . . . . . 281
at maturity, where K is the strike price of the option. In an Asian average-strike option,
the average price S
ave
plays the role of the strike price. That is, if S
T
denotes the
terminal asset price, the holder of a call option receives
max{S
T
S
ave
, 0}.
7. Distinguish between a lookback price option and a lookback strike option.
Answer: Let S
max
denote the maximum price of the underlying over the life of the
option, and S
min
represent the minimum. In a lookback price option, the holder of a
call receives the amount
max{S
max
K, 0}
at maturity, where K is the strike price of the option. In a lookback strike call option,
the minimum price S
min
plays the role of the strike; the holder of the call receives
max{S
T
S
min
, 0}
at maturity, where S
T
is the terminal price of the underlying.
8. Consider an initially at-the-money knock-out put option with the knock-out price H set
above the strike K. Will the price of the put be greater, smaller, or equal to that of a
vanilla put? Why?
Answer: A vanilla put will result in a positive cash ow as long as the terminal asset
price S
T
is smaller than the strike K. A knock-out put will result in a positive cash
ow only if two conditions are met: not only must (a) the terminal asset price S
T
be
smaller than the strike K, it must also be the case that (b) the knock-out barrier was
not breached during the life of the option. Therefore, the knock-out option must cost
less than the vanilla put.
9. Why does a knock-out call cost less than the corresponding vanilla call?
Answer: A knock-out call option is cheaper than the corresponding vanilla call because
of the possibility that the option may get knocked-out, thereby becoming worthless.
Sundaram & Das: Derivatives - Problems and Solutions . . . . . . . . . . . . . . . . . . . . . . . . . . . . . . . 282
10. If you want to invest in the upside potential of a stock but are afraid of overpaying for
options that favor your view, suggest two ways in which you may buy a single barrier
option that implements your view more cheaply.
Answer: Clearly, we wish to buy calls since we are investing in a potential up move in
the stock price. Two barrier versions that do the trick are:
Down-and-out call: this is cheaper than a vanilla call because of the knockout
feature. It provides the upside gain required. Also when the stock falls, this knocks
out, and is therefore cheaper. There is one drawback though, which is that it gives
up the gains when the stock eventually ends up in the money but has previously
dropped resulting in a knockout. But, given the likelihood of this is smaller when
the knockout has occurred, this is not a substantive opportunity cost.
An Up-and-in call. Here, the option activates when the expected view actually
materializes. Since the option may not be activated, it is cheaper. The drawback
here may be that there is enough upside to trigger the option inception, but not
enough momentum to carry the stock over the exercise price at maturity.
11. You are the holder of an up-and-out put option. Now you want to replace it with a
plain vanilla put at the same strike and maturity. What option should you purchase to
achieve this?
Answer: You need to buy an up-and-in put option with the same strike and maturity.
Taken together you now have a portfolio of options that is equivalent to the plain vanilla
put.
12. Which has greater value, an arithmetic-average-price Asian option or a geometric-
average-price one?
Answer: The answer to this question depends on which averaging scheme leads to a
higher average price, arithmetic or geometric. It turns out that the arithmetic average of
a series of n positive numbers will be higher than its geometric average. Thus, arithmetic
average calls will be worth more than geometric average ones, but the opposite will be
the case for puts.
To see this in a simple case, consider the case of two numbers a, b > 0. The arithmetic
average of these numbers is (a+b)/2 and the geometric average is

ab. The dierence


between the arithmetic average and the geometric average is positive (except when
a = b):
(a +b)/2

ab =
_
_
a
2

_
b
2
_
2
Sundaram & Das: Derivatives - Problems and Solutions . . . . . . . . . . . . . . . . . . . . . . . . . . . . . . . 283
which is strictly positive if a = b and is zero if a = b. What is true for two numbers is
also true for any n, as may be veried by the reader.
13. What is the sign of the delta of an up-and-out call option? Explain how the delta
depends on the closeness of the current stock price to the barrier.
Answer: The sign of the up-and-out call depends on the closeness of the current stock
price to the knock-out barrier. If the stock is far from the barrier, then increases in
the price raise the chances of it ending up in the money, but do not vastly increase
the chance that the option will get knocked out. However, if the stock is close to the
barrier, it is likely already in the money, and further increases only increase the chance
of it being knocked out, thus reducing its value. Hence, the delta in the former situation
will be positive, but in the latter case will be negative.
14. Suppose you are given a two-period binomial tree with the following parameters: S =
100, R = 1.01, u = 1.04, and d = 0.96. Consider a two-period Asian call option where
the averaging is done over all three prices observed, i.e., the initial price, the price after
one period, and the price after two periods.
(a) Suppose the option is an average-price option with a strike of 100. What is its
initial price?
(b) Suppose the option is an average-strike option. What is its initial price?
Answer: We are given S = 100, r = 1.01, u = 1.04, and d = 0.96. Thus, the
risk-neutral probability is
q =
R d
u d
= 0.625
The stock price tree has the following prices:
S = 100,
uS = 104, dS = 0.96
u
2
S = 108.16, udS = 99.84, duS = 99.84, d
2
S = 92.16.
Letting

S
uu
denote the average price if two up moves occur, etc., we have

S
uu
= [100 + 104 + 108.16]/3 = 104.05

S
ud
= [100 + 104 + 99.84]/3 = 101.28

S
du
= [100 + 96 + 99.84]/3 = 98.61

S
dd
= [100 + 96 + 92.16]/3 = 96.05
Sundaram & Das: Derivatives - Problems and Solutions . . . . . . . . . . . . . . . . . . . . . . . . . . . . . . . 284
(a) The payos at maturity from an Asian average-price option with a strike of 100
are

C
uu
= 4.05

C
ud
= 1.28

C
du
= 0

C
dd
= 0
Using backwards induction, the value of the option at earlier nodes is:

C
u
=
1
1.01
[(0.625)(4.05) + (0.375)(1.28)] = 2.98

C
d
=
1
1.01
[(0.625)(0) + (0.375)(0)] = 0
Thus, the initial option price is

C =
1
1.01
[(0.625)(2.98) + (0.375)(0)] = 1.84
(b) With an average-strike option, the average price plays the role of the strike price.
Thus, the payos at maturity are

C
uu
= max{108.16 104.05, 0} = 4.11

C
ud
= max{99.84 101.28} = 0

C
du
= max{99.84 98.61} = 1.23

C
dd
= max{92.16 96.05} = 0
Using backwards induction, the value of the option at earlier nodes is:

C
u
=
1
1.01
[(0.625)(4.11) + (0.375)(0)] = 2.54

C
d
=
1
1.01
[(0.625)(1.23) + (0.375)(0)] = 0.76
Thus, the initial option price is

C =
1
1.01
[(0.625)(2.54) + (0.375)(0.76)] = 1.85
15. Consider a two-period binomial tree with the following parameters: S = 100, u = 1.10,
d = 0.90, and R = 1.03. Find the prices of:
(a) A knock-out call option with a strike of 95 and a barrier of 90.
Sundaram & Das: Derivatives - Problems and Solutions . . . . . . . . . . . . . . . . . . . . . . . . . . . . . . . 285
(b) A knock-in call option with a strike of 95 and a barrier of 90.
(c) A vanilla call option with a strike of 95.
Answer: The risk-neutral probability given these parameters is:
q =
R d
u d
= 0.65.
The terminal prices on the binomial tree are:
u
2
S = 121
udS = 99
duS = 99
d
2
S = 81
(a) The knock-out call pays max{S
T
95, 0} if the terminal price is reached without
the barrier of 90 being breached in the rst two periods, and zero otherwise. Thus,
the option is knocked out if the price goes down in the rst period but not if it
goes up. Its payos at maturity are:
C
ko
uu
= 26
C
ko
ud
= 4
C
ko
du
= 0
C
ko
dd
= 0
Using the risk-neutral probabilities, the values of the call at the nodes uS and dS
are, respectively,
C
ko
u
=
1
1.03
[(0.65)(26) + (0.35)(4)] = 17.77
C
ko
d
=
1
1.03
[(0.65)(0) + (0.35)(0)] = 0
Thus, the initial price of the option is
C
ko
=
1
1.03
[(0.65)(16.41) + (0.35)(0)] = 11.21
Sundaram & Das: Derivatives - Problems and Solutions . . . . . . . . . . . . . . . . . . . . . . . . . . . . . . . 286
(b) The knock-in call pays zero if the terminal price is reached without the barrier of
90 being breached in the rst two periods, and max{S
T
95, 0} otherwise. Thus,
its payos at maturity are:
C
ki
uu
= 0
C
ki
ud
= 0
C
ki
du
= 4
C
ki
dd
= 0
Using the risk-neutral probabilities, the values of the call at the nodes uS and dS
are, respectively,
C
ki
u
=
1
1.03
[(0.65)(0) + (0.35)(0)] = 0
C
ki
d
=
1
1.03
[(0.65)(4) + (0.35)(0)] = 2.52
Thus, the initial price of the option is
C
ki
=
1
1.03
[(0.65)(0) + (0.35)(2.52)] = 0.86
(c) A vanilla call with a strike of 95 has the following terminal payos:
C
uu
= 26
C
ud
= 4
C
du
= 4
C
dd
= 0
Using the risk-neutral probabilities, the values of the call at the nodes uS and dS
are, respectively,
C
u
=
1
1.03
[(0.65)(26) + (0.35)(4)] = 17.77
C
d
=
1
1.03
[(0.65)(4) + (0.35)(0)] = 2.52
Thus, the initial price of the option is
C =
1
1.03
[(0.65)(17.77) + (0.35)(2.52)] = 12.07
Sundaram & Das: Derivatives - Problems and Solutions . . . . . . . . . . . . . . . . . . . . . . . . . . . . . . . 287
16. Suppose you are given a two-period binomial tree with the following parameters: S =
100, R = 1.01, u = 1.05, and d = 0.95. Consider an option with a strike of K = 95
whose payo at maturity is
X
T
=
_
(S
T
K)
2
, if S
T
K
0, if S
T
< K
What is the initial price of this option?
Answer: The risk-neutral probability given these parameters is:
q =
R d
u d
= 0.60.
The terminal prices on the binomial tree are:
u
2
S = 110.25
udS = 99.75
duS = 99.75
d
2
S = 90.25
The payo from the option at these nodes is as follows:
X
uu
= (110.25 98)
2
= 150.0625
X
ud
= (99.75 98)
2
= 3.0625
X
du
= (99.75 98)
2
= 3.0625
X
dd
= 0
We can nd the initial option price either by backwards induction or more directly by
taking the expected value of the terminal payos under the risk-neutral probability and
discounting these back to the begining of the tree. We pursue the latter method here.
The risk-neutral probabilities of the four terminal nodes are, respectively, 0.36, 0.24,
0.24, and 0.16. Thus, the initial option price is
X =
_
1
1.02
_
2
[(0.36)(150.0625) + (0.24)(3.0625) + (0.24)(3.0625)] 53.34
Sundaram & Das: Derivatives - Problems and Solutions . . . . . . . . . . . . . . . . . . . . . . . . . . . . . . . 288
17. There have been many instances where US companies have backdated their employee
option grants to especially favorable dates, namely dates when the stock price was low
so as to set low strikes on their issued call options relative to current prices. What exotic
option does this practice resemble? Are there any dierences between this exotic option
and the backdated option grant?
Remark Executive stock options in the US (as in most of the world) are typically granted
at-the-money. Backdating refers to a practice, alleged to have been present at many
US companies until at least the the mid-2000s, in which companies that were granting
executives stock options looked back at their own stock prices over a given period (e.g.,
a year), and chose a low point over that period as the ostensible date of grant. This
ensures the grantee a low strike price and also makes the option in-the-money on the
true date of grant.
Answer: The backdating of employee stock options is evidently akin to granting em-
ployees a oating strike lookback option since they are given the benet of the most
favorable strike price over the lookback period, but there are some formal dierences.
In a oating-strike lookback, the maturity of the option is xed at issue time. In a
backdating setting, the option holder has a xed time (e.g., one year) over which to
look back and choose the lowest price as the strike; the option then has a maturity of
10 years from the date of this lowest strike which is the ostensible date of the grant.
18. How does a xed strike lookback options value change when the lookback period in-
creases?
Answer: As the lookback period increases, the option value must increase, as now there
is a greater period over which a favorable strike price may be found.
19. In a lookback option in general, the period over which the lookback occurs need not
coincide with the maturity of the option. Thus, a oating strike lookback option has two
periods in it: (i) the maturity of the option itself, i.e., T, and (ii) the lookback period
T. How does the value of the option change when
(a) Maturity T increases, keeping xed?
(b) Holding T xed, the lookback period increases (but does not exceed T)?
(c) Maturity T decreases, but increases (but does not exceed T)?
(d) Maturity T increases, and increases (but does not exceed T)?
Answer:
(a) As the maturity T increases, the price of the stock experiences a drift upwards on
average. If is kept xed, then the expected ration of the nal price to the strike
Sundaram & Das: Derivatives - Problems and Solutions . . . . . . . . . . . . . . . . . . . . . . . . . . . . . . . 289
price in the case of a oating strike call remains the same, i.e., S
T
/S
min
is invariant
to T given remains xed. But, if the ratio S
T
/S
min
is xed, and S
T
increases,
then the dierence S
T
S
min
increases, and results in a higher payo on average.
Therefore, if is held xed and T increases, the value of a oating strike call will
increase. Likewise, an analogous argument can be made to show that a oating
strike lookback put will also increase in value.
(b) Keeping T xed, the expected nal price S
T
remains the same. Any increase then
in the lookback period will result in a possibly more favorable strike price, and
so the oating strike lookback option (call or put) will increase in value.
(c) This case is a mixture of the two eects noted in the prior to subquestions. It is
easy to see that the outcome here is indeterminate.
(d) Here the eects are the same as (a) and (b) combined resulting in an increase in
option value.
20. Employee stock options are often reset in their strike when the stock price of the company
has declined over a period of time. That is, although the options are initially issued at-
the-money, their strike prices are lowered if they become deep underwater, that is, if a
sharp decline in stock prices has taken place since the initial grant. Why do you think
companies reset their employee option strikes? What path-dependent option have you
learned about that most closely resembles this practice?
Answer: Firms that use options as employee incentives nd that when these options
are too far out of the money, the incentive eects are lost. Hence, to reinstate these
incentives, companies sometimes reset the option strikes to the current prevailing stock
price. If, for example, a company resets the strike price when the stock price is 30%
below the price on the date of grant, then the initial options is like a package of two
barrier options, one that gets knocked out when the stock price drops 30% from its
initial level, and the other that gets knocked-in when the stock price drops 30% from
its initial level (and is at-the-money when it comes to life).
For more on the valuation and incentive eects of option resetting, see, respectively,
Brenner, Sundaram, and Yermack (2000, Journal of Financial Economics 57, pp.103-
128) and Acharya, John, and Sundaram (2000, Journal of Financial Economics 57,
pp.65-101).
21. Using a three-period binomial tree, value a down-and-out call option. The parameters
you are given are the following: the initial stock price is $100, the strike price is $105,
the barrier is $90, the risk-free rate per-period is 5%, the option maturity is three years,
and the volatility of the stock is 40%. Use the CRR method to construct the binomial
tree.
Sundaram & Das: Derivatives - Problems and Solutions . . . . . . . . . . . . . . . . . . . . . . . . . . . . . . . 290
Answer: This question is one illustration of how some path-dependent payos may be
priced by backwards recursion. (This is not possible for many path-dependent options,
for example, Asian options, or even knock-in barrier options.) We can identify the
problem using a simple program. The program and output are provided below. We
present the stock price tree as well as the tree of option values.
%Program for barrier options on a binomial tree
function val=crr_barrier(s0,k,t,v,rf,H,reb,pc,n);
s = zeros(n+1,n+1); s(1,1)=s0;
dt = t/n;
u = exp(v*sqrt(dt));
d = exp(-v*sqrt(dt));
r = exp(rf*dt);
q = (r-d)/(u-d);
for j=2:n+1;
s(1,j) = s(1,j-1)*u;
for i=2:j;
s(i,j) = s(i-1,j-1)*d;
end;
end;
optval = zeros(n+1,n+1);
for i=1:n+1;
if pc==1; optval(i,n+1) = max(0,s(i,n+1)-k); end;
if pc==0; optval(i,n+1) = max(0,k-s(i,n+1)); end;
end;
for j=n:-1:1;
for i=1:j;
optval(i,j) = (q*optval(i,j+1)+(1-q)*optval(i+1,j+1))/r;
if s(i,j)<=H; optval(i,j)=reb; end;
end;
end;
s
optval
val = optval(1,1);
Sundaram & Das: Derivatives - Problems and Solutions . . . . . . . . . . . . . . . . . . . . . . . . . . . . . . . 291
-------------- OUTPUT -----------------
octave:1> crr_barrier(100,105,3,0.4,0.05,90,0,1,3)
s =
100.00000 149.18247 222.55409 332.01169
0.00000 67.03200 100.00000 149.18247
0.00000 0.00000 44.93290 67.03200
0.00000 0.00000 0.00000 30.11942
optval =
28.25502 64.05474 122.67500 227.01169
0.00000 0.00000 19.48922 44.18247
0.00000 0.00000 0.00000 0.00000
0.00000 0.00000 0.00000 0.00000
ans = 28.255
The input reb refers to the rebate if the option gets knocked out. In the computations,
this is set to zero. The output is to be read in the obvious way. In the rst table, the
initial stock price is 100, the possible prices after one period are 149.18247 and 67.03200
respectively, etc.
Note that in the second period, at the lower node, the value of the option has been set
to zero, since it has been knocked out (the stock price at this node is $67.03 which is
less than the barrier of 90.
22. In the above question, what would we do to the model to make it default to a vanilla
call option? Would the vanilla call be worth more or less than the barrier option?
Answer: To default to a vanilla call, lower the barrier the barrier to a level the stock
price can never reach. For example, suppose we set the barrier to H = 0. This would
just be the standard Black-Scholes option. We get:
octave:2> crr_barrier(100,105,3,0.4,0.05,0,0,1,3)
s =
100.00000 149.18247 222.55409 332.01169
0.00000 67.03200 100.00000 149.18247
0.00000 0.00000 44.93290 67.03200
Sundaram & Das: Derivatives - Problems and Solutions . . . . . . . . . . . . . . . . . . . . . . . . . . . . . . . 292
0.00000 0.00000 0.00000 30.11942
optval =
32.64046 64.05474 122.67500 227.01169
0.00000 8.59684 19.48922 44.18247
0.00000 0.00000 0.00000 0.00000
0.00000 0.00000 0.00000 0.00000
ans = 32.640
As expected, the price of the option is higher with no barrier.
23. (Requires Writing Code) The same barrier option pricing problem in the last two ques-
tions may be solved using the recursive programming approach developed in Chapter
13. Reprogram the pricing model using such an approach. Check that the barrier option
price is the same. Use the same parameter values.
Answer: The program is easy to write and is in fact shorter when written in recursive
form. It is of course, ne for small trees, but becomes computationally slow for big trees.
%Recursive program to price barrier options
function val = crr_rec_barrier(s,k,t,v,rf,H,pc,n);
if n==0;
if pc==1; optval=max(0,s-k); end;
if pc==0; optval=max(0,k-s); end;
elseif s<=H;
optval = 0;
else
h = t/n;
u = exp(v*sqrt(h));
d = exp(-v*sqrt(h));
r = exp(rf*h);
q = (r-d)/(u-d);
optval = (q*crr_rec_barrier(s*u,k,t-h,v,rf,H,pc,n-1)+ ...
(1-q)*crr_rec_barrier(s*d,k,t-h,v,rf,H,pc,n-1))/r;
end;
val = optval;
------- OUTPUT ------
Sundaram & Das: Derivatives - Problems and Solutions . . . . . . . . . . . . . . . . . . . . . . . . . . . . . . . 293
octave:5> crr_rec_barrier(100,105,3,0.4,0.05,90,1,3)
ans = 28.255
octave:6> crr_rec_barrier(100,105,3,0.4,0.05,0,1,3)
ans = 32.640
As we can see values are the same as that obtained from the simple recombining CRR
tree of the earlier questions.
24. A reverse cliquet with quarterly resets pays an annual coupon of 15% less the return
during any quarter in which the return is negative. The index is at 1000 at the beginning
of the year and takes values of 900, 1000, 1100, and 1050 at the end of each of the
succeeding quarters. What is the nal payo of this option?
Answer: The return in the four quarters is:
10%, +11.11%, +10%, 4.55%
Hence the payo of the cliquet will be
15%10%4.55% = 0.45%
25. What has higher value for options of strike K that are at-the-money at inception:
(a) A cliquet option with maturity T = 1 year and a single reset at T = 1/2 year?
(b) A one-shout shout option with maturity T = 1 year?
Answer: A shout option allows the buyer to exercise at any time T

to lock in gains
S
T
K and then at maturity if the stock price is below S
T
the lock in value is the
payo. If the nal price S
T
> S
T
, then the payo is S
T
K instead. Notice that
a one-click cliquet is the same as a shout option except that it has a xed reset date,
and the shout option allows the buyer to shout at any time. The additional exibility
therefore makes the shout option more valuable.
26. (Requires Writing Code) (a) Write a program to price a down-and-in barrier call option
with the following parameters: S = 50, strike K = 50, an annual risk-free rate r = 3%,
volatility = 0.3, and barrier H = 45. Use a CRR pricing tree with a time step of one
year for each period. The maturity of the option is six years.
(b) Modify the program to price the down-and-in put.
(c) Does European put-call parity hold for barrier options? Why?
Answer: Here is the recursive program code in Octave:
Sundaram & Das: Derivatives - Problems and Solutions . . . . . . . . . . . . . . . . . . . . . . . . . . . . . . . 294
%Recursive program to price knock in barrier options
function val = crr_rec_barrier_in(s,k,t,v,rf,H,inflag,pc,n);
if s<=H; inflag = 1; end;
if n==0;
if pc==1; optval=max(0,s-k)*inflag; end;
if pc==0; optval=max(0,k-s)*inflag; end;
else
h = t/n;
u = exp(v*sqrt(h));
d = exp(-v*sqrt(h));
r = exp(rf*h);
q = (r-d)/(u-d);
optval = (q*crr_rec_barrier_in(s*u,k,t-h,v,rf,H,inflag,pc,n-1)+ ...
(1-q)*crr_rec_barrier_in(s*d,k,t-h,v,rf,H,inflag,pc,n-1))/r;
end;
val = optval;
(a) & (b) The call and put values for the down and in options are:
octave:10> crr_rec_barrier_in(50,50,6,0.3,0.03,45,0,1,6)
ans = 4.1341
octave:11> crr_rec_barrier_in(50,50,6,0.3,0.03,45,0,0,6)
ans = 8.8643
Can you explain intuitively why the call is worth so much less than the put?
(c) We can check put-call parity which is:
C P = S Ke
rn
We have
4.1341 8.8643 = 50 50e
0.036/12
Hence, the usual European option put-call parity condition does not hold.
27. (a) Price a ten-year down-and-in barrier call option with the following parameters:
S = 100, strike K = 102, annual risk-free rate r = 2%, volatility = 0.4, and
barrier H = 90. Use a time step of one year on the CRR tree for this problem.
(b) Price the down-and-out barrier call with the same parameters.
Sundaram & Das: Derivatives - Problems and Solutions . . . . . . . . . . . . . . . . . . . . . . . . . . . . . . . 295
(c) Price the vanilla call with the same parameters.
(d) Verify that the sum of the prices you obtain in parts (a) and (b) equals the price
you obtain in part (c).
Answer: We used the program code from the preceding questions to solve these prob-
lems:
octave:14> knockout = crr_rec_barrier(100,102,10,0.4,0.02,90,1,10)
ans = 33.052
octave:15> knockin = crr_rec_barrier_in(100,102,10,0.4,0.02,90,0,1,10)
ans = 18.081
octave:16> vailla = crr_rec_barrier(100,102,10,0.4,0.02,0,1,10)
ans = 51.132
We can see that the knock-out option plus the knock-in option equals the value of the
vanilla call.
28. Consider a shout option with strike K. One way to price the option is as follows. If at
some point in time prior to maturity, you shout when the stock price is S > K, then you
capture the prots SK (to be paid at maturity) and the original shout call option held
is replaced with a new vanilla call option with a strike of S for the remaining maturity.
(a) Explain how you would use a binomial tree to price this option.
(b) Will the tree be recombining? That is, can the option be priced by backwards
recursion?
(c) Is the option path-dependent?
Answer:
(a) The process for solving the pricing problem is as follows. (i) Set up the usual
CRR tree. (ii) Go to maturity and determine the payos for a vanilla call option.
(iii) Perform backward recursion to the preceding period on the tree. (iv) Now
compare the value at each node from backward recursion with the sum of the
shout value (S K, discounted for the period from now to maturity, since the
value is paid only at maturity) plus a vanilla call at strike K (priced using Black-
Scholes since it is European). (v) If the sum of the value of immediate shouting,
i.e., (S K)e
r
+ Call[S, K] is greater than the recursion present value, we
should shout and set the node value equal to the shout value. Finish the backward
recursion to obtain the current value of the shout option.
(b) Yes, the tree can be recombining, since only backward recursion is required.
Sundaram & Das: Derivatives - Problems and Solutions . . . . . . . . . . . . . . . . . . . . . . . . . . . . . . . 296
(c) Yes it is. The payo at maturity depends on when you shouted and that depends
on the path. (Note that even though this option is path-dependent, it can still be
valued by backward recursion).
29. Price a ten-year Asian option with an initial stock price of $50, strike $50, annual risk-
free rate r = 10%, and volatility = 0.35. Price the call and the put, and see whether
put-call parity holds. Use a pricing tree with a time step of one year per period.
Answer: This option is most certainly path-dependent since its nal payo depends on
the average price of the stock along the sample path. Hence, at a given node, the
average stock price is dierent if the stock went up and then down versus down and
then up. Hence, one way to price this option is to use a recursive model. The following
Octave program code presents the pricing model.
%Recursive program to price Asian options
function u = crr_rec_asian(s,k,t,v,rf,pc,n,totn,sum_p);
if n==0;
if pc==1; optval=max(0,sum_p/totn - k); end;
if pc==0; optval=max(0,k - sum_p/totn); end;
else
h = t/n;
u = exp(v*sqrt(h));
d = exp(-v*sqrt(h));
r = exp(rf*h);
q = (r-d)/(u-d);
optval = (q*crr_rec_asian(s*u,k,t-h,v,rf,pc,n-1,totn, sum_p+s*u)+ ...
(1-q)*crr_rec_asian(s*d,k,t-h,v,rf,pc,n-1,totn,sum_p+s*d))/r;
end;
u = optval;
Running the program gives the following values for the call and the put:
octave:1> call = crr_rec_asian(50,50,10,0.35,0.10,1,10,10,0)
ans = 16.779
octave:2> put = crr_rec_asian(50,50,10,0.35,0.10,0,10,10,0)
ans = 1.9605
octave:5> call - put
ans = 14.819
octave:6> 50 - 50*exp(-0.1*10)
ans = 31.606
Sundaram & Das: Derivatives - Problems and Solutions . . . . . . . . . . . . . . . . . . . . . . . . . . . . . . . 297
Also from the check we can see that put-call parity does not hold for Asian options.
30. (Requires Writing Code) For the same parameters as in the previous question, price a
lookback price call and put.
Answer: As in the previous question, this can easily be written in recursive form. We
note again that the model requires inspection of each specic path for maxima and
minima.
The program code in Octave is as follows:
%Recursive program to price Lookback options
function u = crr_rec_lookback(s,k,t,v,rf,pc,n,xs);
if pc==1 && s>xs; xs=s; end;
if pc==0 && s<xs; xs=s; end;
if n==0;
if pc==1; optval=max(0,xs - k); end;
if pc==0; optval=max(0,k - xs); end;
else
h = t/n;
u = exp(v*sqrt(h));
d = exp(-v*sqrt(h));
r = exp(rf*h);
q = (r-d)/(u-d);
optval = (q*crr_rec_lookback(s*u,k,t-h,v,rf,pc,n-1,xs)+ ...
(1-q)*crr_rec_lookback(s*d,k,t-h,v,rf,pc,n-1,xs))/r;
end;
u = optval;
The results for the call and put are:
octave:9> call = crr_rec_lookback(50,50,10,0.35,0.10,1,10,0)
ans = 47.451
octave:10> put = crr_rec_lookback(50,50,10,0.35,0.10,0,10,0)
ans = 18.394
31. (Requires Writing Code) Implement the formula in the chapter appendix for up-and-out
calls, and value the option for the following parameters: S = 102, K = 100, r = 5%
p.a., barrier H = 130, and maturity T = 0.25. Annualized volatility is = 40%.
Sundaram & Das: Derivatives - Problems and Solutions . . . . . . . . . . . . . . . . . . . . . . . . . . . . . . . 298
Program a tree model to do the same and report the value. Compare the tree model to
the closed-form answer.
Answer: The solution to the problem using the closed-form solution is $3.17. The
corresponding Black-Scholes vanilla call price is $9.72.
To price the option using a recursive tree we have:
%Recursive program to price barrier options
%Set outflag to 1 at start, it goes to zero if we hit barrier
function val = crr_rec_barrier(s,k,t,v,rf,H,outflag,pc,n);
if s>=H; outflag = 0; end;
if n==0;
if pc==1; optval=max(0,s-k)*outflag; end;
if pc==0; optval=max(0,k-s)*outflag; end;
else
h = t/n;
u = exp(v*sqrt(h));
d = exp(-v*sqrt(h));
r = exp(rf*h);
q = (r-d)/(u-d);
optval = (q*crr_rec_barrier(s*u,k,t-h,v,rf,H,outflag,pc,n-1)+ ...
(1-q)*crr_rec_barrier(s*d,k,t-h,v,rf,H,outflag,pc,n-1))/r;
end;
val = optval;
The barrier option tends to be quite sensitive to the number of steps in the tree because
it determines which nodes lie on either side of the barrier. The discreteness of the
binomial tree means that the placement of the barrier is fairly crucial. We computed
the option value for dierent values of n, the number of steps in the tree:
octave:18> crr_rec_barrier(102,100,0.25,0.4,0.05,130,1,1,10)
ans = 2.9129
octave:19> crr_rec_barrier(102,100,0.25,0.4,0.05,130,1,1,11)
ans = 4.6394
octave:20> crr_rec_barrier(102,100,0.25,0.4,0.05,130,1,1,12)
ans = 4.7272
octave:21> crr_rec_barrier(102,100,0.25,0.4,0.05,130,1,1,13)
ans = 3.9192
octave:22> crr_rec_barrier(102,100,0.25,0.4,0.05,130,1,1,14)
ans = 4.0316
octave:23> crr_rec_barrier(102,100,0.25,0.4,0.05,130,1,1,15)
ans = 3.3540
Sundaram & Das: Derivatives - Problems and Solutions . . . . . . . . . . . . . . . . . . . . . . . . . . . . . . . 299
The sequence of values demonstrates the nature of approximation error injected into the
pricing model by discreteness. It is very severe for barrier options, and the value of the
closed form equations cannot be understated.
32. In the preceding question, what is the eect of increasing the maturity of the up-and-out
call? Keep the parameters the same but vary the maturity to take the following values
(in years): 0.1, 0.2, 0.3, 0.5, and 1.0.
Answer: As maturity increases, the likelihood of being knocked out increases as well,
resulting in a decline in the value of the option. This reduction more than osets the
greater eect of volatility. The following table shows the option value against maturity:
Maturity (yrs) Up & Out Call
0.1 4.84
0.2 3.70
0.3 2.73
0.5 1.65
1.0 0.71
33. Consider a more complex form of barrier option, the double barrier knock-out call. In
this case, the option gets knocked out if either barrier is breached during the options life.
For this option, using any method of your choosing, price the option for the following
parameters: S = 100, K = 100, lower barrier 80, upper barrier 120, maturity 0.25 years,
risk-free rate of 6% p.a., zero dividends, and volatility of 35%. Answer the following
questions:
(a) What is the price of this option?
(b) What is the price at S = 95 and at S = 105?
(c) What is the price at a volatility of 50%? What does this tell you about the sign of
the vega?
(d) What is the delta of the option at stock prices 95, 100, and 105?
Answer:
(a) Price = 1.46.
(b) Call at 95, price = 1.33; at 105, call = 1.35. We can see that the call price drops
as the stock price moves towards the barriers.
(c) The price drops to 0.53 as the probability of knock out increases. Hence, the vega
will be negative.
(d) Delta at 95 = 0.04345, delta at 100 = -0.0021, delta at 105 = -0.0488.
Sundaram & Das: Derivatives - Problems and Solutions . . . . . . . . . . . . . . . . . . . . . . . . . . . . . . . 300
34. Using a closed-form solution, price the up-and-in put option with the following parame-
ters: S = 100, K = 100, barrier H = 110, rebate of 50, maturity of one year, annual
risk-free interest rate of 3%, and stock volatility of 40%. There are no dividends.
(a) What is the price of the option?
(b) What is the option price if the stock rises to 105?
(c) What is the option price if the stock rises to 109?
(d) What can you say about the sign of the delta and the gamma?
Answer: (a) Price = 17.71.
(b) Put at S = 105 = 14.12.
(c) Put at S = 109 = 11.38.
(d) The delta is negative and the gamma is positive.
35. The current stock price is $100. Price a half-year average strike Asian call option if the
stock volatility is 30%, and the annual risk-free rate is 10%. Use a tree model with six
monthly steps. Compare the price you arrive at with the price of an otherwise identical
average price Asian call at a strike of $100. State intuitively why the prices are dierent.
Answer: An earlier question provided a recursive binomial tree for pricing average price
Asian options. Using that model the price of an average price Asian call works out to
be $6.9646.
Here is the code for the average strike Asian option:
%Recursive program to price Asian options with average strike
function u = crr_rec_asian_avgk(s,k,t,v,rf,pc,n,totn,sum_p);
if n==0;
if pc==1; optval=max(0,s - sum_p/totn); end;
if pc==0; optval=max(0,sum_p/totn - s); end;
else
h = t/n;
u = exp(v*sqrt(h));
d = exp(-v*sqrt(h));
r = exp(rf*h);
q = (r-d)/(u-d);
optval = (q*crr_rec_asian_avgk(s*u,k,t-h,v,rf,pc,n-1,totn, sum_p+s*u)+ ...
(1-q)*crr_rec_asian_avgk(s*d,k,t-h,v,rf,pc,n-1,totn,sum_p+s*d))/r;
end;
u = optval;
Sundaram & Das: Derivatives - Problems and Solutions . . . . . . . . . . . . . . . . . . . . . . . . . . . . . . . 301
We use the model in Octave to arrive at the following price:
octave-3.0.1:9> crr_rec_asian_avgk(100,100,0.5,0.3,0.1,1,6,6,0)
ans = 5.4441
Hence, the value of the average strike option is $5.44, lower than the price of the
standard Asian call of $6.95. Intuitively, in this case, when the stock price is rising, in
the average strike option, the strike is also rising, thereby osetting some of the payo
gains. This does not occur with an average price call, which is therefore of greater value.
36. Using the same parameters as in Question 35, calculate the prices of Asian puts of both
types, average price and average strike. Compare the prices. Explain why one is higher
than the other.
Answer: Using the same program code as in previous questions, the average price Asian
put is worth $4.14 and the average strike one is worth $3.39. Again the average strike
option is worth less. When the price is dropping the strike is also dropping, making it
less valuable than the average price option where the strike remains xed.
37. Repeat Question 35 but for a stock price of $90, leaving all other parameters unchanged.
Value the average price and average strike options and compare their prices.
Answer: The average price Asian call is worth $2.33, and the average strike Asian is
worth $4.90. Now, the average strike option is worth more! This is because starting at
a stock price of $90 means that the average stock price (which becomes the strike) is
on average less than $100, the xed strike.
38. The current stock price is $100. Price a half-year oating strike lookback call option if
the stock volatility is 30%, and the annual risk-free rate is 10%. Use a tree model with
six monthly steps. Also price the lookback put.
Answer: We build a recursive binomial tree to calculate the option values. The payo
for the oating strike lookback call is
S
T
min[S
0
, S
min
]
and for the lookback put the payo is
max[S
0
, S
max
] S
T
The Octave program is as follows:
Sundaram & Das: Derivatives - Problems and Solutions . . . . . . . . . . . . . . . . . . . . . . . . . . . . . . . 302
%Recursive program to price Lookback options (floating strike type)
function u = crr_rec_lookback_avgk(s0,s,t,v,rf,pc,n,xs);
if pc==0 && s>xs; xs=s; end;
if pc==1 && s<xs; xs=s; end;
if n==0;
if pc==1; optval=s - min(s0,xs); end;
if pc==0; optval=max(s0,xs) - s; end;
else
h = t/n;
u = exp(v*sqrt(h));
d = exp(-v*sqrt(h));
r = exp(rf*h);
q = (r-d)/(u-d);
optval = (q*crr_rec_lookback_avgk(s0,s*u,t-h,v,rf,pc,n-1,xs)+ ...
(1-q)*crr_rec_lookback_avgk(s0,s*d,t-h,v,rf,pc,n-1,xs))/r;
end;
u = optval;
We execute this program as follows:
octave-3.0.1:19> crr_rec_lookback_avgk(100,100,0.5,0.3,0.1,1,6,100)
ans = 14.973
octave-3.0.1:20> crr_rec_lookback_avgk(100,100,0.5,0.3,0.1,0,6,100)
ans = 11.158
Hence, the price of the lookback call is $14.97 and that of the lookback put is $11.16.
For comparison, a plain vanilla call is priced at $10.563, and a xed strike lookback call
is priced at $16.035.

Das könnte Ihnen auch gefallen